ENT, POISONING, Cardiovascular system (1), 1. Allergy and Immunology, MALE REPRODUCTIVE SYSTEM, Biostatics and Epidemiology, General principles, Gastrointestinal and nutritional, Female Reproductive System & Breast (1), Haematology and Oncology

Ace your homework & exams now with Quizwiz!

43. A 6-year-old boy is brought to the office for a well-child visit. His weight and height are at the 15th percentile for age and sex. Blood pressure is 94/52 mm Hg, pulse is 92/min, and respirations are 16/min. Pulse oximetry is 99% on room air. Auscultation shows a normal S1 and a split S2 that does not vary with respiration. A 1/6 systolic ejection murmur is heard. Which of the following is the most likely diagnosis? A. Atrial septal defect B.Coarctation of the aorta C. Patent ductus arteriosus D. Tetralogy of Fallot E. Ventricular septal defect

Answer A Educational objective: Atrial septal defects classically present as a split, fixed S2 that does not vary with respiration and a pulmonary flow murmur. Most of these defects close spontaneously by age 5.

18. A 46-year-old man comes to the office due to 2 weeks of hoarseness. He has no fever, cough, sore throat, or difficulty swallowing. He has seasonal allergies and asthma, which has been difficult to control despite multiple medications. Which of the following medications is most likely responsible for this patient's symptoms? A. Inhaled fluticasone B. Inhaled salmeterol C. Oral theophylline D. Oral zafirlukast E. Oral zileuton

Answer A Educational objective: Hoarseness is a change in voice quality that can be due to laryngeal or vocal cord pathology, neurological conditions, or medications. Inhaled steroids are the most common medications associated with hoarseness.

56. A 34-year-old man comes to the emergency department with weakness, nausea, and abdominal discomfort. He has a history of chronic back pain and polysubstance abuse. Laboratory studies show markedly elevated aminotransferase levels (>3500 U/L), high normal total bilirubin, and elevated INR (2.1).Which of the following is the most likely diagnosis? A. Acetaminophen overdose B. Alcoholic hepatitis C. Hepatitis C infection D. Nonalcoholic steatohepatitis E. Reye syndrome

Answer A Educational objective: Patients presenting with markedly elevated hepatic aminotransferases (>3500 U/L) should raise suspicion for acetaminophen toxicity. N-acetylcysteine is the antidote of choice.

9. A large, randomized, placebo controlled study evaluated the efficacy of a statin in preventing myocardial infarction in patients with moderate hyperlipidemia and no history of coronary artery disease. Patients who took the statin had a relative risk of 0.67 of having a myocardial infarction. The absolute risk was 7.5% in the placebo group and 5.0% in the treatment group. What was the relative risk reduction? A. 2.5% B. 33% C. 40% D.50% E. 67%

Answer B Educational objective: Relative risk reduction (RRR) can be calculated in 2 equivalent ways: 1. RRR = (risk in unexposed - risk in exposed) / (risk in unexposed) 2. RRR = 1 - relative risk

51. Which of the following is the best therapy for a patient with a recurrence of Clostridioides difficile colitis if the first episode was treated with oral vancomycin? A. Intravenous metronidazole B. Oral fidaxomicin C. Oral metronidazol D. Oral metronidazole plus oral vancomycin E. Oral rifaximin

Answer B Educational objective: An initial recurrence of Clostridioides difficile infection should be treated with a course of fidaxomicin or a prolonged pulsed and tapered course of oral vancomycin. Adjunctive therapy with bezlotoxumab is considered for patients with an especially high recurrence risk.

2. A 65-year-old woman with hypertension comes to the physician due to unsteadiness and near-falls over the last year. She usually walks to the bathroom at night but recently has needed to hold onto the hallway walls to maintain her balance. The patient is concerned that she has also become more forgetful than usual. On neurologic examination, position and vibration sensation is diminished in both feet and there is a slight decrease in light touch sensation. The rest of the examination is unremarkable. Laboratory testing shows a hemoglobin A1C of 5.2%, TSH of 3.5 µU/mL, and vitamin B12 level of 280 pg/mL (low <200 pg/mL, normal >300 pg/mL). Which of the following is the most appropriate next step in management? A. Antinuclear antibody B. CT scan of the head C. Methylmalonic acid level D. Oral glucose tolerance test E. Serum protein electrophoresis

Answer C Educational objective: Symptomatic vitamin B12 deficiency may be present even if serum B12 levels are borderline. In this setting, serum methylmalonic acid measurement is an important adjunctive test as elevated levels confirm the diagnosis of B12 deficiency.

22. A 38-year-old woman suffering from constipation is evaluated for occasional blood in the stool over the last 6 months. She describes bright red blood coating the stool and on several occasions dripping into the toilet at the end of defecation. The patient has no abdominal or rectal pain, nausea, vomiting, or weight loss. She has never had a colonoscopy and has no family history of cancer. Further evaluation is most likely to show which of the following as the cause of her symptoms? A. Anal fissure B. Colonic villous adenoma C. Internal hemorrhoids D. Rectal ulcer E. Sigmoid diverticulosis

Answer C Educational objective: Hemorrhoids typically present with painless rectal bleeding associated with pruritus or fecal soilage. Anoscopy can confirm the diagnosis in patients age <40 without red flags; however, patients age 40-49 without red flags should at minimum undergo sigmoidoscopy. Complete colonoscopy should be considered if a bleeding source is not identified, the patient has red flags, or the patient is age >50.

28. A 23-year-old man comes to the emergency department with diarrhea, nausea, and colicky abdominal pain for the past 5 days. The diarrhea occurs 5-6 times a day; it was initially watery but since yesterday has become bloody. The patient's past medical history is insignificant, and he has no history of recent travel. His temperature is 36.6 C (98.0 F), blood pressure is 123/82 mm Hg, and pulse is 102/min. He has prominent periumbilical and right lower-quadrant tenderness but no guarding or rebound. Rectal examination shows brownish stool mixed with blood.Which of the following is the most likely etiology in this patient? A. Clostridium difficile B. Clostridium perfringens C. Escherichia coli D. Rotavirus E. Staphylococcus aureus

Answer C Educational objective: Enterohemorrhagic Escherichia coli (EHEC) is a food-borne pathogen that causes acute watery to bloody diarrhea. Diagnosis can be confirmed with a stool assay for Shiga toxin. If EHEC is suspected, empiric antibiotic therapy should be avoided as it may increase the risk of hemolytic uremic syndrome.

7. A 48-year-old woman comes to the physician with 5 days of dizziness accompanied by nausea. She describes the dizziness as brief episodes of a spinning sensation triggered by certain head movements, such as suddenly looking up. The patient has no fever, headache, earache, or changes in hearing. She had similar symptoms several months ago that resolved spontaneously. Which of the following will most likely improve her symptoms? A. Balance training exercises B. Low-dose aspirin C. Low-dose prednisone D. Particle repositioning maneuvers E. Salt-restricted diet

Answer D Educational objective: Benign paroxysmal positional vertigo is characterized by recurrent brief (<1 minute) episodes of vertigo and nausea triggered by changes in head position. Particle repositioning maneuvers (eg, Epley maneuver) are recommended for initial treatment although most cases resolve spontaneously.

8. Researchers examined medical records from 15 years ago and identified women who had low bone mineral density (BMD) and women who had normal BMD. They then reviewed the records and determined the incidence of hip fracture in those 2 groups over time. Which of the following best describes this study design? A. Case-control study B. Cross-sectional study C. Prospective cohort study D. Retrospective cohort study

Answer D Educational objective: In determining study design, it is useful to ask whether the investigators are beginning with the outcome/disease or the exposure/risk factor of interest.

21. A recent study of patients with stage 2 chronic kidney disease (CKD) found that a new drug reduced the progression to end-stage (stage 5) CKD from 37/1,000 placebo-treated cases to 12/1,000 new drug-treated cases over a five-year period. Which of the following is the number needed to treat in order to obtain a single favorable outcome? A. 2.5 B. 10 C. 25 D. 40 E. 100

Answer D Educational objective: The number needed to treat is the reciprocal of the absolute risk reduction rate from an intervention. This number is used by some clinicians to present probabilities regarding outcomes to patients.

48. A 63-year-old man comes to the physician for intermittent left calf pain that is exacerbated by walking more than 200 m (656 ft) and relieved with rest. He has hypertension and has smoked 1 pack of cigarettes daily for 50 years. Current medications include low-dose aspirin, lisinopril, and chlorthalidone. The patient's serum glucose, creatinine, and aspartate aminotransferase are normal. Total cholesterol is 180 mg/dL with LDL cholesterol of 95 mg/dL and triglycerides of 190 mg/dL. The ankle brachial index is 0.72 on the left and 0.95 on the right (normal 1.0-1.3). He is referred for a supervised exercise program. Which of the following medications is also recommended for this patient? A. Cilostazol B. Clopidogrel C. Gemfibrozil D.Lovastatin E. Rosuvastatin

Answer E Educational objective: Statin therapy is recommended for all patients with clinically significant atherosclerotic cardiovascular disease. This includes symptomatic coronary artery disease, peripheral arterial disease, stroke/transient ischemic attack, and arterial revascularization.

42. A 42-year-old woman comes to the emergency department due to fever, chills, and lower abdominal pain. The patient has had no previous surgeries. Temperature is 39 C (102.2 F), blood pressure is 78/56 mm Hg, and pulse is 118/min. The patient has right lower quadrant tenderness, but no rebound or guarding. Pelvic examination is limited by patient discomfort but does reveal right adnexal tenderness. Ultrasound reveals a complex, multilocular adnexal mass with normal Doppler flow. Pregnancy test is negative. Which of the following is the most likely diagnosis in this patient? A. Ectopic pregnancy B. Ovarian torsion C. Polycystic ovary syndrome D. Ruptured ovarian cyst E. Tubo-ovarian abscess

Answer E Educational objective: Tubo-ovarian abscess typically presents with fever, lower abdominal pain, and a complex adnexal mass with normal Doppler flow. Due to the risk of abscess rupture, patients require inpatient management with broad-spectrum antibiotics (eg, cefoxitin plus doxycycline).

15. A 38-year-old woman at 28 weeks gestation comes to the emergency department with left leg pain. The patient has no chronic medical conditions and her pregnancy has been uncomplicated. On physical examination, the patient has diffuse pain, swelling, erythema, warmth, and tenderness of the left lower extremity. Compressive ultrasonography shows a large thrombus in the femoral vein. Complete blood count and creatinine are normal. Which of the following is the best treatment option for this patient? A. Direct oral anticoagulant B. Inferior vena cava filter C. Low-molecular-weight heparin D. Unfractionated heparin E. Warfarin

Answer C Educational objective: Low-molecular-weight heparin (LMWH) is the preferred anticoagulant for most patients throughout pregnancy due to its ease of administration, predictable anticoagulant effect, and lack of need for frequent monitoring. Unfractionated heparin (UFH) is an alternative in those with renal insufficiency. Patients should be converted from LMWH to UFH at term.

41. A 66-year-old woman comes to the clinic with frequent heartburn. She attributes it to her medications for hypertension (lisinopril and felodipine) but does not wish to stop these as she has been intolerant of other blood pressure medications in the past. The patient has tried over-the-counter antacids and H2 blockers without relief. Her other medical problems include mild, persistent asthma. The patient has smoked a half pack of cigarettes daily for the last 45 years. Her blood pressure is 144/90 mm Hg, and heart rate is 64/min. Her BMI is 20 kg/m 2 .Before starting pantoprazole which of the following risks should be discussed with this patient? A. Acute coronary events B. Asthma exacerbation C. Bone loss and fractures D. Gallstones E. Rise in serum creatinine

ANSWER C Educational objective: Long-term proton pump inhibitor therapy may be associated with bone loss and fractures. Other side effects include atrophic gastritis, B12 and iron malabsorption, hypomagnesemia, and increased risk of certain infections such as Clostridiodes difficile colitis and pneumonia.

4. A study evaluated the utility of MRI in men with persistent mild elevations in prostate-specific antigen levels and a negative transrectal ultrasound-guided prostate biopsy. Study participants underwent an MRI followed by a second biopsy (considered the gold standard). Abnormal MRI findings suggestive of prostate cancer had a sensitivity of 99%, a specificity of 38%, a negative predictive value of 100%, and a positive predictive value of 27%. For a patient in this study sample who had abnormal MRI findings, what was the probability of finding prostate cancer on biopsy? A. 27% B. 38% C. 99% D. 100%

Answer A Educational objective: The positive predictive value of a test is the probability that a disease is present given a positive test result.

19. Which of the following parameters represents the probability that a patient with a negative result on a screening test for a certain condition does not actually have that condition? A. Negative likelihood ratio B. Negative predictive value C. Positive likelihood ratio D. Positive predictive value E. Specificity

Answer B Educational objective: The negative predictive value represents the probability that a patient with a negative result on a screening test for a certain condition truly does not have that condition.

4. A 52-year-old man comes to the physician due to palpitations that feel like "skipped beats" and occasional pounding in the chest. He feels fatigued and "drained" by the end of the day. The patient's sexual performance has also decreased recently. He does not have chest pains, shortness of breath, lightheadedness, or syncope. On examination, blood pressure is 152/89 mm Hg and pulse is 78/min. BMI is 33 kg/m 2 . Twenty-four-hour ECG (Holter) monitoring shows occasional premature ventricular contractions, episodes of bradycardia with rates around 30/min between 2-5 AM, 2 episodes of 7-second sinus pauses, and a short run of atrial fibrillation at a rate of 122-130/min at 4:30 AM. Which of the following is the best next step in management of this patient? A. Evaluation for alcohol abuse and dependence B. Exercise stress testing C. Full nighttime polysomnography D. Referral for pacemaker implantation E. Selective serotonin reuptake inhibitor

Answer C Educational objective: Bradycardia, asystole, and tachyarrhythmias during the nocturnal hours may occur in obstructive sleep apnea (OSA) and should be evaluated with overnight polysomnography. OSA-mediated bradycardia and asystole are not indications for permanent pacemaker implantation. Instead, continuous positive airway pressure (CPAP) would be indicated.

10. A 63-year-old man is diagnosed with advanced-stage small cell lung cancer, with a right-sided lung mass and malignant pleural effusion. There are no distant metastases. His performance status is excellent and he receives appropriate therapy. One month later, he complains of right-sided tinnitus and decreased hearing. Which of the following best explains this patient's new symptoms? A. Electrolyte abnormalities B. Immune-mediated injury C. Medication side effect D. Metastatic disease E. Middle ear infection

Answer C Educational objective: Common adverse effects associated with cisplatin include nephrotoxicity, tinnitus and hearing loss, electrolyte abnormalities, severe nausea and vomiting, and neurotoxicity.

24. A 34-year-old woman comes to the office with nasal obstruction, which seems to have worsened over the past year. She finds it difficult to breathe through her nose during exercise and occasionally at night. She also has intermittent nasal discharge. The patient has seasonal allergies that usually respond well to overthe-counter antihistamines. Physical examination is unremarkable. Rhinoscopy shows grey polypoid masses in the nasal cavity. Which of the following is the best initial treatment for this patient? A. Antihistamines B. Endoscopic surgery C. Intranasal glucocorticoids D. Lesion biopsy E. Leukotriene antagonists

Answer C Educational objective: Nasal polyposis, a common complication of chronic rhinosinusitis, should be managed initially with topical glucocorticoid sprays.

45. A 53-year-old postmenopausal woman comes to the office for a follow-up visit. The patient recently had surgery for early-stage, estrogen receptor-positive breast cancer diagnosed on screening mammography. She was started on an aromatase inhibitor postoperatively but discontinued the therapy due to intolerable side effects. Today she is starting tamoxifen as an adjuvant endocrine treatment. The patient has no other chronic medical problems. Her mother has type 2 diabetes mellitus and had a minor stroke. Her father died from metastatic colon cancer. Vital signs are within normal limits. BMI is 29 kg/m 2 . This patient is at highest risk for which of the following complications from tamoxifen therapy? A. Diabetes mellitus B. Myocardial infarction C. Osteoporosis D. Ovarian cancer E. Venous thrombosis

Answer E Educational objective: Tamoxifen, an estrogen antagonist in the breast, is used for adjuvant therapy in some patients with estrogen receptor-positive breast cancer. Tamoxifen is associated with an increased risk for venous thromboembolism.

40. A 62-year-old woman comes to the physician to discuss motion sickness. The patient says that her husband is taking her on a 7-day sea cruise to Alaska, and she is concerned because she frequently experiences nausea and vomiting when traveling by ship or airplane. Her past medical history is notable only for hypertension, which is well-controlled on chlorthalidone.Which of the following is the most appropriate medication for this patient? A. Cetirizine B. Metoclopramide C. Ondansetron D. Prochlorperazine E. Scopolamine

Answer E Educational objective: Antihistamines and anticholinergics are useful in the prevention and treatment of motion sickness. Scopolamine is preferred for prevention during extended travel as it is less sedating than other medication options.

47. A 2-month-old term African-American girl is brought to the physician for a routine well-child visit. She is exclusively breastfed every 3-4 hours for 15 minutes on each side. The patient has been growing well and developing normally. She takes no medications and has no allergies. Her mother eats a well-balanced diet and takes no medications.Which of the following nutritional deficiencies is this child at greatest risk of developing? A. Calcium B. Folate C. Protein D. Vitamin B12 E. Vitamin D

Answer E Educational objective: Breast milk is the best form of infant nutrition although it is relatively deficient in vitamin D. Exclusively breastfed infants should receive 400 international units of vitamin D per day. Vitamin D supplementation should also be given to infants who consume < 32 ounces (1 liter) of formula per day.

38. A 17-day-old girl is brought to the physician for a newborn visit. She was born vaginally to a 19-year-old primigravid mother following an uncomplicated pregnancy. Her birth weight was 3.6 kg (7 lb 10 oz). She is exclusively breastfeeding. The infant was seen in the office at age 5 days and her weight then was 3.4 kg (7 lb 8 oz); today it is 3.7 kg (8 lb 3 oz). She has been feeding well and making a normal number of wet and dirty diapers. Her vital signs are normal. Physical examination shows a well-nourished infant with scleral icterus and diffuse jaundice to the umbilicus. The liver edge is palpable at the right costal margin; no splenomegaly is appreciated. The remainder of the examination is normal. Laboratory results are as follows: Liver function studies,Total bilirubin 13.8 mg/dL,Direct bilirubin 0.7 mg/d,Aspartate aminotransferase (SGOT) 22 U/.L,Alanine aminotransferase (SGPT) 28 U/L, Which of the following is the most appropriate next step in this patient's management? A. Hepatic ultrasound B. Home phototherapy C. Inpatient phototherapy D. Lactation consult E. Observation only

Answer E Educational objective: Breast milk jaundice is a benign, indirect hyperbilirubinemia that results from β-glucuronidase present in breast milk. It presents around age 2 weeks in otherwise well-appearing, hydrated infants with no evidence of direct hyperbilirubinemia. Breastfeeding should be encouraged as the jaundice resolves spontaneously without harm by age 3 months.

53. A 24-year-old woman with no medical history comes to the office for an annual checkup. She recently broke up with her boyfriend and describes irregular menses and has concerns about gaining weight. Further questioning reveals that since high school the patient has had a pattern of consuming large amounts of carbohydrate-rich foods when feeling depressed, angry, or overwhelmed and then fasting and exercising 3-4 hours a day to prevent weight gain. The patient's vital signs and physical examination are normal. She is 167.5 cm (5' 6") tall and weighs 65 kg (143.3 lb). Laboratory evaluation is within normal limits.Which of the following is the most likely diagnosis? A. Anorexia nervosa B. Binge-eating disorder C. Body dysmorphic disorder D. Borderline personality disorder E. Bulimia nervosa

Answer E Educational objective: Bulimia nervosa is characterized by excessive preoccupation with weight and recurrent episodes of binge eating and compensatory behaviors. Patients are normal to overweight in contrast to the low weight seen in those with anorexia nervosa.

27. A 14-year-old boy is brought to the office for his annual physical examination. The patient just started 9th grade and is planning on playing football, wrestling, and running track as a sprinter this year. He is interested in taking creatine supplements to improve his athletic performance.Which of the following is the most accurate information that should be provided to this patient? A. Creatine can improve performance by increasing oxygen-carrying capacity B. Creatine can lower the heart rate and reduce performance anxiety C. Creatine is safe and recommended for competitive athletes D. Creatine should be avoided due to the risk for arrhythmias and hypertension E. The long-term effects of creatine are not known

Answer E Educational objective: Creatine is the most commonly used performance-enhancing agent by athletes, but there is no conclusive evidence that this supplement improves fitness. Adverse effects include weight gain and acute interstitial nephritis that may lead to chronic renal disease. The long-term effects, especially in growing adolescents and children, are also unknown

30. A 46-year-old woman is admitted to the hospital with diarrhea and generalized weakness. She has had several prior admissions for diarrhea, hypokalemia, and near-syncope. The patient has a history of hypertension, type 2 diabetes mellitus, and diverticulosis.Vital signs and physical examination are within normal limits.Laboratory evaluation shows mild hypokalemia. Colonoscopy shows mild diverticulosis and dark pigmentation of the colonic mucosa with a striated pattern. There are no masses or polyps.Which of the following is the most likely diagnosis? A. Addison's disease B. Autonomic neuropathy C. Celiac disease D. Hemochromatosis E. Laxative abuse

Answer E Educational objective: Factitious diarrhea due to laxative abuse usually presents in women with watery diarrhea, crampy abdominal pain, weight loss, dizziness due to volume depletion, and hypokalemia. Patients abusing anthraquinone-containing laxatives (eg, senna) may have melanosis coli on colonoscopy, which appears as dark pigmentation of the colonic mucosa with a striated pattern.

49. A 78-year-old woman comes to the physician following treatment for a peptic ulcer. She was seen for melanotic stools 8 weeks earlier. A 1-cm duodenal ulcer was found on endoscopy; biopsy specimens had a positive urease test but no evidence of malignancy. The patient was treated with a 10-day course of omeprazole, amoxicillin, and clarithromycin. She reports no further symptoms. Which of the following is the most appropriate next step in evaluation of this patient? A. Long-term proton pump inhibitor therapy B. Reassurance and periodic follow-up C. Repeat endoscopy D. Serologic test for Helicobacter pylori E. Urea breath test

Answer E Educational objective: Following treatment for Helicobacter pylori, confirmation of cure should be obtained in patients with peptic ulcer, persistent symptoms, H pylori-associated MALT lymphoma, or resection of gastric cancer. Testing options include urea breath test and stool antigen test.

36. A 60-year-old man comes to the office due to lower back pain over the last 4-5 months. The pain is worse at the end of the day and occasionally occurs at night. The patient becomes fatigued more easily than previously. He has no muscle weakness, numbness of the extremities, or urinary problems. The patient is a former smoker with a 25-pack-year history. Physical examination shows generalized pallor. Neurological examination is normal. Laboratory results are as follows: Complete blood count Hemoglobin 9.8 g/L Mean corpuscular volume 85 µm 3 Platelets 226,000/mm Leukocytes 6,500/mm Neutrophils 60% Eosinophils 1% Lymphocytes 29% Monocytes 10% Serum chemistry panel Sodium 138 mEq/L Potassium 4 mEq/L Bicarbonate 24 mEq/L Blood urea nitrogen 28 mg/dL Creatinine 2.1 mg/dL Calcium 11.2 mg/dL Glucose 98 mg/dL Liver studies Albumin 4 g/dL Total protein, serum 9.5 g/dL Total bilirubin 1 mg/dL Direct bilirubin 0.8 mg/dL Alkaline phosphatase 110 U/L Aspartate aminotransferase (SGOT) 58 U/L Alanine aminotransferase (SGPT) 25 U/L Which of the following is the best next step in management of this patient? A. Antinuclear antibody assay B. Bone marrow biopsy C. Bone scan D. K

Answer E Educational objective: Multiple myeloma typically presents with bone pain in the back or chest, hypercalcemia, renal failure, and anemia in older patients. Serum protein electrophoresis shows an abnormal monoclonal protein.

10. A 6-year-old girl is seen in clinic for follow-up after being admitted to the hospital for dactylitis. The patient's pain has improved and is currently well controlled. She was diagnosed with sickle cell disease at birth and has been hospitalized twice for pain crises. The patient has no other chronic medical conditions. Vital signs and physical examination are normal. Which of the following is the most appropriate strategy for management of stroke risk in this patient? A. Antiplatelet therapy B. Chronic transfusion therapy C. Direct oral anticoagulant therapy D. Echocardiogram E. Transcranial Doppler ultrasound

Answer E Educational objective: Patients with sickle cell disease are at high risk for stroke due to sickling in the cerebral vasculature. Transcranial Doppler (TCD) screening should be obtained annually from age 2-16, and patients with elevated TCD velocities benefit from prophylactic, chronic erythrocyte transfusions.

55. A 42-year-old man complains of fatigue over the last 6 months. He has no abdominal pain, vomiting, fever, or weight loss. His past medical history is significant for ulcerative colitis diagnosed 3 years ago. The disease is controlled with oral 5-aminosalicylic acid and steroid enemas. His temperature is 37.0 C (98.6 F), blood pressure is 134/90 mm Hg, pulse is 74/min, and respirations are 15/min. Examination shows mild tenderness in the left lower quadrant. There is no hepatomegaly or splenomegaly.Laboratory results are as follows:Albumin 3.8 g/dL,Total bilirubin 1.7 mg/dL,Alkaline phosphatase 816 U/L,Aspartate aminotransferase (SGOT) 45 U/L,Alanine aminotransferase (SGPT) 54 U/L,Erythrocyte sedimentation rate is 102 mm/hr and antineutrophil cytoplasmic antibody screen is positive with a perinuclear pattern (perinuclear anti-neutrophil cytoplasmic antibodies).Which of the following is the most likely diagnosis? A. Bacterial cholangitis B. Drug-induced hepatotoxicity C. Metastatic colon cancer D. Primary biliary cirrhosis E. Primary sclerosing cholangitis

Answer E Educational objective: Primary sclerosing cholangitis is a chronic progressive disorder characterized by inflammation, fibrosis, and stricturing of medium-sized and large intrahepatic and extrahepatic bile ducts. It is frequently associated with ulcerative colitis. Laboratory studies usually show elevated liver function tests in a cholestatic pattern.

21. An 8-week-old full-term boy is brought to the physician for evaluation of bowel movements. The infant is exclusively breast fed for 15 minutes on each side every 3 hours. He previously had 5 or 6 liquid, yellow, seedy stools a day during the first month of life. Over the past few weeks, he has been stooling once a week and the stools have the same consistency and color. The boy's last stool was 3 days ago. Weight and length are at the 40th percentile. Examination shows no abnormalities.Which of the following is the most likely cause of this patient's decreased stooling? A. Constipation B. Gastrointestinal tract obstruction C. Hypothyroidism D. Inadequate fiber intake E. Normal infant stooling

Answer E Educational objective: Stooling patterns change throughout infancy and childhood. Breastfed infants may stool as frequently as every feeding or as infrequently as once a week. The diagnosis of constipation is based not only on frequency but on pain with defecation and hard stools.

9. A 57-year-old man with known prostate cancer comes to the physician due to progressive, severe lower back pain. The patient has been taking naproxen and acetaminophen every day with little relief. He has no lower extremity weakness, perianal numbness, or urinary incontinence and has not fallen. The patient is currently being treated with leuprolide and zoledronic acid. Imaging studies show multiple osteoblastic lesions in his thoracic and lumbar spine without evidence of compression fractures. Which of the following is the best next step in managing this patient? A. Discontinue leuprolide B. Docetaxel and prednisone C. Fentanyl patch D.Methadone E. Oral morphine

Answer E Educational objective: The World Health Organization analgesic ladder recommends starting treatment with nonopioids for mild cancer-related pain. Weak opioids should be added if pain control is inadequate or pain is of moderate severity. Severe pain, or pain unresponsive to weak opioids, should initially be treated with short-acting strong opioids; after the total daily dose is established, a long-acting opioid should be started, with shortacting opioids reserved for breakthrough pain.

46. A 40-year-old man comes to the emergency department with black, tarry stools for the past 2 days. He is otherwise in good general health and takes no prescription medications. His supine blood pressure is 120/80 mm Hg and pulse is 84/min; after 1 minute of standing they are 118/76 mm Hg and 90/min. Physical examination is remarkable for pallor. Upper gastrointestinal endoscopy shows a nonbleeding duodenal ulcer with visible vessel, and endoscopic therapy is performed.Which of the following values for hemoglobin represents the optimal threshold for initiating blood transfusion in this patient? A. <11 g/dL B. <10 g/dL C. <9 g/dL D. <8 g/dL E. <7 g/dL

Answer E Educational objective: The threshold for blood transfusion in most stable patients with upper gastrointestinal bleeding is hemoglobin <7 g/dL. Patients at risk for morbidity in the setting of severe anemia (eg, unstable coronary artery disease) may benefit from maintaining hemoglobin >9 g/dL.

7. A 52-year-old man comes to the office for a preventive visit. He has no history of coronary artery disease or stroke but does have a 40-pack-year smoking history. The patient has no family history of early coronary disease. Vital signs are normal. BMI is 29 kg/m 2 . Laboratory results are as follows: Total cholesterol 262 mg/dL , HDL 50 mg/dL , LDL 162 mg/dL , Triglycerides 150 mg/dL Fasting glucose 100 mg/dL . The patient's calculated 10-year risk of cardiovascular disease is 10.6%. He is counseled about the health risks of smoking and the importance of smoking cessation. Which of the following is the most appropriate next step in management of this patient's dyslipidemia? A. Fish oil supplementation B. Lifestyle modification only C. Low-dose aspirin D. Oral ezetimibe E. Oral niacin therapy F. Oral rosuvastatin

Answer F Educational objective: Lipid-lowering therapy with statins is recommended for primary prevention in patients age 40-75 with a 10year risk of atherosclerotic cardiovascular disease (ASCVD) ≥10%. Statins are also recommended for patients with clinical ASCVD, those age 40-75 with diabetes and LDL ≥70 mg/dL, and those with LDL levels ≥190 mg/dL, regardless of ASCVD risk score.

34. A 42-year-old woman comes to the physician with bilateral knee pain that severely limits her mobility. She also complains of widespread morning joint stiffness that takes several hours to improve. These symptoms have slowly worsened over several months. She has been taking over-the-counter ibuprofen and aspirin but has experienced little relief of symptoms. The patient has no other medical problems and does not use tobacco, alcohol, or illicit drugs. Her vital signs are within normal limits. Physical examination shows tenderness and swelling of several metacarpophalangeal joints, as well as both wrists and knees. Laboratory results are as follows: Hemoglobin 9.4 g/dL Serum iron 30 μg/dL (normal 50-150 μg/dL) Total iron-binding capacity 230 μg/dL (normal 250460 μg/dL) Ferritin 300 ng/mL (normal 15200 ng/mL) Which of the following is the most appropriate next step to improve this patient's anemia? A. Combination folic acid and vitamin B12 supplementation B. Disease-modifying antirheumatic drug therapy C. Erythropoietin supplementation D. Iron supplementation E. Periodic blood transfusions (as needed)

answer B Educational objective: Anemia of chronic disease is a disorder of iron utilization that most commonly occurs in the setting of chronic inflammation. It is characterized by a normocytic anemia with decreased serum iron, decreased total iron- binding capacity, decreased iron saturation, and normal or elevated serum ferritin. Treating the underlying inflammatory disorder will often improve the anemia.

32. A 52-year-old postmenopausal woman with no significant past medical history is diagnosed with HER2positive breast cancer. The primary tumor is a 1.7-cm ductal carcinoma with a single positive axillary lymph node identified on dissection. Estrogen and progestin receptor status is negative. The patient's chest x-ray, complete blood count, and renal and liver function tests are normal. Which of the following should be performed on this patient before proceeding with trastuzumab therapy? A. Bone density testing B. Bone marrow biopsy C. Echocardiography D. Skin testing for tuberculosis E. Ultrasound of the abdomen

answer C Educational objective: Trastuzumab is commonly used to treat HER2-positive breast cancer. It is beneficial as adjuvant therapy in early-stage cancer as well as in metastatic disease. However, it has the potential for significant cardiotoxicity. An echocardiogram is recommended before beginning therapy.

31. A 38-year-old HIV-negative man comes to the office for evaluation of a neck mass that he first discovered 2 weeks ago. The patient has no other associated symptoms or significant past medical history. Physical examination shows enlarged, firm, nontender, and mobile left-sided anterior cervical lymph nodes. The rest of the examination is unremarkable. Complete blood count, comprehensive metabolic panel, and chest x-ray are within normal limits. Cervical lymph node biopsy is consistent with classical Hodgkin lymphoma. Which of the following is the best next step in managing this patient? A. Bone marrow biopsy B. Laparotomy with splenectomy C. Local radiation therapy D. Positron emission tomography (PET)/CT scan E. Serum protein electrophoresis

answer D Educational objective: Classical Hodgkin lymphoma typically presents with enlarged lymph nodes or a mediastinal mass on chest x-ray. Tissue biopsy confirms the diagnosis. Positron emission tomography (PET)/CT scan of the chest, abdomen, and pelvis is advised for staging and determining treatment options.

6. An analysis of results from 18,317 patients pooled from 5 large randomized controlled trials on drug X revealed the following relative rates (RR) for drug X compared to the control: RR of cardiovascular mortality = 0.90 (95% confidence interval [CI], 0.73-1.38) RR of heart failure = 2.09 (95% CI, 1.52-2.88) RR of myocardial infarction = 1.18 (95% CI, 0.86-1.31) Based on the results of this meta-analysis, there is a statistically significant increase in which of the following events among patients taking drug X? A. Cardiovascular mortality and myocardial infarction B. Cardiovascular mortality only C. Heart failure and myocardial infarction D. Heart failure only E. Myocardial infarction only

Answer D Educational objective: The relative risk is the risk of an event in the exposed group compared to the risk of that event in the unexposed group. A confidence interval that crosses the null value is not statistically significant.

60. A 32-year-old woman comes to the physician with bloating, abdominal pain, and diarrhea. The exclusion of dairy products from her diet results in significant symptomatic relief. The patient's physical examination and basic laboratory test results are unremarkable. Antitissue transglutaminase antibody is negative.Which of the following is the most appropriate management of this patient? A. Avoidance of nonsteroidal anti-inflammatory drugs B. Complete exclusion of dairy products C. Daily iron supplementation D. Daily probiotic intake E. Maintenance of calcium and vitamin D intake

Answer E Educational objective: Maintenance of calcium and vitamin D intake is essential in patients with lactose malabsorption. Complete avoidance of dairy products is not usually recommended as most patients can tolerate up to 12 g of lactose daily (equal to 1 glass of milk). Live culture yogurt, lactase enzyme replacement, and the combination of dairy with other foods may be helpful.

63. Which of the following is the preferred antibiotic for treatment of Clostridium difficile colitis and can be used in either an oral or rectal preparation? A. Fidaxomicin B. Metronidazole C. Nitazoxanide D. Rifaximin E. Vancomycin

Answer E Educational objective: Primary Clostridium difficile infection is treated with oral vancomycin or fidaxomicin. Vancomycin is also administered in a rectal preparation for patients with significant ileus to enhance antibiotic delivery to the colon.

7. A 36-year-old woman comes to the emergency department during winter with a 1-day history of headache. She lives in a rural area in a small house built nearly 100 years ago. She has a severe, bilateral throbbing headache accompanied by nausea and generalized weakness. Her husband has mild but otherwise similar symptoms. She is afebrile and physical examination is unremarkable. Which of the following is the best next step in managing this patient? A. Carboxyhemoglobin level B. Lumbar puncture C. Nasal influenza swab D. Serum acetaminophen level E. Urine toxicology screen

ANSWER A Educational objective: Carbon monoxide toxicity is characterized by headache, altered mentation, and nonspecific symptoms (eg, dizziness, nausea). Diagnosis is confirmed by elevated serum carboxyhemoglobin. Treatment includes administration of 100% oxygen via non-rebreathing face mask.

3. A 16-year-old boy is brought to the emergency department with abdominal and groin pain that began suddenly 10 hours ago. The pain started in the lower abdomen and then radiated to the right groin. He has vomited 4 times. The boy is sexually active with 2 female partners and does not use condoms. His temperature is 37.2 C (99 F), blood pressure is 104/62 mm Hg, and pulse is 108/min. He has mild lower abdominal tenderness with palpation. The right hemiscrotum is swollen, erythematous, and tender to palpation. The right testicle is firm with a horizontal lie with no cremasteric reflex. Which of the following is the most important next step in management of this patient? A. Emergency surgery B. Empiric antibiotics for gonorrhea and chlamydia C. Scrotal support and analgesia D. Transillumination of the scrotum E. Ultrasound of the scrotum with Doppler

ANSWER A Educational objective: Testicular torsion is a urologic emergency that presents with acute, severe scrotal pain, horizontal testicular lie, and an absent cremasteric reflex. Ultrasound is not required for the diagnosis of classic cases. Definitive management consists of surgical detorsion and fixation with exploration of the contralateral hemiscrotum and should be done within 6 hours to maximize viability.

14. A 40-year-old woman comes to the office due to increasing pelvic pain and urinary urgency for the last few months. The patient initially had increasing urgency and was voiding every 1-2 hours. She now has increasing pelvic pain and pressure that requires her to void every 30-45 minutes. The patient's pelvic pain is typically relieved with voiding but returns prior to her next void. She has had no hematuria, postcoital bleeding, or urinary leakage. The patient has fibromyalgia and type 2 diabetes mellitus. She had 2 vaginal deliveries in her late 20s. She is sexually active with a male partner and has had a tubal ligation. The patient does not use tobacco, alcohol, or illicit drugs. Vital signs are normal. BMI is 28 kg/m 2 . On pelvic examination, the external genitalia and vagina appear normal and well-supported. The cervix is multiparous and nonfriable, and physiologic discharge is noted throughout the vaginal vault. On bimanual examination, there is tenderness to palpation over the urethra but no cervical motion tenderness. Laboratory results are as follows: Urinalysis Glucose negative Leukocyte esterase negative Nitrites negative White blood cells 1-2/hpf Urine culture a

ANSWER B Educational objective: Interstitial cystitis causes chronic pelvic pain that increases with bladder filling and is relieved with voiding. Urinalysis is typically normal. Treatment includes patient education, avoidance of triggers, and oral pharmacotherapy (eg, amitriptyline, pentosan polysulfate sodium).

5. A 65-year-old man comes to the urgent care clinic complaining of frequent urination and perineal pain for the last 2 days. He also mentions malaise and chills. He has never had similar symptoms before. He is in a monogamous relationship with his wife. The patient's temperature is 37.8 C (100 F), blood pressure is 120/70 mm Hg, pulse is 94/min, and respirations are 18/min. Rectal examination shows a boggy and tender prostate. Laboratory results are as follows: Hemoglobin 13.1 g/dL White blood cells 12,000/µL with 7% band forms Platelets 329,000/µL The patient's creatinine level is 0.9 mg/dL. Dipstick urinalysis is positive for leukocyte esterase. Which of the following is the most appropriate next step in management of this patient? A.. Alpha-adrenergic blocking agents B. CT scan of the pelvis C. Culture of mid-stream urine sample D. Prostate specific antigen measurement E. Urethral catheterization

ANSWER C Educational objective: Acute bacterial prostatitis presents in a manner similar to other urinary tract infections but with the addition of perineal pain, pronounced systemic symptoms (fever, chills, acute illness), and a tender, boggy prostate on examination. Urine culture should be obtained to help direct antibiotic therapy.

A 56-year-old man is brought to the emergency department by his wife due to decreased appetite andlethargy over the last 2 days. He has a history of liver cirrhosis secondary to chronic alcoholism. Hismedications include propranolol, spironolactone, and furosemide. The patient underwent variceal bandligation 6 months ago for primary prophylaxis of bleeding. His temperature is 37.5 C (99.5 F), bloodpressure is 100/55 mm Hg, pulse is 104/min, and respirations are 24/min. On physical examination, thepatient appears lethargic but answers questions correctly. His attention span is short. Abdominalexamination shows distension with shifting dullness and mild diffuse tenderness to palpation. A jerkingtremor of the hands is noted when his arms are held out. There is bilateral pitting edema of his lowerextremities. Laboratory results are as follows: White blood cells12,000/µLTotal bilirubin2.5mg/dLAspartateaminotransferase110U/LAlanineaminotransferase52 U/LAlkaline phosphatase125U/LCreatinine1.4mg/dLAlbumin2.1mg/dL Chest x-ray shows no abnormalities. Abdominal x-ray shows gas in the small and large bowel without airfluid levels.Which of the following is the most appropriate next step in management

ANSWER C Educational objective: Spontaneous bacterial peritonitis can have a subtle presentation and should be considered in a patient with cirrhosis and ascites accompanied by fever or a change in mental status. Diagnosis is confirmed with paracentesis, with a positive ascites fluid culture and neutrophil count >250/µL as the main criteria.

1. A 29-year-old woman comes to the office due to prolonged gum bleeding at a recent dentist visit. During a routine cleaning, the patient repeatedly had to have blood irrigated from her mouth. She has also had easy bruising but no fevers, weight changes, gastrointestinal bleeding, or hematuria. The patient has a history of chronic temporomandibular joint pain, which is moderately controlled with over-the-counter acetaminophen. Vital signs are normal. Examination reveals normal gingiva and oropharyngeal mucosa. Complete blood count, PT, PTT, and peripheral blood smear are normal. Which of the following is the most likely cause of this patient's abnormal bleeding? A. Cutaneous small-vessel vasculitis B. Hemophilia A C. Lupus anticoagulant D. Vitamin K deficiency E. von Willebrand disease

ANSWER E Educational objective: Von Willebrand disease is the most common inherited bleeding disorder; it is transmitted in an autosomal dominant fashion, so a family history of bleeding is often present. Most cases are mild or asymptomatic, but a minority of patients can develop clinically significant skin or mucosal bleeding. Laboratory evaluation usually reveals normal platelet count and coagulation studies; diagnosis is usually made with VWF antigen, VWF activity, and factor VIII activity levels.

19. A 27-year-old woman comes to the office for an annual examination. The patient has had no postcoital or intermenstrual bleeding, pelvic pain, or abnormal vaginal discharge. She uses oral contraceptive pills, is currently sexually active with 1 partner, and has had 4 lifetime partners. Pelvic examination is normal. Pap test results reveal a high-grade squamous intraepithelial lesion. Which of the following is the best next step in management of this patient? A. Colposcopy B. Cryotherapy C. HPV co-testing D. Repeat Pap test in 1 year E. Trichloroacetic acid

Answer A Educational objective: A high-grade squamous intraepithelial lesion Pap test result requires colposcopic examination and biopsy of cervical abnormalities due to a high risk of progression to cervical cancer.

1. A 17-year-old girl comes to the office for a health maintenance examination. The patient became sexually active at age 14. She has been monogamous with a male partner for the last year and has had no postcoital bleeding or abnormal vaginal discharge. She uses oral contraceptives and condoms consistently. All recommended vaccinations are current, and HIV testing at her last visit a year ago was negative. Physical examination is normal. The patient inquires about screening for sexually transmitted infections (STIs). Which of the following additional tests is most appropriate? A. Chlamydia trachomatis nucleic acid amplification testing B. No additional STI screening is indicated C. Nontreponemal serology for syphilis D. Pap test and human papillomavirus cotesting E. Trichomonas vaginalis nucleic acid amplification testing

Answer A Educational objective: Annual chlamydia and gonorrhea screenings are recommended for all sexually active women age <25. In addition, HIV screening is recommended at least once by age 16-18, regardless of sexual activity.

1. A 24-year-old, nulliparous woman comes to the office for an initial prenatal visit. The patient wears eyeglasses for myopia. Her father died suddenly several years ago. Examination shows long fingers and extremities, pectus carinatum, mild scoliosis, and a 2/6 early diastolic murmur at the left sternal border. Ultrasound shows an 8-week intrauterine pregnancy. Which of the following is the most likely cause of mortality in this patient? A. Aortic disease B. Infective endocarditis C. Mitral valve prolapse D. Pulmonary hypertension E. Systolic heart failure

Answer A Educational objective: Aortic disease, with progressive dilation and significant risks of aortic dissection/rupture, is the leading cause of morbidity and mortality in patients with Marfan syndrome. This risk is significantly higher during pregnancy due to hypervolemia and increased stress on the aortic wall.

2. A 42-year-old man is being evaluated in the emergency department following a high-speed motor vehicle accident. The patient experiences a sudden onset of chest pain. He has no shortness of breath, abdominal pain, or neck pain. His blood pressure is 122/70 mm Hg, pulse is 90/min, and oxygen saturation is 98% on room air. Physical examination shows multiple bruises on the anterior aspect of the chest. The abdomen is soft and nontender. Bedside ultrasound examination reveals no pericardial or abdominal fluid. Chest x-ray shows a widened mediastinum and several adjacent rib fractures without lung opacities or pneumothorax. Which of the following should be evaluated for in this patient? A. Aortic injury B. Cardiac contusion C. Esophageal rupture D. Flail chest E. Pulmonary contusion

Answer A Educational objective: Aortic injury is the most serious complication after blunt chest trauma and is associated with high mortality. Chest x-ray can show a wide mediastinum and usually requires further imaging (chest CT or transesophageal echocardiogram) to confirm the diagnosis.

23. A 17-year-old boy is brought to the emergency department with acute right ear pain and swelling. He was competing in a mixed martial arts tournament and was kicked in the side of the head 3 hours ago. Examination shows a 3-cm area of erythema and tenderness with fluctuant swelling over the fossa of the right ear. The tympanic membrane is clear and intact. Which of the following is the most appropriate next step in management of this patient? A. Incision and drainage B. Levofloxacin otic drops C. Pressure dressing D. Reassurance and observation E. Temporal bone CT scan without contrast

Answer A Educational objective: Auricular hematomas are caused by blunt trauma to the external ear. Classic findings include an erythematous, tender swelling in the auricular fossa. Incision and drainage should be performed immediately to prevent "cauliflower ear". A pressure dressing is subsequently applied to prevent reaccumulation of blood and serous fluid.

9. A 62-year-old business executive comes to the office due to episodes of lightheadedness over the last 6 months. These happen primarily in the morning when he is getting dressed. During these episodes, the patient feels that the room "is getting dark" and that he is going to pass out. One time, he briefly lost consciousness. He has no nausea, chest pain, palpitations, headaches, vision problems, shortness of breath, tongue bites, or urinary incontinence. His medications include valsartan for high blood pressure. The patient is a former smoker with a 25-pack-year history. His blood pressure is 134/85 mm Hg while supine and 140/82 mm Hg after standing for 2 minutes, and pulse is 72/min. There are no heart murmurs or carotid bruits. Lungs are clear on auscultation. Peripheral pulses are full and symmetric in all 4 extremities. ECG shows normal sinus rhythm. Which of the following is the best next step in the evaluation of this patient? A. Carotid sinus massage B. Discontinuation of valsartan and clinical follow-up C. Dix-Hallpike maneuver D. Electroencephalogram E. Treadmill exercise test

Answer A Educational objective: Carotid sinus massage testing is recommended in patients age >40 with syncope of uncertain etiology to diagnose carotid sinus hypersensitivity. Carotid sinus massage is considered diagnostic if the patient develops syncope along with asystole for >3 seconds and/or there is a fall in systolic blood pressure of >50 mm Hg. It should be avoided in patients with ipsilateral carotid artery stenosis or carotid bruits and/or history of transient ischemic attack or stroke within the past 3 months. Treatment requires permanent pacing.

13. A 5-year-old boy is brought to the office for a routine well-child visit. He has been growing and developing well and recently reached the height limit of his forward-facing car seat. Physical examination is normal. Which of the following is the most appropriate recommendation regarding injury prevention for this patient? A. The child should sit in the back row of the vehicle with a belt-positioning booster seat B. The child should sit in the back row of the vehicle with a forward-facing car seat until he reaches the seat's weight limit C. The child should sit in the front or back row of the vehicle with a belt-positioning booster seat D. The child should sit in the front or back row of the vehicle with lap and shoulder belts E. The child should sit in the front row of the vehicle with a belt-positioning booster sea

Answer A Educational objective: Children should transition from a forward-facing car safety seat to a belt-positioning booster seat when they reach the car seat's height or weight limit. All children age <13 should ride in a back passenger row.

6. A 14-year-old boy is brought to the physician for follow-up of blood pressure measurements >95th percentile at the last 3 office visits. He has no underlying health problems and does not use illicit drugs, alcohol, or tobacco. The patient takes no medications. His father has hypertension and type 2 diabetes mellitus. There is no family history of heart attack or sudden unexplained death. Weight is at the 98th percentile and height is at the 65th percentile. His supine blood pressure is 135/82 mm Hg, pulse is 72/min, and respirations are 16/min. The remainder of his physical examination is normal. Urinalysis is normal. Laboratory results are as follows: Complete blood count Hemoglobin 14 g/dL Platelets 230,000/ µL Leukocytes8,000/µL Serum chemistry Sodium 140 mEq/L Potassium 4 mEq/L Chloride 100 mEq/L Bicarbonate 22 mEq/L Blood urea nitrogen 15 mg/dL Creatinine 0.5 mg/dL Fasting glucose 85 mg/dL Fasting lipid panel Cholesterol 175 mg/dL HDL 50 mg/dL LDL 100 mg/dL. VLDL 24 mg/dL Which of the following should be included in the evaluation of this patient? A. Echocardiogram B. Plasma aldosterone and renin C. Plasma and urine catecholamines D. Renal ultrasound E. Urine toxicology

Answer A Educational objective: Children with primary (essential) hypertension are often obese and have an otherwise normal physical examination. Untreated hypertension in childhood and adolescence can lead to premature atherosclerosis and cardiovascular disease. Echocardiogram should be included in the initial evaluation.

15. A 60-year-old man comes to the physician due to nausea and abdominal pain 5 days after a coronary stenting procedure. His other medical problems include carotid stenosis and type 2 diabetes mellitus. On physical examination, there is painless, patchy purple mottling of the skin of the lower extremities as shown below. Peripheral pulses are palpable and normal. Laboratory studies show increased creatinine to 2.9 mg/dL (0.9 mg/dL 5 days ago) and eosinophilia. Which of the following is the most likely diagnosis? A. Cholesterol embolism B. Contrast nephropathy C. Delayed iodine allergy D. Diabetic nephropathy E. Femoral pseudoaneurysm

Answer A Educational objective: Cholesterol embolization should be suspected in patients with acute or subacute renal failure, typical skin manifestations (eg, livedo reticularis, blue toe syndrome), and/or cerebral or intestinal ischemia, after recent arteriography or cardiac catheterization.

23. A 70-year-old woman comes to the physician with fatigue over the last 3-4 months. She feels very tired at the end of the day. She has no fevers, back pain, or weight loss. The patient had a normal colonoscopy 4 years ago. Physical examination shows pallor and cervical lymphadenopathy. Laboratory results are as follows: Hemoglobin 12.0 g/L Mean corpuscular volume 85 fl Platelets 224,000/ µL Leukocyte 54,500/ µL Neutrophils 16% Lymphocytes 75% Monocytes 9% Some variants of lymphocytes and smudge cells are present. Which of the following is the most likely diagnosis? A. Chronic lymphocytic leukemia B. Chronic myeloid leukemia C. Hairy cell leukemia D. Hodgkin's disease E. Lymphoblastic leukemia

Answer A Educational objective: Chronic lymphocytic leukemia is most common in elderly patients. Peripheral blood smear is characterized by predominant lymphocytosis and smudge cells.

39. A 60-year-old man comes to the office for presurgical evaluation before right total knee replacement. He has no issues other than knee pain. The patient has a history of coronary artery disease and underwent drug-eluting stent implantation into the left anterior descending artery a year ago. Current medications are low-dose aspirin, prasugrel, metoprolol, and rosuvastatin. Echocardiogram shows normal left ventricular systolic function and no wall motion abnormalities. Which of the following is the best preoperative recommendation for this patient? A. Continue aspirin, discontinue prasugrel B. Continue both antiplatelet medications C.Continue prasugrel, discontinue aspirin D. Discontinue both antiplatelet medications E. Postpone surgery

Answer A Educational objective: Dual antiplatelet therapy (DAPT) is recommended for a minimum of 6-12 months following placement of either a bare-metal or drug-eluting stent. Elective surgeries should be delayed until the minimum duration of DAPT is completed. At that point, P2Y12 receptor blockers (eg, clopidogrel, prasugrel) should be stopped prior to surgery and aspirin should be continued perioperatively unless there is a high risk of severe bleeding complications (eg, neurosurgery).

43. A 62-year-old postmenopausal woman comes to the office due to 2 weeks of vaginal bleeding. The patient had 5 vaginal deliveries and used combination oral contraceptives between pregnancies until undergoing a tubal ligation with her last delivery. She has smoked a pack of cigarettes daily for 40 years. She has had 6 lifetime sexual partners. BMI is 38 kg/m 2 . Pelvic examination shows dark red blood from the cervix. Endometrial biopsy reveals endometrial carcinoma. Which of the following is the greatest risk factor for this patient's condition? A. Body mass index B. Grand multiparity C. Number of sexual partners D. Oral contraceptive use E. Smoking history

Answer A Educational objective: Endometrial cancer is caused by endometrial proliferation from unopposed estrogen without the protective effects of progesterone. Obesity increases the risk of endometrial cancer due to conversion of androgens to estrogens in the peripheral adipose tissue, resulting in increased estrogen levels.

8. A 55-year-old asymptomatic Black man with no significant medical history comes to the office for hypertension follow-up. Blood pressure today is 145/90 mm Hg, and it has been 140-145/85-90 mm Hg at home. BMI is 29 kg/m 2 . Cardiovascular examination is normal. Laboratory studies, including basic metabolic panel and urinalysis, are normal. ECG shows normal sinus rhythm and left ventricular hypertrophy. Which of the following is the best initial therapy for this patient? A. Amlodipine B. Losartan C. Metoprolol D. Quinapril E. Spironolactone

Answer A Educational objective: First-line antihypertensive options for most patients include thiazide diuretics, ACE inhibitors, angiotensin receptor blockers, or long-acting calcium channel blockers (CCBs). For Black patients, thiazides and CCBs are preferred.

18. A 50-year-old woman, gravida 3 para 3, comes to the office due to worsening urinary frequency. Over the last few months, the patient has increased her voiding frequency to 15-20 times per day. In addition, she has pain with bladder filling, although voiding often relieves the pain. The patient has had no fever or loss of continence. She has well-controlled type 2 diabetes mellitus. Vital signs are normal. The bladder base and urethra are tender to palpation. Urinalysis shows 1-2 WBC/hpf and 1-2 RBC/hpf; nitrite and leukocyte esterase are negative. Which of the following is the most likely diagnosis in this patient? A. Interstitial cystitis B. Nongonococcal urethritis C. Urgency incontinence D. Urinary tract infection E. Vesicovaginal fistula

Answer A Educational objective: Interstitial cystitis (bladder pain syndrome) is a chronic (≥6 weeks) condition that presents with pain that worsens with bladder filling and is relieved with voiding (resulting in frequent urination). Urinalysis and urine culture are normal. Treatment is not curative but includes patient education, behavioral modification, and pharmacotherapy (eg, amitriptyline, pentosan polysulfate).

26. A 30-year-old woman, gravida 1 para 1, comes to the office due to fever, chills, and left breast pain that began yesterday. The patient is 7 weeks postpartum and for the first 6 weeks breastfed her infant exclusively. She returned to work a week ago; the patient breastfeeds only while at home and pumps once during the day. Temperature is 38.9 C (102 F), blood pressure is 120/70 mm Hg, and pulse is 114/min. There is a 5-cm, erythematous, tender area in the upper outer quadrant of the left breast; there is no fluctuance. Which of the following is the best next step in management of this patient? A. Continue breastfeeding and prescribe dicloxacillin B. Order breast ultrasound and perform aspiration of the erythematous area C. Perform incision and drainage of the erythematous area D. Recommend cold compresses and prescribe oxycodone E. Switch to formula feeding and prescribe trimethoprim-sulfamethoxazole

Answer A Educational objective: Lactational mastitis is a common postpartum infection in which clogged milk ducts become infected, resulting in fever, breast pain, and localized erythema/warmth/tenderness. Diagnosis is made clinically and treatment consists of frequent breastfeeding and antibiotics

6. A 49-year-old woman, gravida 1 para 1, comes to the office due to leakage of urine. The patient first noticed intermittent loss of urine a few years ago. The leakage has progressively worsened and now she is constantly leaking a small amount of urine. The patient soaks through a thin incontinence pad 2 or 3 times a day. She has no dysuria, hematuria, or urge to void. The patient has type 1 diabetes mellitus managed with an insulin pump. Her last hemoglobin A1c was 10.4%. At age 30 she had an operative vaginal delivery complicated by a large perineal laceration. She does not use tobacco, alcohol, or illicit drugs. Vital signs are normal. Pelvic examination reveals leakage of urine that increases with Valsalva. Postvoid residual volume is increased. There is decreased sensation to light touch and pinprick involving the perineum and both feet. Laboratory results are as follows: Urinalysis Protein. trace Blood. trace Urine culture. negative Which of the following is the best next step in management of this patient? A. Intermittent self-catheterization B. Pelvic floor muscle exercises C. Placement of midurethral sling D. Placement of suprapubic catheter E. Tolterodine therapy

Answer A Educational objective: Overflow incontinence presents with continuous leakage of urine secondary to urinary retention caused by underactive detrusor function (eg, diabetes mellitus) or outflow obstruction (eg, pelvic organ prolapse). Treatment focuses on correcting reversible causes, cholinergic agonists, and intermittent selfcatheterization.

8. A 60-year-old man with a 7-year history of gastroesophageal reflux disease undergoes screening endoscopy for Barrett esophagus. He has a 15-pack-year smoking history. Upper gastrointestinal endoscopy shows moderate esophagitis with red and velvety mucosa extending from the gastroesophageal junction into the distal esophagus. There is no evidence of gastritis. Biopsy results are confirmed by the pathologist as Barrett esophagus with high-grade dysplasia.Which of the following is the most appropriate next step in management of this patient? A. Endoscopic eradication therapy B. Increased proton pump inhibitor use to twice a day with repeat endoscopy in 6-12 months C. Intensive endoscopic surveillance in 3-6 months D. Routine endoscopic surveillance in 3-5 years E. Testing and treatment of Helicobacter pylori infection

Answer A Educational objective: Patients with Barrett esophagus and high-grade dysplasia require endoscopic eradication therapy (preferred) or esophagectomy.

39. A 68-year-old woman comes to the office and inquires about screening mammography. The patient has had normal biennial mammographies in the past and reports no breast symptoms. Medical conditions include hypertension, type 2 diabetes mellitus treated with insulin, hypercholesterolemia, end-stage renal disease on hemodialysis, ischemic stroke with residual deficits, and congestive heart failure with an ejection fraction of 15%. The patient has been hospitalized 3 times in the past year due to decompensated heart failure. She has a 50-pack-year smoking history. The patient has no personal or family history of breast or ovarian malignancies. She ambulates using a walker. Which of the following is the most appropriate recommendation regarding breast cancer screening for this patient? A. Advise no additional routine screening B. Continue biennial screening mammography C. Obtain MRI of the breasts D. Order ultrasonography of the breasts E. Recommend periodic self- and clinical breast examinations

Answer A Educational objective: Patients with limited life expectancy are unlikely to live long enough to benefit from the detection of malignancies and are more likely to experience harm (both physical and psychological) from the diagnosis. Patients with an expected lifespan of <10 years should not be screened for malignancy (eg, breast, colon).

13. A 34-year-old woman comes to the physician with palpitations that are "very bothersome". They have occurred almost every day over the last year without any clear precipitating factor and feel like "pounding in the chest" for several minutes. She has not had any syncope or presyncope. Family history is unremarkable. The patient appears mildly anxious. Physical examination and 12-lead ECG are normal. Laboratory testing shows hemoglobin of 14 g/dL and TSH level of 2.2 µU/mL. Which of the following is the best next step in management of this patient? A. Ambulatory ECG monitoring B. Selective serotonin reuptake inhibitor therapy C. Symptomatic therapy with a beta-blocker D. Tilt-table testing E. Treadmill exercise ECG

Answer A Educational objective: Patients with unexplained daily palpitations should be evaluated with ambulatory ECG monitoring (Holter monitoring). Patients with less frequent symptoms can be evaluated with a continuous loop event recorder.

27. A 48-year-old man comes to the office due to a painless left neck mass. He first noticed the mass 6 weeks ago and it has gradually enlarged since. He has no skin changes, fever, cough, hoarseness, or dysphagia. The patient has a history of tonsillectomy in childhood and frequent office visits for sore throat. He has smoked 1 pack of cigarettes daily for 18 years. His mother has a history of thyroid cancer. On examination, the patient has a 2-cm firm, non-pulsatile, nontender mass in the left mid-jugular area. Ear, nose, throat, and thyroid examinations are unremarkable, with the exception of extensive dental caries. Cardiopulmonary examination is normal. There are no other palpable abnormalities in the neck. Which of the following is the best next step in management of this patient? A. Contrast-enhanced CT scan of the neck D. CT scan with positron emission tomography C. Fine-needle aspiration biopsy D. Radioactive iodine uptake study E. Trial of oral antibiotics

Answer A Educational objective: Persistent neck masses carry a high likelihood of malignancy, especially in smokers and patients who are asymptomatic or have only nonspecific systemic symptoms. A CT scan with contrast is the preferred initial test in most patients with a persistent neck mass.

46. A 16-year-old girl is brought to the office as she has not reached menarche. The patient began to have breast development at age 12 and has had both axillary and pubic hair development. Her mother and sister underwent menarche at age 14. Vital signs are normal. Height and weight are at the 60th and 15th percentiles, respectively. Physical examination shows Tanner stage 3 breast and pubic hair development. External genitalia appear normal and internal pelvic examination is deferred. Pelvic ultrasound shows a normal uterus. Which of the following is the best next step in management of this patient? A. FSH level B. Karyotype analysis C. MRI of the pituitary D. Reassurance and reevaluation in 6 months E. Testosterone level

Answer A Educational objective: Primary amenorrhea, the absence of menarche by age 15 in girls with secondary sexual characteristics, is often due to genetic or anatomic abnormalities that result in hypothalamic-pituitary-ovarian axis dysfunction. Therefore, initial management includes evaluating for a uterus and obtaining a FSH level.

6. A 32-year-old woman comes to the office with ear pain. She has seasonal allergies for which she has been taking over-the-counter decongestants. Two days ago, the patient began having throbbing right ear pain that became intense yesterday. The pain is much better today, but she noticed a white discharge on her pillow when she awoke this morning. Otoscopic examination shows a cloudy and red tympanic membrane with an area of perforation. Which of the following is the most likely outcome of this patient's condition? A. Complete resolution with oral and topical antibiotics B. Complete resolution with topical acetic acid C. Extension to the mastoid process D. Mild to moderate long-term hearing loss E. Need for reconstructive surgery

Answer A Educational objective: Rupture of the tympanic membrane in adults with acute otitis media is characterized by purulent otorrhea and relief of the initial ear pain. It usually resolves completely with a combination of oral and topical antibiotics.

12. A 42-year-old woman comes to the office to discuss sexual symptoms. The patient divorced 5 years ago and had not had sexual intercourse until 9 months ago, when she became sexually active with a new partner. She enjoys spending time with her partner but has no desire for, and does not initiate, sexual activity, which causes significant stress on her relationship. The patient has no pain during sexual activity and no medical conditions. Her last menstrual period was a week ago. Pelvic examination is normal. Which of the following is most appropriate treatment for this patient? A. Oral flibanserin B.Oral ospemifene C. Oral sertraline D. Pelvic floor physical therapy E. Vaginal estrogen

Answer A Educational objective: Sexual interest/arousal disorder, a common sexual disorder in women, is marked by persistent low sexual desire. Flibanserin has been approved for premenopausal women by the US Food and Drug Administration as it produces modest improvements in sexual desire.

10. A 63-year-old man with diabetes is hospitalized with right foot methicillin-resistant Staphylococcus aureus osteomyelitis complicated by sepsis and respiratory failure. He spends 3 days in the intensive care unit on mechanical ventilation but is now extubated and has resumed eating. He has generalized weakness, mild cough, and foot pain. The patient has no chest pain, abdominal pain, bleeding, or diarrhea. He has no prior history of pneumonia, heart failure, or coagulopathy. His current medications include intravenous vancomycin, subcutaneous heparin, short- and long-acting subcutaneous insulin, and omeprazole. His vital signs are within normal limits.Laboratory values show hemoglobin of 12.2 g/dL, creatinine of 1.4 mg/dL, and albumin of 2.5 mg/dL. Coagulation profile is unremarkable. Which of the following should be strongly considered when transferring this patient to the medical floor? A. Discontinuing omeprazole B. Starting an albumin infusion C. Starting oral diabetic medications D. Switching from intravenous vancomycin to oral rifampin E. Switching from pharmacologic to mechanical thromboprophylaxis

Answer A Educational objective: Stress ulcer prophylaxis may be beneficial in high-risk critically ill patients but should be discontinued once the patient leaves the intensive care unit and no longer has an indication. Unnecessary use of proton pump inhibitors may be associated with adverse affects (eg, nosocomial pneumonia, Clostridium difficile infection) and higher costs.

24. A 72-year-old man comes to his primary care provider for a preventative examination. He currently feels well and does not have any complaints, aside from occasional pain in his knees. The patient has a 20pack-year smoking history but quit 30 years ago. He does not drink alcohol. Blood pressure is 142/70 mm Hg and heart rate is 78/min. The patient's BMI is 25 kg/m 2 . Physical examination is unremarkable. He is up-to-date on vaccinations. Colonoscopy 3 years ago and a lipid panel 1 year ago were normal. Which of the following tests is recommended for this patient? A. Abdominal ultrasound B. Carotid ultrasound C. No further testing D. Stress electrocardiogram (ECG) testing E. Transthoracic echocardiogram

Answer A Educational objective: The United States Preventive Services Task Force recommends screening male active or former smokers age 65-75 with a one-time abdominal ultrasound to evaluate for abdominal aortic aneurysm.

39. A 57-year-old woman is admitted for acute bilateral pulmonary embolism. On admission, she was found to have a right-sided pleural effusion, T-wave inversion in precordial leads, and elevated troponin I levels. On her 6th day of anticoagulation therapy, she develops right hemiparesis and word-finding difficulty. There are decreased breath sounds in the right lower lung field with dullness to percussion. Cardiac examination is unremarkable. Neurologic examination shows 2/5 muscle strength in the right upper and lower extremities. The remainder of the examination is within normal limits. Laboratory results are as follows: Hemoglobin 14.0 g/dL Platelets 50,000/µL Leukocytes 7,500/µL Activated partial thromboplastin time 60 sec (normal <25-40) Fibrin degradation products Negative Emergency head CT scan reveals no blood in the subarachnoid space or brain parenchyma. Which of the following is the most likely cause of this patient's neurologic complications? A. Antibody-mediated platelet activation B. Disseminated intravascular coagulation C. Nonimmune platelet degradation D. Platelet sequestration and redistribution E. Venous thromboembolism

Answer A Educational objective: The combination of arterial/venous thrombosis and thrombocytopenia in patients receiving heparin therapy strongly suggests type 2 heparin-induced thrombocytopenia. Heparin should be discontinued immediately and non-heparin anticoagulation initiated.

20. A study compared length of stay following early cholecystectomy (within 48 hours of presentation) as opposed to delayed cholecystectomy (after resolution of pain and laboratory study normalization) in patients with acute gallstone pancreatitis. Average length of stay was reported as 3.5 days in the early cholecystectomy group and 5.8 days in the delayed cholecystectomy group; the p-value was 0.0056. Assuming a p-value cutoff of <5%, which of the following is the best interpretation of these results regarding the difference in length of stay? A. Assuming no real difference, there is a 0.0056 probability of observing this result due only to chance B. If the study were repeated 100 times, the difference would be between 3.5 and 5.8 in 95% of cases C. The chance that the null hypothesis is true is 0.0056 D. The confidence interval is needed to determine statistical significance E. The results are not statistically significant as the p-value is >5%.

Answer A Educational objective: The p-value represents the probability of observing a given result (or more extreme) due to chance alone, assuming the null hypothesis is true.

25. A 75-year-old man comes to the physician due to tinnitus. He describes the noise as a constant highpitched ringing that has been present for more than a year. The sound does not pulsate and does not vary with position. He has no dizziness, imbalance, or problems with hearing or vision. The patient's medications include lisinopril and amlodipine for hypertension. His blood pressure is 130/72 mm Hg and pulse is 78/min. He has no abnormalities of the head, ears, eyes, nose, or throat. There are no carotid bruits and cervical spine motion is normal. Examination of the cranial nerves is also normal. Which of the following is the most appropriate next step in managing this patient's tinnitus? A. Audiometric testing B. Discontinuation of lisinopril C. MRI D. Non-contrast CT scan of the head E. Selective serotonin reuptake inhibitor

Answer A Educational objective: Tinnitus is a perception of sound inside the ear that can be pulsatile (usually of vascular etiology) or nonpulsatile (usually of non-vascular etiology). Non-pulsatile tinnitus is frequently associated with sensorineural hearing loss, and audiometric screening is recommended.

33. A 45-year-old man comes to the emergency department with upper abdominal pain. He describes it as "nagging" and constant and rates it 6/10. The pain started 6 hours ago and was not relieved by over-thecounter antacids. It gets somewhat better when he sits up and leans forward. The patient has had 2 episodes of vomiting since the pain started. He smokes a pack of cigarettes daily. He drinks 4-6 cans of beer a day and several more on weekends. He does not use recreational drugs. Temperature is 37.8 C (100 F), blood pressure is 100/70 mm Hg, pulse is 110/min, and respirations are 20/min. Abdominal examination shows mild epigastric tenderness without guarding or rebound. ECG shows sinus tachycardia and T-wave inversion in leads V4-V6. Chest x-ray shows a small left-sided pleural effusion.Which of the following is the most likely diagnosis in this patient? A. Acute pancreatitis B. Intra-abdominal abscess C. Mesenteric ischemia D. Myocardial infarction E. Peptic ulcer perforation

Answer A Educational objective: Acute pancreatitis is characterized by epigastric abdominal pain associated with nausea and vomiting. Heavy alcohol use and gallstone disease are the most common causes. Potential complications include pleural effusion, acute respiratory distress syndrome, ileus, and renal failure.

2. A 6-year-old boy is brought to the physician during the spring with a 3-week history of runny nose and nasal congestion. He also has been rubbing his eyes, especially when he plays in the backyard. He has had no fever or other symptoms. Vital signs are normal. Examination shows dark circles under the eyes and cobblestoning of the posterior oropharynx. The nasal mucosa is pale and covered with clear rhinorrhea. Which of the following is the most appropriate treatment for this patient's condition? A. Intranasal corticosteroid B. Intranasal decongestant C. Oral antibiotic D. Oral first-generation antihistamine E. Oral leukotriene antagonist

Answer A Educational objective: Allergic rhinitis is caused by exposure to environmental allergies. Common manifestations include rhinorrhea, nasal congestion, sneezing, ocular pruritus, cough, and fatigue. First-line treatment consists of an intranasal glucocorticoid spray.

65. A 39-year-old woman comes to the emergency department with intense upper abdominal pain that started after her dinner an hour ago. She also has right shoulder pain and feels nauseated. Her symptoms subside before she is examined by the physician. The patient describes 2 similar episodes of lesser intensity in the past 6 months.Her vital signs are within normal limits. Abdominal examination shows no tenderness or guarding. Laboratory studies show normal liver function tests and lipase levels. Which of the following is the most appropriate next step in management? A. Abdominal ultrasound B. CT of the abdomen with contrast C. Omeprazole prescription and routine follow-up D. Stool Helicobacter pylori antigen testing E. Upper gastrointestinal endoscopy

Answer A Educational objective: Biliary colic typically presents with postprandial epigastric abdominal pain that can radiate to the right shoulder and may be associated with nausea, vomiting, or diaphoresis. Abdominal examination and laboratory tests are usually normal. Abdominal ultrasound is the initial imaging study of choice.

54. A 46-year-old man comes to the physician with right upper-quadrant fullness and occasional nausea. He has been obese since his early teens and his medical follow-up has been inconsistent. The patient's blood pressure is 140/90 mm Hg and body mass index is 37 kg/m 2 . Examination shows hepatomegaly.Fasting laboratory results are as follows:Albumin 4.0 mg/dL,Total bilirubin 1.0 mg/dL,Alkaline phosphatase 100 U/L,Aspartate transaminase122 U/L,Alanine transaminase 131 U/L,Blood glucose 107 mg/dL,Creatinine 0.8 mg/dLViral hepatitis serologies are negative and ultrasound of the liver shows hyperechoic liver architecture.Which of the following is the most appropriate next step in management of this patient? A. Alcohol use screening B. Bariatric surgery referral C. Liver biopsy D. Urine toxicology screening E. Weight loss and metformin

Answer A Educational objective: Diagnostic criteria for nonalcoholic fatty liver disease include the presence of hepatic steatosis on imaging or biopsy, absence of significant alcohol use, and exclusion of other etiologies of fatty liver (eg, viral and autoimmune hepatitis, hemochromatosis, medications). The primary treatment is weight reduction.

11. A 46-year-old woman comes to the physician due to exertional dyspnea over the last 6 months. She was diagnosed with Hodgkin's lymphoma 2 years ago and was treated with doxorubicin, bleomycin, vinblastine, and dacarbazine. Chest examination shows a laterally displaced apical impulse. Laboratory studies, including complete blood count and lactate dehydrogenase, are normal. Which of the following conditions is the most likely cause of this patient's presentation? A. Anthracycline toxicity B. Bleomycin toxicity C.Pleural disease D.Recurrence of lymphoma E. Secondary malignancy

Answer A Educational objective: Doxorubicin and other anthracyclines cause cardiotoxicity. Risk factors for developing cardiotoxicity include exposure at an early age, pre-existing cardiovascular disease, concurrent use of other cardiotoxic agents, chest irradiation, and bone marrow transplantation.

13. A 36-year-old woman comes to the emergency department for evaluation of sudden-onset left lower chest pain and difficulty in breathing. One week ago, the patient had swelling of the right leg, but she did not seek medical attention. The patient takes no prescription or over-the-counter medication. She has not had any recent trauma or surgery, and she has not traveled. She has no significant personal or family history of disease. Her BMI is 25 kg/m 2 . Urine pregnancy test is negative. CT scan of the chest with contrast reveals filling defects in the pulmonary artery. Which of the following is the most likely predisposing condition to this patient's disease? A. Activated protein C resistance B. Antiphospholipid antibodies C.Antithrombin III deficiency D. Low protein C levels E. Prothrombin gene mutation

Answer A Educational objective: Factor V Leiden is the most common inherited thrombophilia, accounting for approximately 40%-50% of the inherited thrombophilias.

7. A 9-month-old full-term girl is brought in for a well-child visit. She is breastfed exclusively every 3-4 hours for approximately 20 minutes on each breast. The patient takes vitamin D supplement drops, and her immunizations are up to date. She lives in a house that was built 2 years ago. Family history is unremarkable. The patient's growth, development, and physical examination are normal. Laboratory results are as follows: Hemoglobin 10 g/dL Mean corpuscular volume 70 fL Platelets 280,000/mm Leukocytes 7,500 cells/mm Which of the following is the best next step in management of this patient? A. Initiate iron supplementation B. Observe and repeat hemoglobin at age 1 year C. Order hemoglobin electrophoresis D. Order serum ferritin, total iron binding capacity, and transferrin saturation E. Perform fecal occult blood test

Answer A Educational objective: Inadequate ingestion of iron-rich foods is a significant risk factor for iron deficiency anemia in infants. Universal screening for anemia should be performed at age 9-12 months. Treatment consists of a trial of ferrous sulfate supplementation.

62. A 72-year-old man is evaluated for refractory constipation. He recently had an episode of stool impaction requiring manual disimpaction and is now passing flatus and small amounts of hard stool. He has no abdominal pain, vomiting, or hematochezia. The patient is receiving palliative care for stage IV lung cancer with multiple skeletal metastases and has adequate pain control with a fentanyl patch and oral immediaterelease morphine. His other medications include a fiber supplement, senna-docusate combination, and oral lactulose.Examination shows a frail-looking, cachectic man. Bowel sounds are normal. His abdomen is soft, nondistended, and nontender.Which of the following is the most appropriate next step in managing this patient's constipation? A. Add methylnaltrexone therapy B. Administer mineral oil enema C. Change morphine to hydromorphone D. Discontinue fentanyl patch E. Start misoprostol therapy

Answer A Educational objective: Initial management of opioid-induced constipation typically includes increasing fluid/fiber intake and physical activity in addition to therapy with daily senna-docusate or an osmotic laxative (eg, polyethylene glycol, lactulose). Refractory constipation may respond to treatment with methylnaltrexone, a peripherally acting opioid antagonist.

29. A 34-year-old woman comes to the physician for abdominal pain, bloating, and cramping over the past year. Her bowel movements fluctuate between constipation for several days and then 3-4 loose bowel movements in a single day. She often awakens at night with abdominal cramps followed by diarrhea. On examination, the abdomen is nontender without hepatosplenomegaly. Rectal examination shows normal rectal tone and brown stool in the rectal vault that is guaiac negative. Laboratory testing shows a white blood cell count of 6,200/µL, hemoglobin of 10.8 mg/dL, mean corpuscular volume of 82 fL, and creatinine level of 0.8 mg/dL.Which of the following is the most appropriate next step in management of this patient? A. Further diagnostic testing B. Low-dose tricyclic antidepressant C. Lubiprostone D. Reassurance and frequent office visits E. Rifaximin

Answer A Educational objective: Irritable bowel syndrome (IBS) is characterized by chronic abdominal pain with alternating episodes of constipation and diarrhea. Warning signs/symptoms (nocturnal or worsening abdominal pain, weight loss, rectal bleeding, and laboratory abnormalities) suggest an alternate diagnosis and require additional investigation.

58. A 32-year-old woman is being evaluated for abnormal liver function tests. She has mild fatigue but no abdominal symptoms. Her other medical problems include gastroesophageal reflux disease, vitiligo, and hypothyroidism. She also takes iron supplements for anemia diagnosed 1 year ago. Abdominal examination is unremarkable.Laboratory results are as follows:Complete blood count,Hemoglobin 10.2 g/dL ,Mean corpuscular volume 72 fL ,Platelets 245,000 / µL ,Leukocytes 3900/µL,Albumin 3.2 g/dL,Alkaline phosphatase 58 U/L,Aspartate aminotransferase 98 U/L,Alanine aminotransferase 100 U/L,TSH 4.0 µU/mL.The viral hepatitis panel is negative. Which of the following is the most likely diagnosis? A. Celiac disease B. Hereditary hemochromatosis C. Lymphocytic colitis D. Occult alcohol abuse E. Primary biliary cirrhosis

Answer A Educational objective: Most patients with celiac disease are asymptomatic or have non-specific manifestations (eg, fatigue, minor gastrointestinal complaints, borderline iron deficiency anemia, transaminase elevations). Diagnosis can be confirmed with tissue transglutaminase IgA antibody.

30. A 66-year-old man comes to the physician due to fatigue for the last 6 months. His medications include valsartan for hypertension and atorvastatin for hyperlipidemia. He has a 30-pack-year smoking history but quit 10 years ago. The patient has had no laboratory studies for the last 2 years. His blood pressure is 130/80 mm Hg and heart rate is 90/min. Physical examination shows conjunctival pallor. His lungs are clear to auscultation, and no heart murmurs are heard. The abdomen is soft and nontender. There is no peripheral edema or sensory loss in the lower extremities. Laboratory results are as follows: Hemoglobin 7.5 g/dL White blood cells 2,000/ µL Platelets 56,000/ µL Mean corpuscular volume 106 fL Creatinine 1.2 mg/dL Vitamin B12 level Normal Peripheral blood smear shows ovalo-macrocytes and neutrophils with reduced segmentation. Which of the following is the best next step in management of this patient? A. Bone marrow biopsy B. Erythropoietin therapy C. Gastroscopy for gastric biopsy D. Iron supplementation E. Serum protein electrophoresis

Answer A Educational objective: Myelodysplastic syndrome is a hematologic disorder seen in patients age >65 and is characterized by macrocytic anemia, leukopenia, and thrombocytopenia. Ovalo-macrocytes are commonly seen on peripheral smear, along with neutrophils with decreased segmentation.

68. A 54-year-old man admitted for acute pancreatitis 5 days ago has worsening abdominal pain. His temperature is 38.5 C (101.3 F), blood pressure is 100/52 mm Hg, and pulse is 105/min. Physical examination is significant for epigastric tenderness.Liver function tests and serum creatinine are normal. Blood cultures are obtained. Ultrasound of the abdomen performed on admission showed no gallstones and normal common bile duct size.Which of the following is the best next step in management of this patient? A. CT scan of the abdomen with contrast B. Intravenous ceftriaxone C. Intravenous octreotide D. Nothing by mouth and close monitoring E. Repeat ultrasound of the abdomen

Answer A Educational objective: Patients with acute pancreatitis who deteriorate 72 hours after presentation should typically receive CT scan of the abdomen with contrast to evaluate for possible complications (eg, pancreatic necrosis, infection).

24. A 35-year-old man comes to the emergency department with intense abdominal pain for the past 5 hours. He has a past medical history of nephrolithiasis and chronic right knee pain due to a meniscus tear. He takes daily naproxen. On physical examination, the patient appears distressed and lies in a fetal position. The abdomen is tense and diffusely tender. X-ray of the abdomen is shown below. Which of the following is the most appropriate next step in management of this patient? A. Intravenous antibiotics and surgical consult B. Intravenous fluids and urology consult C. CT of the abdomen with contrast D. Pain management and observation E. Upper gastrointestinal endoscopy

Answer A Educational objective: Peptic ulcer disease, especially duodenal ulcers, can be complicated by perforation and may present with signs of peritonitis with free air under the diaphragm. Most patients will require surgery. Initial medical management includes nasogastric suction, intravenous fluids, broad-spectrum antibiotics, and intravenous proton pump inhibitor therapy.

20. A 35-year-old woman with ulcerative pancolitis comes to the physician with occasional crampy abdominal pains and infrequent diarrhea. She has no nausea, vomiting, blood in the stool, or weight loss. She was diagnosed 8 years ago and her medications include daily 5-aminosalicylic acid and glucocorticoid suppositories. Six months ago, an exacerbation of her colitis was treated with oral glucocorticoids and antibiotics. Her family history is negative for inflammatory bowel disease and cancer. Which of the following is the best management for this patient? A. Offer surveillance colonoscopy now and then annually B. Offer surveillance colonoscopy now and then every 5 years C. Recommend prophylactic colectomy with ileal pouch D. Recommend surveillance colonoscopy starting at age 45 E. Surveillance colonoscopy is not indicated as the colitis is controlled

Answer A Educational objective: Ulcerative colitis is associated with an increased risk for colorectal cancer. Patients should begin surveillance colonoscopy 8 years after the diagnosis (surveillance may begin later if only the left colon is involved). Colonoscopy should be repeated every 1-2 years to assess for development of colonic dysplasia.

24. A randomized trial conducted several years ago compared an open to a laparoscopic procedure for an acute surgical problem. Whenever an eligible patient was admitted and enrolled in the study, the surgical resident responsible for that patient randomly selected a sealed envelope to allocate the patient to either the open or the laparoscopic procedure group. The resident then contacted the corresponding attending to perform the procedure. The laparoscopic procedure took longer, and the presence of the attending was not required at nighttime for the open procedure. As a result, depending on the on-call attending and how busy the operating room schedule was, the nighttime residents would sometimes hold the translucent envelopes up to the light and try to pick an envelope allocating the patient to an open procedure. This would allow them to avoid calling in an attending or occupying the operating room for prolonged periods of time. Consequently, more open procedures were happening overnight, introducing a bias into the analysis. Which of the following contributed the most to this bias? A. Allocation concealment B. Berkson bias C. Blinding D. Friend control bias E. Sampling bias

Answer A Educational objective: Allocation concealment means that the person randomizing participants in a trial does not know to which group the next participant will be assigned. Allocation concealment occurs prior to the beginning of the intervention, when participants are being assigned to groups.

27. A 17-year old girl with Turner syndrome comes to the physician for a routine check-up. She initially presented at age 15 with primary amenorrhea and a karyotype showed 45,X. A recent echocardiograph showed a bicuspid aortic valve with normal aortic root diameter and no other abnormalities. The patient should have ongoing cardiac surveillance due to the lifelong risk for which of the following problems? A. Aortic dissection B.Coronary artery aneurysm C.Hypertrophic cardiomyopathy D. Mitral valve stenosis E. Pulmonary arterial hypertension

Answer A Educational objective: Patients with Turner syndrome are at lifelong risk for aortic dilatation and dissection, and the presence of a bicuspid aortic valve is a significant risk factor. Periodic echocardiography and MRI surveillance is required in these patients

34. A 65-year-old man is brought to the emergency department from his office due to palpitations that started an hour ago. He has a history of coronary artery disease and myocardial infarction 2 years ago. He also has diet-controlled type 2 diabetes mellitus. The patient's blood pressure is 122/90 mm Hg and his oxygen saturation is 96% on room air. He appears mildly diaphoretic but not in respiratory distress. He has a palpable rapid pulse. His ECG is shown in this exhibit. Which of the following medications is most likely to improve this patient's condition? A. Adenosine B. Amiodarone C.Digoxin D. Metoprolol E. Verapamil

Answer B Educational objective: A wide-complex tachycardia should be considered to be ventricular in origin unless it is definitively interpreted as supraventricular tachycardia with aberrancy. In stable patients, treatment consists of a loading dose of amiodarone.

49. A 65-year-old woman with long-standing type 2 diabetes mellitus is found to have high-grade albuminuria. She also has mild nonproliferative diabetic retinopathy. Her blood pressure is 112/80 mm Hg. Serum creatinine is 1.2 mg/dL and potassium is 4.9 mEq/L.Prescribing an angiotensin II receptor blocker as opposed to an ACE inhibitor would decrease the risk of which of the following in this patient? A. Acute renal failure B.Angioedema C.Hyperkalemia D.Hypotension E. Malignancy

Answer B Educational objective: ACE inhibitors are the leading cause of drug-induced angioedema. In comparison, angiotensin II receptor blockers are associated with a significantly lower risk of angioedema.

4. A 63-year-old man comes to the urgent care clinic with scrotal pain over the last 2 days. He has also been experiencing urinary frequency and urgency attributed to a "prostate problem." His temperature is 38.1 C (100.8 F), blood pressure is 130/75 mm Hg, pulse is 86/min, and respirations are 15/min. Physical examination shows left scrotal swelling and a tender scrotal mass; there is no urethral discharge. Laboratory studies show a white blood cell count of 14,000/µL with a left shift. Urinalysis shows bacteriuria and pyuria. Which of the following is the most likely organism responsible for this patient's findings? A. Chlamydia trachomatis B. Escherichia coli C. Neisseria gonorrhoeae D. Pseudomonas E. Staphylococcus aureus

Answer B Educational objective: Acute epididymitis in younger patients is usually due to sexually transmitted organisms such as Chlamydia trachomatis or Neisseria gonorrhoeae. Empiric therapy should include ceftriaxone and doxycycline. In older men, epididymitis is usually non-sexually transmitted and caused by common gram-negative organisms. Fluoroquinolones are effective in most cases.

47. A 34-year-old man with a history of type 1 diabetes mellitus comes to the emergency department with retrosternal chest pain for the past 12 hours. His symptoms started last night and woke him up several times because he could not find a comfortable sleeping position. He also has pain in his left shoulder on deep inspiration. His temperature is 37.8 C (100.2 F), blood pressure is 110/70 mm Hg, and pulse is 100/min. No murmurs are heard on cardiac auscultation. Lungs are clear to auscultation. An ECG is obtained as shown in the exhibit.Which of the following is the most likely diagnosis? A. Acute coronary syndrome B.Acute pericarditis C. Aortic dissection D. Pericardial effusion E. Pulmonary embolism

Answer B Educational objective: Acute pericarditis is characterized by pleuritic chest pain, pericardial friction rub, ECG changes, and new or worsening pericardial effusion. ECG in acute pericarditis is characterized by diffuse, concave upward ST segment elevation and PR depression.

5. A 22-month-old boy is brought to the office with fever and ear pain. His parents say that "he's been tugging on both ears non-stop this morning." He has otherwise been eating and drinking well and is playful. The boy has no previous history of acute otitis media and his vaccinations are up-to-date. He has no known drug allergies. Temperature is 38.3 C (101 F). On examination, bilateral tympanic membranes are dull, red, and bulging moderately, with mildly decreased mobility with air insufflation. The remainder of the physical examination, including palpation of the neck and mastoids, is unremarkable. Which of the following is the best next step in management of this patient? A. Cefdinir B. High-dose amoxicillin C. High-dose amoxicillin with clavulanate D. Observation, follow-up in 2 days if no improvement E. Tympanocentesis

Answer B Educational objective: Children with severe acute otitis media who are age <2 years or have bilateral disease require antibiotic therapy. High-dose amoxicillin is first-line therapy.

11. A 64-year-old man with ischemic cardiomyopathy and systolic heart failure comes to the physician for follow-up. The patient has New York Heart Association functional class II symptoms and shows no signs of fluid overload. His daily medications include furosemide, carvedilol, lisinopril, atorvastatin, and low-dose aspirin. The patient's blood pressure is 155/90 mm Hg and pulse is 60/min. His potassium level is 4.2 mEq/L and serum creatinine level is 0.9 mg/dL. Adding losartan to this patient's current regimen is most likely to result in which of the following? A. Higher risk for heart failure-associated hospital admission B. Higher risk for renal failure C. Lower risk for all-cause mortality D. Lower risk for arrhythmia E. Lower risk for hyperkalemia

Answer B Educational objective: Dual blockade of the renin-angiotensin system with both ACE inhibitors and ARBs does not provide any significant benefit in cardiovascular or all-cause mortality compared to ACE inhibitors alone, and is associated with an increased risk for hyperkalemia, hypotension, and worsening renal function.

9. A 4 year-old boy is brought to the physician for a well-child visit. The patient eats a well-balanced diet and drinks 4 cups of tap water and 4 ounces of juice each day. He does not like milk. He brushes his teeth 3 times a day with fluoride toothpaste and takes an oral fluoride supplement 2 times a day. He was weaned from breastfeeding around age 18 months and never used a pacifier. His father has a history of root canals for severe dental caries. Examination shows multiple opaque, white flecks in a lacy pattern on the central and lateral upper and lower incisors. Which of the following is the most likely cause of these findings? A. Early dental caries B. Excess fluoride intake C. Inadequate calcium intake D. Local trauma secondary to brushing E. Normal primary dentition

Answer B Educational objective: Fluoride toothpaste and drinking water are important for preventing dental caries. Fluorosis can be prevented by limiting the intake of fluoride toothpaste to <0.05 mg/kg/day. Excess fluoride intake can result in hypomineralization of the dental enamel and presents as opaque, white flecks on the teeth.

17. A 22-year-old woman comes to the physician with easy bruising for the past several months. Her past medical history is significant for an uncomplicated appendectomy 4 years ago. Her temperature is 37.2 C (99 F). Chest and abdominal examination is unremarkable. There are a few ecchymoses over both legs. Laboratory results are as follows: Hematocrit 45% Platelets 9,000/µL Leukocytes 5,500/µL Neutrophils 56% Eosinophils 1% Lymphocytes 36% Monocytes 7% Fibrinogen 250 mg/dL (normal 150-350mg/dL) Prothrombin time 13 sec Which of the following is the most likely cause of this patient's condition? A. Acute myeloid leukemia. B.Immune thrombocytopenia C. Myelodysplastic syndrome D. Thrombotic microangiopathy E. von Willebrand disease

Answer B Educational objective: Immune thrombocytopenia is an autoimmune disorder characterized by increased platelet destruction and impaired platelet production. There is no definitive test for diagnosis, but patients will have isolated thrombocytopenia with otherwise normal blood counts. Treatment usually involves systemic glucocorticoids for patients with severe thrombocytopenia (platelets <30,000/µL) or significant bleeding.

25. A family physician is interested in the number of new cases of asthma that were diagnosed in a community within a 3-month period. Which of the following is the most appropriate measure to describe this value? A. Attack rate B. Incidence C. Period prevalence D. Point prevalence E. Standardized incidence ratio

Answer B Educational objective: Incidence answers the question: How many new cases of the disease developed in a population during a particular period of time?

3. A 34-year-old woman, gravida 0, comes to the office for evaluation of irregular menstrual cycles. She had been using oral contraceptive pills (OCPs) regularly since becoming sexually active at age 16, but stopped a year ago after she got married. Her menses started at age 12, and her cycles were irregular until she began taking OCPs. Now she has a menstrual period every 3-4 months that lasts 10-14 days with passage of large clots. BMI is 32 kg/m 2 . Blood pressure is 132/86 mm Hg and pulse is 82/min. Coarse hair is present on the face and in the midabdominal region. Which of the following laboratory tests is most likely abnormal in this patient? A. 17-hydroxyprogesterone B. Fasting glucose C. FSH D. Prolactin E. TSH

Answer B Educational objective: Insulin resistance is a common finding in patients with polycystic ovary syndrome. All patients with this syndrome should undergo screening with either a 75-g 2-hour oral glucose tolerance test or fasting glucose with hemoglobin A1c.

6. A 4-year-old girl with a peanut allergy is brought to the emergency room for difficulty breathing. The girl accidentally ate part of a peanut butter cookie and experienced facial swelling and wheezing approximately 30 minutes later. Her parents administered an epinephrine auto-injector with no relief. The girl's blood pressure is 108/65 mm Hg, respiratory rate is 24/min, and pulse oximetry shows 92% on room air. Auscultation shows inspiratory stridor and mild expiratory wheezing. Oxygen is administered via nasal cannula. Which of the following is the next best step in management of this patient? A. Inhaled albuterol B. Intramuscular epinephrine C. Intravenous fluid replacement D. Oral diphenhydramine E. Oral prednisolone

Answer B Educational objective: Intramuscular epinephrine administration is the most critical step in the treatment of anaphylaxis as it can relieve airway obstruction and hypotension. Epinephrine can be administered intramuscularly every 5 minutes until symptoms are relieved. Patients with a history of anaphylaxis should carry self-injectable epinephrine with them at all times.

8. A 3-year-old boy is brought to the physician for evaluation of his behavior. Over the past 3 months, he has become fussier and moodier than usual. He was previously very talkative but his mother has noticed a decline in his vocabulary. She cannot identify any social stressors or changes at home except that the family moved into a different apartment building approximately 5 months earlier. The mother is unsure of the year it was built. The child has also been very constipated and had intermittent abdominal pain. Yesterday, he had 2 episodes of nonbloody nonbilious emesis and has been eating less than usual. Which of the following is the most appropriate next step in management of this patient? A. Abdominal radiograph B. Blood lead level C. Consulting child protective services D. Hearing screen E. Vanderbilt assessment scales

Answer B Educational objective: Lead poisoning in children can cause behavioral problems (eg, anxiety, attention-deficit hyperactivity, loss of milestones), crampy abdominal pain, vomiting, and constipation. Venous lead level is the gold standard for diagnosis. Treatment of moderate to severe lead poisoning includes chelation.

17. A new serologic marker is evaluated for pancreatic cancer screening. The researchers are concerned that the diagnosis will be made at an earlier point in time but that the prognosis or disease course will remain unchanged. As a result, the screened patients will appear to live longer from the time of diagnosis until the time of death. The researchers' concern is best described as which of the following? A. Confounding B. Lead-time bias C. Length-time bias D. Observer bias E. Sampling bias

Answer B Educational objective: Lead-time bias occurs when a screening test diagnoses a disease earlier; as a result, the time from diagnosis until death appears prolonged even though there is no actual improvement in survival.

16. A 60-year-old man comes to the office complaining of difficulty hearing for the past few weeks. His past medical history is significant for coronary artery disease, type 2 diabetes mellitus, hypertension, congestive heart failure, and stage 2 chronic renal failure. The patient was recently hospitalized for acute decompensated heart failure. His medications include low-dose aspirin, diuretics, a beta-blocker, and an angiotensin-converting-enzyme inhibitor. Examination shows bilateral hearing loss. Which of the following medications is the most likely cause of this patient's hearing problem? A. Aspirin B. Furosemide C. Hydrochlorothiazide D. Lisinopril E. Metoprolol

Answer B Educational objective: Loop diuretics can cause hearing loss and/or tinnitus. These ototoxic effects typically occur in patients who are taking high doses of loop diuretics, have coexistent renal failure, or are being treated with other known ototoxic medications (aminoglycosides).

48. A 32-year-old woman comes to the office for an annual health maintenance examination. On routine screening, the patient discloses that her husband has hit her multiple times. He has become increasingly controlling and violent and is drinking more heavily since losing his job recently. The patient has not told anyone else due to fear of retaliation. Multiple bruises in various stages of healing are noted over her chest, back, and abdomen. Which of the following is the best next step in management of this patient? A. Contact a family member B. Create a safety plan C. Refer to couples therapy D. Report the situation to the police E. Speak with the patient's husband

Answer B Educational objective: Management of patients who disclose intimate partner violence includes assessing the risk of immediate danger and assisting in creation of a safety plan that includes a safe place to go should urgent escape be necessary.

6. A 33-year-old man with type I diabetes comes to the physician to discuss gastroparesis. He eats frequent, small meals but has abdominal pain, bloating, and nausea after eating. Over the last 6 months, the patient has lost 5 kg (11 lb). His insulin regimen has been optimized, and his most recent hemoglobin A1c was 6.7%. Metoclopramide therapy is initiated, and he is scheduled for a follow-up appointment. This patient should be monitored for which of the following complications of drug therapy? A. Change in mental status B. Extrapyramidal neurologic signs C. Prolongation of QT interval D. Respiratory depression E. Sudden loss of vision

Answer B Educational objective: Metoclopramide is indicated for treatment of diabetic gastroparesis and is the preferred first-line medication. However, it is associated with a significant risk for extrapyramidal side effects, including tardive dyskinesia.

40. A 15-year-old girl comes to the office for a routine examination. The patient has no chronic medical conditions. Her menses occur monthly; she has associated breast tenderness and abdominal bloating that begin a week prior to menses and resolve on the second day of her menstrual period. She also has some associated changes in appetite and decreased energy, but they do not require her to miss school or other activities. The patient is not sexually active. Vital signs and physical examination are normal. Which of the following is the most appropriate management for this patient's symptoms? A. Combination oral contraceptives B. No further management indicated C. Progestin-containing subdermal implant D. Selective serotonin uptake inhibitor E. Vitamin B6 and calcium supplements

Answer B Educational objective: Mild molimina symptoms (ie, do not cause distress or functional impairment) during the luteal phase of the menstrual cycle are normal and do not require treatment. In contrast, patients with severe symptoms that cause functional impairment (ie, premenstrual syndrome or premenstrual dysphoric disorder) require treatment; first-line management is with a selective serotonin reuptake inhibitor.

12. A 28-year-old woman comes to the office with abdominal pain and diarrhea for the last 6 months. She was treated for an anal fissure 4 months ago and renal colic 2 months ago. The patient is afebrile. On examination, she has several shallow oral ulcers and tenderness in the right lower quadrant without rebound. Her hemoglobin is 10 g/dL and platelet count is 500,000/mm 3 . Rectosigmoidoscopy is unremarkableWhich of the following is the most likely diagnosis? A. Celiac disease B. Crohn's disease C. Irritable bowel syndrome D. Lactose intolerance E. Ulcerative colitis

Answer B Educational objective: Most patients with Crohn's disease have involvement of the colon and small intestine, but the rectum is spared in 50% of cases. Extraintestinal manifestations are common and can include oral aphthous ulcers, anal fissure, and nephrolithiasis.

17. A 31-year-old woman comes to the clinic for routine follow-up. The patient has no concerns other than occasional palpitations that she notices while sitting on the couch in the evenings. She has generalized anxiety disorder with frequent panic attacks, involving chest discomfort and shortness of breath, that have been well-controlled for the past year with escitalopram. The patient was also recently diagnosed with mitral valve prolapse. Blood pressure is 118/72 mm Hg, pulse is 60/min, and respirations are 12/min. Physical examination is unremarkable other than a midsystolic click and a late systolic murmur at the cardiac apex. The patient asks about potential complications of mitral valve prolapse. Which of the following is the most important determinant of adverse cardiovascular events in this patient? A. Anxiety disorder B. Degree of mitral regurgitation C. Nonexertional chest pain D. Resting bradycardia E. Ventricular premature beats

Answer B Educational objective: Most patients with mitral valve prolapse have a risk for cardiovascular morbidity and mortality similar to that of the general population. However, moderate to severe mitral regurgitation and a reduced ejection fraction <50% are associated with an increased risk for adverse cardiovascular events, including cardiac arrhythmia, embolic stroke, infective endocarditis, decompensated heart failure, and sudden cardiac death.

12. An 18-year-old woman is diagnosed with iron deficiency anemia secondary to heavy menstrual periods from von Willebrand disease. The patient is prescribed oral iron therapy and educated about potential side effects of treatment, such as abdominal discomfort and nausea. Which of the following is the best method of increasing oral iron absorption? A. Taking iron with a proton pump inhibitor B.Taking iron with juice C.Taking iron with milk D.Using an enteric-coated preparation E. Using ferrous gluconate formulation

Answer B Educational objective: Oral iron therapy is the gold standard treatment for iron deficiency anemia and should be taken with juice or a vitamin C supplement to enhance intestinal absorption. Concomitant use of antacids, H2 receptor antagonists, or proton pump inhibitors can decrease the efficacy of iron therapy. Foods containing calcium, phosphorus, phytate, and tannate can also decrease absorption.

18. A 74-year-old man comes to the physician for follow-up of type 2 diabetes mellitus. His diabetes is managed with diet and metformin, and he has shown good compliance with medical visits. The patient has no history of chest pain, dyspnea, lightheadedness, syncope, or falls. He does not use tobacco, alcohol, or illicit drugs. Blood pressure and pulse are 145/75 mm Hg and 67/min while supine, and 122/64 mm Hg and 72/min after standing for 2 minutes. Physical examination is otherwise unremarkable. His last HbA1c was 7% and creatinine was 0.8 mg/dL. The postural blood pressure changes are reproduced during the subsequent visit, but the patient consistently reports no symptoms. This patient has an elevated risk for which of the following? A. Abdominal aortic aneurysm B. All-cause mortality C. Chronic venous insufficiency D. Parkinson disease E. Subarachnoid bleeding

Answer B Educational objective: Orthostatic hypotension is associated with an increased risk of cardiovascular disease and all-cause mortality.

37. A 58-year-old woman comes to the office due to leakage of urine. The patient has had increased urinary frequency over the last 3 months, often voiding 1 or 2 times every hour and multiple times each night. She has difficulty initiating urination and voids a small volume of urine. The patient now has continued leakage of urine, requiring her to wear an incontinence pad. She has no dysuria or hematuria. The patient has a history of depression and switched from sertraline to amitriptyline 4 months ago. She underwent a total abdominal hysterectomy 2 years ago for endometrial hyperplasia. She does not use tobacco, alcohol, or illicit drugs. BMI is 30 kg/m 2 . Vital signs are normal. Abdominal examination reveals a lower abdominal mass. Pelvic examination reveals continuous leakage of urine from the urethra; no pooling is seen in the vaginal vault. A small amount of urine is leaked with cough-stress test; postvoid residual volume is 500 mL. Which of the following is the most likely diagnosis in this patient? A. Interstitial cystitis B. Overflow incontinence C. Urethral diverticulum D. Urgency incontinence E. Vesicovaginal fistula

Answer B Educational objective: Overflow incontinence is caused by underactive detrusor function or bladder outlet obstruction. Anticholinergics (eg, tricyclic antidepressants, antihistamines, antimuscarinics) relax the detrusor muscle, increase bladder capacity, and cause urinary retention. Chronic urinary retention leads to dribbling, incomplete emptying, and a large postvoid residual volume.

11. A new cardiovascular medication is found to decrease the incidence of premature ventricular contractions on electrocardiogram. This is an example of: A. Both disease-oriented evidence and patient-oriented evidence B. Disease-oriented evidence C. Patient-oriented evidence that matters D. Neither disease-oriented evidence nor patient-oriented evidence

Answer B Educational objective: Patient-oriented evidence that matters (POEM) deals with outcomes relevant to patients (eg, morbidity, mortality). Disease-oriented evidence (DOE) is concerned with surrogate end points (eg, laboratory values or other measures of response).

25. A 13-year-old girl is brought to the office due to lower abdominal pain. The pain began a year ago, occurs every month, and lasts for several days before resolving on its own. However, over the last 2 months, the pain has become more severe, and the patient has associated dyschezia. The patient has no chronic medical conditions or previous surgeries. She has not had a menstrual period and is not sexually active. Vital signs are normal. BMI is at the 50th percentile. The abdomen has tenderness to palpation over the bilateral lower quadrants and an enlarged suprapubic mass. On pelvic examination, there is a smooth, blue mass protruding through the vaginal introitus. Which of the following is the most likely diagnosis in this patient? A. Endocervical polyp B. Imperforate hymen C. Pelvic organ prolapse D. Sarcoma botryoides E. Uterine leiomyoma

Answer B Educational objective: Patients with an imperforate hymen have cyclic lower abdominal pain, primary amenorrhea, an enlarged suprapubic mass (ie, uterus), and a bluish bulge protruding from the vagina. Management is surgical (ie, hymenectomy).

11. A 66-year-old man comes to the emergency department with abdominal pain and poor appetite. The previous day, he underwent a screening colonoscopy and had 2 polyps removed from the sigmoid colon. He has no nausea, vomiting, or bloody stools. His temperature is 38.1 C (100.6 F), blood pressure is 110/70 mm Hg, and pulse is 98/min. His abdomen is tender in the periumbilical area and left lower quadrant without rebound tenderness. White blood cell count is 12,300/mm 3 , and serum creatinine level is 1 mg/dL. Upright abdominal films show no free air under the diaphragm.Which of the following is the best next step in management of this patient? A. Barium enema D. CT scan of the abdomen with water-soluble contrast C. Noncontrast CT scan of the abdomen D. Oral antibiotics and follow-up in 2 days E. Sigmoidoscopy

Answer B Educational objective: Perforation is a possible complication of colonoscopy. Immediate abdominal (upright or lateral decubitus) and upright chest radiographs may identify free air. Patients with negative x-rays should undergo CT scan with a water-soluble contrast agent, which can detect subtle perforation.

34. A 20-year-old woman, gravida 1 para 0 aborta 1, comes to the office due to 7 months of amenorrhea. Menarche was at age 12; menses were typically every 28 days and consisted of 5-7 days of bleeding. The patient has gained 6.8 kg (15 lb) over the past year despite increased exercise and has become increasingly fatigued. She has no chronic medical conditions. The patient underwent an elective termination of pregnancy at age 18. Blood pressure is 110/60 mm Hg and pulse is 52/min. BMI is 25 kg/m 2 . On physical examination, the skin is dry, and on pelvic examination, the ovaries are small and nontender. The remainder of the examination is otherwise unremarkable. Urine pregnancy test is negative. Which of the following is the most likely diagnosis in this patient? A. Asherman syndrome B. Hypothyroidism C. Pituitary adenoma D. Polycystic ovary syndrome E. Primary ovarian insufficiency

Answer B Educational objective: Primary hypothyroidism typically presents with fatigue, bradycardia, weight gain, and dry skin, as well as secondary amenorrhea.

7. A large multicenter double-blinded randomized placebo-controlled study is being planned to evaluate a new medication. Primary endpoints addressing mortality benefits are chosen, and a comprehensive analysis strategy is specified, including for predefined subgroups. In discussing the advantages of this study design, which of the following represents a benefit of randomization? A. Randomization can be performed at the level of subgroup analysis B. Randomization distributes known and unknown variables equally among the study groups C. Randomization ensures that the study results are statistically significant D. Randomization ensures that the study sample is representative of the population E. Randomization prevents group allocation concealment from study participants and investigators

Answer B Educational objective: Randomization distributes known and unknown variables equally among the study groups.

19. A 32-year-old woman comes to the emergency department with palpitations over the last hour. She describes prior episodes of similar palpitations that usually last 10-15 minutes and terminate spontaneously. She is otherwise healthy and does not have any other medical problems. She does not use tobacco, alcohol, or illicit drugs. The patient's blood pressure is 115/70 mm Hg. Lungs are clear to auscultation and cardiac examination is notable only for tachycardia. Her electrocardiogram (ECG) is shown in this exhibit. Which of the following is the best initial treatment for this patient's condition? A. Amiodarone B. Carotid sinus massage C. Conscious sedation and cardioversion D. Dix-Hallpike maneuver E. Lidocaine

Answer B Educational objective: Stable patients with regular, narrow complex tachycardia should be managed initially with vagal maneuvers and/or intravenous adenosine. Patients with persistent tachyarrhythmia (narrow or wide complex) causing hemodynamic instability should be managed with immediate synchronized direct-current cardioversion.

4. A 16-year-old girl is brought to the office by her mother for contraception counseling. She has been sexually active with a male partner for the past 5 months. They use condoms intermittently, and the patient desires a more reliable form of contraception. She has no chronic medical conditions or allergies. The patient takes no medications and does not use tobacco, alcohol, or illicit drugs. Vital signs and physical examination are normal. Results of her gonorrhea, chlamydia, and HIV tests are negative. Benefits and risks of the various contraception options are discussed. The patient and her mother request the most effective method. Which of the following is the best next step in management of this patient? A. Administer intramuscular depot medroxyprogesterone B. Insert an intrauterine device C. Perform a Pap test D. Prescribe combined estrogen/progestin contraceptive pills E. Recommend spermicidal condoms

Answer B Educational objective: The intrauterine device (copper or levonorgestrel) and progestin implant are the most effective forms of contraception. These methods provide years of uninterrupted contraception and are rapidly reversible on removal.

36. A 49-year-old man is evaluated for a 3-week history of fatigue and anorexia. Two sets of blood cultures grow Streptococcus gallolyticus (Streptococcus bovis biotype I). Echocardiogram shows vegetations on the mitral valve. Other than antibiotic treatment, what additional step is recommended in this patient? A. Bronchoscopy B.Colonoscopy C. CT scan of the head D. Cystoscopy E. Fecal occult blood testing

Answer B Educational objective: There is a strong association between infections with Streptococcus gallolyticus (S bovis biotype I) and colonic neoplasms. Patients with S gallolyticus endocarditis should have further evaluation with colonoscopy to look for an underlying occult malignancy.

45. A 44-year-old man comes to the office for hypertension follow-up. Systolic blood pressure readings during the patient's prior visits were 150-160 mm Hg, and diastolic readings were 90-100 mm Hg. He has a strong family history of type 2 diabetes mellitus and stroke. BMI is 31.5 kg/m 2.Which of the following medications is associated with the highest risk of developing diabetes mellitus in this patient? A. Carvedilol B.Chlorthalidone C.Lisinopril D.Losartan E. Nifedipine

Answer B Educational objective: Thiazide diuretics can cause hyperglycemia and are associated with an increased risk of diabetes mellitus. They are not typically used as first-line antihypertensive agents in patients with diabetes mellitus but are beneficial as add-on therapy due to their antihypertensive efficacy and reduced risk of cardiovascular events (eg, stroke).

20. A 16-year-old girl comes to the office for evaluation of hair growth on her upper lip and lower abdomen. The hair first appeared around menarche and has increased gradually. The patient's menses started at age 13, and she has had erratic menstrual periods lasting up to 14 days every 3 weeks to 6 months. She is not sexually active. The patient has no headaches, breast discharge, or recent weight changes. BMI is 34 kg/m 2 . Blood pressure is 116/70 mm Hg, and pulse is 76/min. Laboratory results are as follows: Prolactin 17 ng/mL (normal: 2-29) TSH 2 μU/mL LH 80 mIU/mL FSH 60 mIU/mL 17-hydroxyprogesterone 10 ng/mL (normal <15) Testosterone 80 ng/mL (normal: 20-75) Dehydroepiandrosterone sulfate (DHEA-S) 360 µg/mL (normal: 75-370) Which of the following is the first-line medication for this patient's condition? A. Eflornithine topical cream B. Ethinyl estradiol/norgestimate C. Hydrocortisone D. Finasteride E. Spironolactone

Answer B Educational objective: Treatment for polycystic ovary syndrome focuses on reducing hirsutism, regulating menstrual cycles, and lowering the long-term risk of endometrial cancer. Combination oral contraceptives are the first-line treatment.

26. A 78-year-old woman comes to the physician following an emergency department visit. The patient reported recurrent palpitations but had no chest pain, shortness of breath, or syncope. She was diagnosed with atrial fibrillation. The arrhythmia then resolved spontaneously and she was released for outpatient follow-up. The patient's other medical problems include hypertension and hyperlipidemia. She takes diltiazem, losartan, atorvastatin, and aspirin. Blood pressure is 135/75 mm Hg and pulse is 75 and regular. Physical examination is normal. Electrocardiogram shows normal sinus rhythm. Transthoracic echocardiogram shows a mildly dilated left atrium and left ventricular ejection fraction of 60%. Which of the following is the most appropriate next step in management? A. Amiodarone B.Anticoagulation C. Beta blocker D. Exercise stress test E. Transesophageal echocardiogram

Answer B Educational objective: Warfarin or target-specific oral anticoagulants have been shown to significantly reduce the risk of embolization in patients with atrial fibrillation and moderate to high risk for thromboembolic events. The CHA2 -DS2 -VASc score is recommended for assessment of stroke risk in patients with nonvalvular atrial fibrillation

9. A 50-year-old man comes to the emergency department due to 4 weeks of fatigue, lower extremity swelling, and increased abdominal girth. He reports no prior medical problems but has not seen a physician for many years. Physical examination shows abdominal distension with shifting dullness and bilateral ankle edema. Laboratory testing and ascitic fluid analysis are as follows: Complete blood count Hematocrit 40% Platelets 110,000/mm Leukocytes 7,200/mm 3,Serum chemistry Sodium 130 mEq/L,Creatinine 1.2 mg/dL,Glucose 120 mg/dLLiver function studies Albumin 3.2 g/dL ,Aspartate aminotransferase (SGOT) 52 U/L,Alanine aminotransferase (SGPT) 46 U/L,Ascitic fluid ,Neutrophils 200/mm ,Albumin 1.0 g/dL ,Total protein 2.2 g/dL ,Which of the following is the most likely cause of this patient's current condition? A.Chronic pancreatitis B. Liver cirrhosis C. Nephrotic syndrome D. Spontaneous bacterial peritonitis E. Tuberculous peritonitis

Answer B Educational objective: A serum-to-ascites albumin gradient (serum albumin concentration − ascites albumin concentration) >1.1 g/dL indicates portal hypertensive etiologies (eg, cardiac ascites, cirrhosis), whereas a gradient <1.1 g/dL suggests nonportal hypertensive etiologies (eg, malignancy, pancreatitis, nephrotic syndrome, tuberculosis). Ascites fluid total protein <2.5 g/dL is consistent with a low-protein state (eg, cirrhosis, nephrotic syndrome)

5. A 3-year-old girl is brought to the emergency room for difficulty breathing that began 1 hour ago. She was playing outside at a family picnic when her parents noticed that her lips and tongue were swollen and her voice was hoarse. Her temperature is 37.2 C (99 F), blood pressure is 72/41 mm Hg, pulse is 134/min, and respirations are 36/min. Examination shows inspiratory stridor and mild wheezing bilaterally. She has erythematous, raised macules across her trunk. Which of the following is the most appropriate initial treatment for this patient's condition? A. Inhaled albuterol B. Intramuscular epinephrine C. Intravenous corticosteroids D. Intravenous normal saline bolus E. Subcutaneous epinephrine

Answer B Educational objective: Anaphylaxis is characterized by the acute onset of skin or mucosal changes (ie, hives), hypotension, and respiratory distress. In many cases, a trigger may not be immediately identifiable. Intramuscular epinephrine is the gold standard of treatment.

18. A 64-year-old man comes to the office for routine follow-up. He has a history of chronic obstructive pulmonary disease and notes mild dyspnea when walking long distances or ascending several flights of stairs, which is unchanged from his previous visits. Six months ago, the patient was diagnosed with testosterone deficiency after work-up for fatigue and decreased libido revealed low serum testosterone levels. He was initiated on injectable testosterone replacement therapy and notes a significant improvement in his symptoms. Other medical problems include hypertension and gastroesophageal reflux. The patient is an ex-smoker with a 20-pack-year history. Blood pressure is 126/78 mm Hg and pulse is 72/min. Oxygen saturation while breathing ambient air is 96% at rest and 90% after several minutes of walking. Breath sounds are distant with no wheezing. Heart sounds are normal. The abdomen is soft and nontender. Prostate size is normal. Laboratory results are as follows: Complete blood count Hematocrit 56% Platelets 385,000/mm Leukocytes 7,800/mm 3 Serum chemistry Sodium 138 mEq/L Potassium 4.5 mEq/L Blood urea nitrogen 14 mg/dL Creatinine 0.6 mg/dL Glucose 96 mg/dL Liver function

Answer B Educational objective: Approximately 25% of men on testosterone therapy develop polycythemia (hemoglobin >16.5 g/dL). Polycythemia can be associated with venous thromboembolism and cardiovascular complications. Therefore, patients require hematocrit monitoring; if levels exceed 54%, testosterone should be reduced or discontinued.

45. A 52-year-old man with a history of untreated hepatitis C comes to the office due to progressive abdominal distension over the last 6 months. He has gained 6.8 kg (15 lb) during this period. Blood pressure is 112/70 mm Hg and heart rate is 85/min. Examination shows a distended and nontender abdomen with a fluid wave and shifting dullness. There is pitting pedal edema to the mid-calves bilaterally. Serum chemistry results are as follows:Sodium 131 mEq/L,Potassium 3.5 mEq/,Creatinine 1.0 mg/dL,Albumin 3.4 g/dL Abdominal ultrasound shows a moderate amount of ascites, moderate splenomegaly, and no focal liver masses. Diagnostic paracentesis reveals ascitic fluid nucleated cell count of 200/mm 3 , protein of 2.3 g/dL, and albumin of 2.0 g/dL.Which of the following is the most appropriate initial therapy for this patient's ascites? A. Fluid restriction and bumetanide B. Furosemide and spironolactone C. Periodic large-volume paracentesis D. Propranolol and losartan E.Salt restriction and chlorthalidone

Answer B Educational objective: Ascites due to cirrhosis is typically managed with abstinence from alcohol, avoidance of drugs that cause renal vasoconstriction, salt restriction (<2 g/day), and oral diuretics (spironolactone and furosemide). Refractory ascites may be treated with serial therapeutic paracentesis and may require transjugular intrahepatic portosystemic shunt or liver transplant.

1. A 36-year-old woman, gravida 2 para 1, comes to the office to initiate prenatal care at 8 weeks gestation. The patient has mild intermittent asthma, and her last albuterol inhaler use was 8 months ago. Her husband has eczema and a severe peanut allergy. Physical examination is normal, and ultrasound shows an 8-week intrauterine gestation. The patient asks what she can do to prevent the development of food allergies in her child. Which of the following is the most appropriate advice for this patient? A. Avoid peanut products during pregnancy and use soy-based formula instead of breast milk B. Consume an allergen-unrestricted diet throughout pregnancy and while breastfeeding C. Eliminate peanut products during pregnancy and breastfeed exclusively for at least 6 months D. Increase peanut consumption during pregnancy and while breastfeeding E.No dietary allergen restrictions, but use hydrolyzed formula instead of breast milk

Answer B Educational objective: Children who have a first-degree relative with food allergies or a family history of atopy or asthma are at increased risk for food allergies. Neither maternal avoidance nor supplementation of allergenic foods during pregnancy and lactation reduces the risk for childhood food allergies. All pregnant women should eat a healthy, well-balanced diet.

14. A 48-year-old man is evaluated for 6 months of intermittent upper abdominal pain and loose stools. He describes his stools as bulky and difficult to flush. The patient has no dysphagia, vomiting, black stools, or weight loss. He admits to drinking alcohol daily but has recently decreased his intake. Abdominal examination is unremarkable. Which of the following is most likely to establish the diagnosis in this patient? A. Anti-tissue transglutaminase antibodies B. CT scan of the abdomen C. Lactose breath hydrogen test D. Serum lipase E. Upper gastrointestinal endoscopy

Answer B Educational objective: Chronic pancreatitis usually presents with episodic or daily abdominal pain and findings of pancreatic insufficiency (eg, malabsorption, diabetes mellitus). Diagnosis is typically confirmed with abdominal CT or magnetic resonance cholangiopancreatography showing calcifications, dilated ducts, and an enlarged pancreas.

21. A 45-year-old man is hospitalized with a bleeding duodenal ulcer. He is transfused with 2 units of packed red blood cells. Two hours after starting the transfusion, the patient reports shaking chills. His temperature is 38.8 C (102 F), blood pressure is 120/76 mm Hg, pulse is 100/min, and respirations are 18/min. The transfusion is stopped, and he receives acetaminophen. Direct antiglobulin test and plasma free hemoglobin level are negative. The reaction resolves 6 hours after the transfusion began. Which of the following actions could have prevented this reaction? A. Careful cross-matching of blood B. Leukoreduction C. Using whole blood D. Warming blood E. Washing red blood cells

Answer B Educational objective: Febrile nonhemolytic transfusion reaction is the most common adverse reaction that occurs within 1-6 hours of transfusion. Patients usually develop fever, chills, and malaise without hemolysis. Leukoreduction of donor blood can prevent febrile nonhemolytic reaction and reduce the risk of human leukocyte antigen alloimmunization and cytomegalovirus transmission.

48. A 17-year-old boy is evaluated for intermittent yellow eyes but is otherwise asymptomatic. Physical examination is unremarkable.Laboratory testing shows a normal hemoglobin level, mild unconjugated hyperbilirubinemia, and normal serum alkaline phosphatase and aminotransferase levels.Which of the following is the most appropriate next step in this patient's evaluation? A. Abdominal ultrasound B. Blood smear and reticulocyte count C. Bone marrow biopsy D. Liver biopsy E. Serum protein electrophoresis

Answer B Educational objective: Gilbert syndrome is a benign condition that typically presents in adolescent boys with isolated unconjugated hyperbilirubinemia. Hematologic and liver chemistry tests should be obtained to exclude alternate conditions (eg, hemolytic anemia, hepatocellular disease), but invasive studies are not necessary. No treatment is needed.

16. A 50-year-old man comes to the physician for follow-up after admission for alcohol withdrawal. He has a 20-year history of heavy drinking but says he has been abstinent since discharge. The patient was diagnosed with cirrhosis during the hospital stay. His blood pressure is 112/78 mm Hg and heart rate is 79/min. On examination, the abdomen is mildly distended with flank dullness. There is trace lowerextremity edema. Esophagogastroduodenoscopy demonstrates small nonbleeding esophageal varices with areas of redness. Prothrombin time is 12 seconds, serum albumin is 3.6 g/dL, and creatinine is 1.1 mg/dL. Which of the following is the best initial therapy for this patient? A. Angiotensin-converting enzyme inhibitors B. Beta-adrenergic receptor blockers C. Endoscopic sclerotherapy D. H2 histamine receptor blockers E. Octreotide

Answer B Educational objective: In patients with small esophageal varices, prophylactic treatment with nonselective beta blockers (eg, propranolol, nadolol) is recommended to reduce the progression to large varices and the risk of variceal hemorrhage. Endoscopic variceal ligation can be used as an alternate primary preventive therapy in patients with contraindications to beta blocker therapy.

6. A 10-month-old boy is brought in for follow-up on anemia. He was previously drinking whole cow's milk, and screening hemoglobin showed 10.6 g/dL at age 9 months. The patient has since been drinking ironfortified formula and eating pureed spinach and meat daily. He is also taking ferrous sulfate supplementation each day. Growth and physical examination are normal. Current hemoglobin is 12 g/dL. The parents ask if iron therapy should be continued. Which of the following is the most appropriate response? A. Iron therapy can be discontinued as the patient is eating iron-rich foods B. Iron therapy should be continued for a few more months C. Management depends on the patient's current reticulocyte count D. Management depends on the patient's current serum ferritin level E. The patient was likely misdiagnosed with anemia as his hemoglobin level is currently normal

Answer B Educational objective: Inadequate ingestion of iron-rich foods is an important risk factor for iron deficiency anemia in infants. Oral iron therapy should be administered until 2-3 months after reaching the age-appropriate hemoglobin level to ensure that iron stores are replete.

37. A 2-year-old boy is brought to the emergency department with vomiting and episodes of crying. The patient started crying 4 hours ago and was inconsolable for 20 minutes. Afterward, he was happy and playful but began crying again an hour later and then had 6 episodes of emesis, which became bilious. On examination, he appears tired but not ill. His temperature is 37.2 C (99 F), blood pressure is 92/46 mm Hg, and pulse is 156/min. Physical examination is normal. A peripheral intravenous line is placed and the boy is given a 20 mL/kg normal saline bolus. Which of the following is the most appropriate next step in management of this child? A. Abdominal radiograph B. Abdominal ultrasound C. Emergency surgical consultation D. Hydration and reassurance E. Upper gastrointestinal series

Answer B Educational objective: Intussusception is most common in infants age 6-36 months and typically presents with episodes of crampy abdominal pain and vomiting. A "target sign" is classically seen on abdominal ultrasound, the diagnostic test of choice. First-line treatment consists of a water-soluble or air-contrast enema.

66. A 72-year-old man with end-stage renal disease comes to the emergency department with moderate abdominal pain and bloody stools following hemodialysis. He has no vomiting, back pain, or urinary symptoms. His temperature is 37.4 C (99 F), blood pressure is 110/80 mm Hg, pulse is 88/min and regular, and respirations are 16/min. The abdomen is soft but tender in the periumbilical and left lower quadrant areas. Bowel sounds are present. There is no rebound tenderness or rigidity. Rectal examination shows stool mixed with bright red blood. His hematocrit is 30% (unchanged from prior) and white blood cell count is 11,000/µL.Which of the following is the best next step in management of this patient? A. Colonoscopy B. Contrast-enhanced CT scan of the abdomen C. Magnetic resonance angiography D. Stool testing for Clostridium difficile toxin E. Upper gastrointestinal endoscopy

Answer B Educational objective: Ischemic colitis presents with abrupt-onset abdominal pain and hematochezia. CT scan is performed initially to rule out extensive bowel injury or perforation. Diagnosis is confirmed with colonoscopy. Treatment is supportive, with surgery considered for patients with clinical deterioration or bowel infarction.

23. A 26-year-old Asian woman with no significant past medical history comes to the physician due to frequent bloating, flatulence, and frothy diarrhea exacerbated by foods such as milk and ice cream. Her older sister was diagnosed with ulcerative colitis. Physical examination and basic laboratory testing are unremarkable. Which of the following is most likely to establish this patient's diagnosis? A. Anti-tissue transglutaminase antibodies B. Breath hydrogen test C. Colonoscopy D. Duodenal aspirate E. Stool testing for bacterial antigen

Answer B Educational objective: Lactose malabsorption typically presents with gastrointestinal symptoms (eg, bloating, flatulence, frothy diarrhea) after dairy intake and has a high prevalence in Eastern Asian populations. The lactose breath hydrogen test is highly sensitive and specific and can confirm the diagnosis.

33. An asymptomatic 40-year-old woman comes to the office for a routine health maintenance examination. The patient has a history of non-Hodgkin lymphoma diagnosed at age 30, for which she received combination chemotherapy and radiation therapy to the chest. Follow-up positron emission tomography scans showed no relapse of the disease. She does not use tobacco, alcohol, or illicit drugs. There is no family history of cancer. Physical examination is unremarkable. Which of the following is the most appropriate next step in management of this patient? A. Annual chest x-ray B. Annual mammogram C. Colonoscopy D. Exercise stress testing E. Pulmonary function tests

Answer B Educational objective: Patients who received chemotherapy for non-Hodgkin lymphoma are at risk for long-term complications, including secondary malignancy and cardiovascular effects. Therefore, they should receive annual screening mammograms (and possible breast MRI examinations) beginning 8 years after therapy but no later than age 40.

8. A 2-month-old girl is brought to the office for a routine well visit. She was diagnosed with sickle cell disease at birth. Currently she is doing well and is without new concerns. Hemoglobin is 11 g/dL. Which of the following is the best next step in management of this patient? A. Begin iron supplementation B. Begin penicillin prophylaxis C.Delay vaccinations until age 6 months D. Obtain a right upper quadrant ultrasound E. Obtain an echocardiogram

Answer B Educational objective: Patients with sickle cell disease are at increased risk of infection from encapsulated organisms due to splenic dysfunction (functional asplenia). Penicillin prophylaxis should be initiated at age 2 months and continued until at least age 5 years.

27. A 61-year-old man is evaluated for recurrent episodes of blurry vision. He has no significant past medical history and is an active smoker. His blood pressure is 155/90 mm Hg and pulse is 78/min. His oxygen saturation is 95% on room air, and his BMI is 31 kg/m 2 . The spleen is palpable on inspiration. Laboratory results are as follows: Hematocrit 64% Platelets 520,000/mm Leukocytes 9,800/mm 3 Sodium 140 mEq/L Creatinine 0.8 mg/dL Total bilirubin 1.1 mg/dL Which of the following is the most likely additional finding in this patient? A. Hematuria B. Low serum erythropoietin C. Philadelphia chromosome D. Prolonged nocturnal desaturations E. Vitamin B12 deficiency

Answer B Educational objective: Polycythemia vera is a myeloproliferative disorder due to erythropoietin-independent overproduction of red blood cells. Diagnosis is confirmed via JAK2 mutation and low erythropoietin levels. Patients at low risk for thrombosis can be managed with aspirin, phlebotomy, and symptomatic treatment.

57. A 40-year-old man with alcoholic cirrhosis is brought to the emergency department (ED) with bloody vomiting. On initial evaluation, he is obtunded and arousable to vigorous stimuli only. The patient's blood pressure is 81/45 mm Hg and heart rate is 121/min. Dried blood is seen around the mouth. The abdomen is distended with a positive fluid shift. Two large-bore intravenous lines are placed, and 2 liters of normal saline are given wide open. When in the ED, he has another episode of hematemesis with clots.Which of the following is the most appropriate next step in management of this patient? A. Diagnostic paracentesis B. Endotracheal intubation C. Nasogastric tube placement D. Octreotide infusion E. Upper gastrointestinal endoscopy

Answer B Educational objective: Unstable patients with liver cirrhosis and variceal hemorrhage should initially be managed with airway protection and hemodynamic resuscitation. Once stabilized, patients can benefit from an octreotide infusion, prophylactic antibiotics, and urgent upper endoscopy.

7. A 2-year-old girl comes to the physician with an intensely pruritic rash that erupted shortly after eating a peanut butter cookie. She has no prior history of rash and has no other symptoms. The patient's mother has a history of anaphylaxis to shellfish. The girl's temperature is 37 C (98.6 F), blood pressure is 108/60 mm Hg, pulse is 78/min, and respirations are 16/min. Examination shows a raised rash on her back. The rest of the examination is normal. Which of the following is the best next step in the management of this patient? A. Intramuscular epinephrine B. Oral diphenhydramine C. Oral loratadine D. Skin prick test E. Topical hydrocortisone

Answer C Educational objective: Acute urticaria (hives) is a well-demarcated, pruritic, papular, polymorphic, and transient lesion that may appear in isolation or in groups on any part of the body. Possible etiologies include infection, medication, and food. A second-generation antihistamine (eg, cetirizine, loratadine, fexofenadine) is the preferred treatment for symptomatic relief.

10. A 50-year-old man comes to the emergency department due to difficulty breathing over the last hour. He also has periorbital, circumoral, and facial edema. He has no itching or skin rash. The patient has a recent history of ST elevation myocardial infarction that required cardiac catheterization and coronary artery stenting. In the emergency department, his temperature is 36.8 C (98.4 F), blood pressure is 110/70 mm Hg, pulse is 78/min, and respirations are 24/min. Which of the following medications is most likely responsible for this patient's present symptoms? A. Aspirin B. Clopidogrel C. Lisinopril D. Metoprolol E. Simvastatin

Answer C Educational objective: Angiotensin-converting enzyme (ACE) inhibitors are the most common cause of acquired angioedema. Angioedema due to ACE inhibitors usually begins within the first days to weeks of therapy but may occur at any time.

38. A 54-year-old woman comes to the office for a routine health maintenance examination. Menopause was at age 50 and she does not take hormone therapy. All previous Pap tests have been normal. Cervical cancer screening is performed and the results are as follows: Satisfactory for evaluation Endocervical/transformation zone component present Negative for intraepithelial lesion or malignancy Note: Benign-appearing endometrial cells present Which of the following is the most appropriate next step in management of this patient? A. Cervical conization B. Colposcopy C. Endometrial biopsy D. Repeat Pap test in 6 months E. Routine Pap testing only

Answer C Educational objective: Benign-appearing endometrial cells on Pap testing in high-risk premenopausal women age ≥45 (eg, chronic anovulation, obesity) and in all postmenopausal women requires an endometrial biopsy to evaluate for endometrial hyperplasia/cancer.

28. A 26-year-old woman comes to the office for medical evaluation. She has no specific problems but says she tires easily. She has heavy menses every month lasting 4-5 days. The patient was diagnosed with anemia 6 months ago and has been taking oral iron supplements since. She does not use tobacco, alcohol, or illicit drugs. Her physical examination findings are within normal limits. Laboratory studies are as follows: Complete blood count Hemoglobin 10.8 g/dL Platelets 210,000/mm Leukocytes 5,200/mm Hematology Mean corpuscular volume 64 µm3 Erythrocyte count 5.8 million/mm Red cell volume 14.2% (normal: distribution width 11.5%-14.5%) Ferritin, serum 324 ng/mL Which of the following is the best next step in management of this patient? A. Bone marrow biopsy B. Colonoscopy and esophagogastroduodenoscopy C. Hemoglobin electrophoresis D. Serum lactate dehydrogenase and haptoglobin E. Vitamin B 12 and folic acid levels

Answer C Educational objective: Beta thalassemia minor is usually clinically asymptomatic but presents with mild anemia (hemoglobin >10 g/dL), low mean corpuscular volume (<75 µm 3 ) disproportionate to the degree of anemia, mildly elevated ferritin, and normal red cell distribution width with a Mentzer index <13.

22. Which of the following chest pain descriptions by a patient should most raise concern for acute coronary syndrome? A. Constant pins and needles sensation B. Fleeting fluttering sensations C. Poorly localized, burning pain D. Sharp pain on deep inspiration E. Tearing pain radiating to the back

Answer C Educational objective: Chest pain due to myocardial ischemia is classically described as a heavy or squeezing sensation behind the sternum. However, other presentations may also represent acute myocardial ischemia, especially in women, older patients, and patients with diabetes mellitus. Alternate descriptions include burning sensation, vague discomfort, or a fullness or "lump" in the chest.

14. A 2-month-old girl is brought to the office for a well-child visit. She was born full-term, and weight is on the 50th percentile. She is breastfed exclusively and takes vitamin D daily. The girl can lift her head and chest when prone but is unable to roll over. She is not yet reaching for toys, and her hands remain fisted about half the time. The girl startles in response to sudden loud noises, but she does not turn toward her parents when called by name. The patient recently started smiling in response to her parents' faces. She can track objects 180 degrees, but her parents note that her eyes sometimes appear unaligned. Which of the following is the most appropriate management of this patient's development? A. Obtain brain MRI B. Perform audiologic assessment C. Provide reassurance D. Refer for ophthalmologic evaluation E. Refer to physical therapy

Answer C Educational objective: Developmental milestones should be assessed at every well-child visit. Infants age 2 months should lift the head and chest when prone, alert to loud sounds, track past midline, and have a social smile.

3. A 72-year-old man comes to the emergency department with intermittent substernal chest pain on minimal exertion over the last 2 days that lasts 10-15 minutes each time. He has no shortness of breath, palpitations, syncope, or diaphoresis. He took 2 low-dose aspirins at home prior to arrival. The patient's past medical history is significant for hypertension and diet-controlled type 2 diabetes mellitus. He is a former smoker with a 40-pack-year history. His blood pressure is 154/86 mm Hg and pulse is 77/min. Cardiac examination is within normal limits. Electrocardiogram shows 1.5-mm horizontal ST-segment depression in leads V2-V6. Initial troponin I level is normal. He appears comfortable at rest and declines cardiac catheterization. Which of the following is the best initial treatment for this patient? A. Abciximab B. Cilostazol C. Clopidogrel D. Niacin E. Nifedipine

Answer C Educational objective: Dual antiplatelet therapy with aspirin and P2Y12 receptor blockers (eg, clopidogrel, prasugrel, or ticagrelor) is recommended in all patients with acute coronary syndrome regardless of treatment approach (conservative or invasive).

44. A 59-year-old woman comes to the office for a routine health maintenance visit. She has well-controlled hypertension and type 2 diabetes mellitus. The patient underwent menopause at age 52 and has had no postmenopausal bleeding. She has never taken hormone replacement therapy. Her mother was diagnosed with endometrial cancer at age 68. BMI is 25 kg/m 2 . Physical examination is unremarkable. The patient is concerned about her risk of endometrial cancer and asks about a risk reduction strategy. Which of the following is the most appropriate recommendation for this patient? A. Annual endometrial biopsies B. Annual pelvic examination and ultrasound C. No additional management indicated D. Progestin-releasing intrauterine device E. Prophylactic hysterectomy

Answer C Educational objective: Endometrial cancer typically presents with abnormal uterine bleeding, postmenopausal bleeding, or as an abnormal Pap test (eg, benign-appearing endometrial cells). Screening for endometrial cancer in asymptomatic, low- or average-risk women is not indicated due to the lack of evidence for reducing patient mortality.

11. A 9-month-old full-term boy is brought to the physician for a well-child visit. He is breastfeeding and enjoying pureed foods. However, he has no teeth and has not started finger foods. His mother is concerned that he is behind other children in his playgroup who are of similar age and have teeth. Review of systems is positive for significant drooling. The child's weight and height have been tracking along the 60th percentile for age and sex. Examination shows a well-appearing boy with no primary dentition. Which of the following is the most appropriate next step in management of this child? A. Chromosomal testing B. Initiation of calcium supplementation C. Reassurance and observation D. Referral to a dentist E. X-ray of the oral cavity

Answer C Educational objective: Eruption of the primary dentition generally begins with the mandibular central incisors at age 6-10 months. Children who have no primary teeth by age 16 months should be evaluated for genetic syndromes, malnutrition, or congenital missing teeth.

5. A 72-year-old man is scheduled for right knee replacement surgery due to severe osteoarthritis. He has no other medical problems. He takes ibuprofen and occasionally oxycodone for knee pain. He also takes herbal supplements that he purchases from a health food store. Which of the following herbal supplements increases the risk of bleeding in this patient? A. Echinacea B. Ephedra C. Ginkgo biloba D. Kava E. St John's wort

Answer C Educational objective: Herbal supplements are used frequently by the general population. Many such supplements (eg, ginkgo biloba, ginseng, and saw palmetto) increase risk of bleeding and should be discontinued 14 days prior to surgical procedures.

5. A 22-year-old woman comes to the office for evaluation of genital lesions. The patient first noticed the lesions 2 days ago after returning home from a Caribbean vacation. The lesions have become increasingly painful, and the pain has not improved with cold compresses or ibuprofen. She is currently sexually active with multiple partners and uses a progestin-containing intrauterine device. Vital signs are normal. Pelvic examination reveals lesions similar to the ones in the exhibit. There are multiple tender inguinal lymph nodes. Which of the following is the most likely diagnosis in this patient? A. Chancroid B. Granuloma inguinale C. Herpes simplex virus D. Lymphogranuloma venereum E. Primary syphilis

Answer C Educational objective: Herpes simplex virus (types 1 and 2) infection can present with multiple clusters of painful vesicles and ulcers, as well as tender inguinal lymphadenopathy. Diagnosis is clinically confirmed with viral DNA PCR from ulcer scrapings. Management includes a course of oral antiviral therapy (eg, valacyclovir) for 7-10 days.

12. A 56-year-old man comes to the office due to a 3-week history of hoarseness. The patient is a school teacher and has never had similar symptoms during his 20-year career. He has had no recent upper respiratory illness, cough, shortness of breath, or difficulty swallowing but has been having vague right ear discomfort. The patient takes lisinopril for hypertension and over-the-counter antacids for occasional heartburn. He has smoked a pack of cigarettes daily for 30 years but does not use alcohol or illicit drugs. Blood pressure is 122/72 mm Hg, pulse is 78/min, and BMI is 28 kg/m 2 . His voice sounds hoarse. On examination, there is no nasal discharge, the tympanic membranes appear normal, and the oropharyngeal mucous membranes are pink with no abnormal lesions. The remainder of the physical examination is normal. Which of the following is the most appropriate next step in management of this patient's symptoms? A. Antireflux therapy with proton pump inhibitor B. Change in antihypertensive therapy C. Complete otolaryngologic evaluation D. Smoking cessation and voice therapy E. Steam inhalation and voice rest

Answer C Educational objective: Hoarseness is characterized by coarsening of vocal quality and is typically due to abnormalities in the larynx (eg, edema, hemorrhage, laceration, mass lesions). Most causes of acute hoarseness are benign. However, patients with symptoms lasting >2 weeks or with a history of tobacco or heavy alcohol use, systemic symptoms, stridor, dysphagia, or referred ear pain require referral for consideration of laryngoscopy.

4. A 32-year-old woman comes to the physician due to headache and jaw pain. She has had 2 weeks of dull, achy pain at the left temple that fluctuates in severity but never completely resolves. The pain radiates to the left ear and is worse with chewing. On examination, the patient has pain and crepitus with opening of the jaw and tenderness over the left masseter muscle. Which of the following is the most appropriate medication for this patient's symptoms? A. Fluoxetine B. Hydrocodone C. Naproxen D. Nortriptyline E. Prednisone

Answer C Educational objective: Initial management of temporomandibular joint dysfunction includes soft diet, warm compresses, and passive stretching. Nonsteroidal anti-inflammatory drugs are the preferred medication.

5. A double-blinded randomized controlled trial assigned patients to a treatment arm with a new antihypertensive agent or a control arm with an existing antihypertensive agent. Researchers performed intention-to-treat analysis by calculating patient outcomes based on original group assignments, regardless of adherence to the regimen or withdrawal from the study. This type of analysis typically leads to: A.A larger drop-out rate among the participants B. A larger percentage of excluded patients in the treatment group C. A more conservative estimate of the effect D. A more statistically significant result

Answer C Educational objective: Intention-to-treat analysis analyzes each participant based on randomization group (even if the participant stops the treatment or takes a different treatment). This tends to provide a conservative estimate of treatment effect.

10. A 2-year-old girl is brought to the emergency department with lethargy after playing with her grandfather's open pillbox. Her temperature is 36.7 C (98.1 F), blood pressure is 70/36 mm Hg, pulse is 35/min, and respirations are 18/min. Pulse oximetry shows 98% on room air. The patient is sleepy but arousable. An electrocardiogram (ECG) shows sinus bradycardia with a PR interval of 220 msec. Fingerstick blood glucose is 46 mg/dL. In addition to intravenous fluids, which of the following is the most appropriate next step in management of this patient? A. Antibody administration B. Benzodiazepine administration C. Glucagon administration D. Octreotide administration E. Urine alkalinization

Answer C Educational objective: Intravenous glucagon and dextrose are the first-line treatment for beta blocker-induced hypoglycemia. Isotonic fluid boluses should be given for hypotension, and atropine should be given for symptomatic bradycardia.

7. A 5-week-old boy is brought to the emergency department due to 8 hours of vomiting and refusal to breastfeed. The emesis was initially the color of breast milk but has become green. According to the parents, the infant was breastfeeding well before today. The boy has no fever or diarrhea. His temperature is 37.2 C (99 F), blood pressure is 80/42 mm Hg, and pulse is 162/min. On examination, he appears lethargic with a sunken fontanelle and dry mucous membranes. The abdomen is distended with absent bowel sounds and diffuse tenderness to palpation. No masses are noted.Which of the following is the most likely diagnosis in this infant? A. Duodenal atresia B. Intussusception C. Malrotation with volvulus D. Necrotizing enterocolitis E. Pyloric stenosis

Answer C Educational objective: Malrotation with midgut volvulus is a surgical emergency that typically presents with bilious emesis and abdominal distension in the newborn period. Upper gastrointestinal series is diagnostic, but infants who are unstable or in whom volvulus is highly suspected should undergo immediate laparotomy.

4. A 3-year-old girl is brought to the emergency department with a 2-day history of nonbilious vomiting and diarrhea. She is fussy but consolable and her diapers are slightly less wet today. According to her parents, a "stomach bug" has been going around at the girl's day care center. Her temperature is 38 C (100.4 F), blood pressure is 90/60 mm Hg, heart rate is 110/min, and respirations are 20/min. The child has lost approximately 3% of her weight compared to a well-child visit last week. Her distal capillary refill time is approximately 2-3 seconds. The rest of the examination is unremarkable. Which of the following is the best initial fluid choice for rehydration of this child? A. Intravenous 0.9% normal saline B. Intravenous D5W with 0.45% normal saline C. Small, frequent sips of oral rehydration solution D. Small, frequent sips of water E. Small, frequent sips of juice

Answer C Educational objective: Mild to moderate dehydration from gastroenteritis should be managed by oral rehydration therapy on an outpatient basis. Parenteral therapy is reserved for severe dehydration or those patients who fail oral rehydration solution.

42. A 30-year-old woman with no medical history comes to the physician for a preemployment physical examination. A 3/6 late systolic murmur is heard at the left lower sternal border and apex when in the supine position, and the murmur gets longer with standing. There is a normal splitting of the second heart sound. The apical impulse is not displaced. The remainder of the physical examination is unremarkable. Which of the following is the most likely diagnosis? A. Atrial septal defect, secundum type B. Bicuspid aortic valve C.Mitral valve prolapse D. Pulmonic stenosis E. Tricuspid stenosis

Answer C Educational objective: Mitral valve prolapse is associated with single or multiple nonejection clicks and/or mid-to-late systolic murmur of mitral regurgitation. Abrupt standing or Valsalva maneuver decrease venous return and cause an earlier systolic click and longer murmur. Squatting or leg elevation causes a delayed systolic click and shorter murmur.

16. A 23-year-old woman comes to the emergency department after an episode of dizziness. The patient was in the crowded subway when she felt nauseated and dizzy but did not lose consciousness. She still feels mild nausea and has profound fatigue. The patient has a history of a "heart murmur" as a child. She recently returned from a camping trip in New Hampshire. Her electrocardiogram is shown in this exhibit. Which of the following is the most likely diagnosis? A. Atrial fibrillation B. Atrial premature beats C. Atrioventricular conduction block D. Sinus arrhythmia E. Wolff-Parkinson-White syndrome

Answer C Educational objective: Mobitz type 1 atrioventricular block is characterized by progressive prolongation of PR interval followed by a "dropped" or absent QRS complex (non-conducted P wave). Patients may be asymptomatic, but some patients have fatigue, lightheadedness, syncope, angina, or heart failure.

14. A 38-year-old man comes to the emergency department with a one-day history of left-sided chest and neck pain. The pain was worse when lying in a supine position and he eventually fell asleep in a semirecumbent position. He also had a sore throat a week ago but has otherwise felt well. The patient's temperature is 37.2 C (99 F), blood pressure is 132/80 mm Hg, and pulse is 90/min. Heart sounds are normal and lungs are clear to auscultation. There are no skin rashes.Laboratory results are as follows: Leukocytes 11,000/µL , Creatinine 0.8 mg/dL Troponin T, 0.09 ng/mL (normal <0.01 serum ng/mL) Erythrocyte sedimentation rate 52 mm/hr, Electrocardiogram shows sinus rhythm with 1-2 mm diffuse ST-segment elevation, except for a 2-mm STsegment depression in lead aVR. Chest x-ray reveals normal cardiac size and clear lung fields. Which of the following is the best initial therapy for this patient? A. Acetaminophen plus oxycodone B. Aspirin plus clopidogrel C. Ibuprofen plus colchicine D. Naproxen E. Prednisone

Answer C Educational objective: Most patients with acute viral myopericarditis should be treated with a combination of nonsteroidal antiinflammatory drugs (NSAIDs) and colchicine. Glucocorticoids are used in a minority of patients who have a contraindication to NSAIDs or have persistent symptoms that are refractory to standard therapy.

35. A 2-week-old boy is brought to the office for a routine examination. He was born at term after an uncomplicated pregnancy. Birth weight was 3.2 kg (7 lb) and current weight is 3.3 kg (7 lb 4 oz). He has been breastfed exclusively every 3 hours and has no symptoms. His blood pressure is 72/50 mm Hg, pulse is 110/min, and respirations are 32/min. Pulse oximetry is 98% on room air. Auscultation reveals a 3/6 harsh, blowing holosystolic murmur that is best heard at the left mid-sternal border. The rest of the examination is normal. Which of the following is the most likely cause of this patient's murmur? A. Large ventricular septal defect B.Peripheral pulmonary stenosis C.Small ventricular septal defect D.Still's murmur E. Venous hum

Answer C Educational objective: Most patients with ventricular septal defect (VSD) present in the neonatal period after pulmonary vascular resistance has declined. The clinical presentation depends on the size of the defect, ranging from an isolated, asymptomatic holosystolic murmur (small VSD) to heart failure (large VSD).

24. A 32-year-old woman comes to the office due to 2 months of vaginal irritation and pain with intercourse. Three months ago, the patient had an operative vaginal delivery complicated by a third-degree perineal laceration. She has breastfed her infant exclusively and has not resumed menses. Pelvic examination reveals a well-healed perineum with no point tenderness. There is no tenderness with speculum insertion; the vaginal mucosa is thin and pale. Which of the following is the most likely etiology of this patient's symptoms? A. Candida albicans infection B. Ectopic endometrial glands and stroma C. Low estrogen levels D. Persistent inflamed granulation tissue E. Pudendal nerve entrapment

Answer C Educational objective: Patients who are breastfeeding can have vulvovaginal atrophy (eg, vaginal irritation, dyspareunia, thin vaginal mucosa) due to low estrogen levels associated with lactation. Treatment is with lubricants and vaginal estrogen.

4. An 84-year-old man comes to the emergency department due to a progressively worsening throbbing headache, nausea, and dizziness over the past week. His past medical history is significant for chronic obstructive pulmonary disease, knee osteoarthritis, and type 2 diabetes mellitus. The patient's medications include tiotropium, albuterol as needed, acetaminophen, and metformin. He smoked 1 pack of cigarettes daily for 25 years but quit 15 years ago. The patient lives alone without any social support and has been experiencing financial difficulties. Given the frigid outdoor temperatures, he has stayed indoors at home for the past few weeks without any sick contacts. His temperature is 36.7 C (98 F), blood pressure is 144/78 mm Hg, and heart rate is 92/min and regular. Oxygen saturation is 95%. He is alert and oriented to self only. The remainder of his physical examination is normal. His fingerstick glucose level is 160 mg/dL. Which of the following is the most likely diagnosis? A. Acute alcohol toxicity B. Carbon dioxide retention C. Carbon monoxide poisoning D. Methemoglobinemia E. Viral meningitis

Answer C Educational objective: Patients with carbon monoxide poisoning may present with headache (most common) and other nonspecific symptoms (eg, altered mental status, malaise, nausea, dizziness). This often occurs during cold winter climates in the setting of smoke inhalation or with use of defective heating systems or fuelburning appliances in poorly ventilated areas. Diagnosis is confirmed by checking the carboxyhemoglobin level.

29. A 23-year-old woman comes to the office due to pelvic pain for the past week. The pain is worse with sexual intercourse and she has some associated postcoital spotting. The patient has had a copper intrauterine device (IUD) in place for 2 years. She has had 3 sexual partners in the last 6 months. Vital signs are normal. Speculum examination reveals a purulent endocervical discharge; IUD strings are visualized at the cervical os. Bimanual examination reveals cervical motion tenderness but no adnexal masses. Which of the following is the best next step in management of this patient? A. Admit for inpatient management and keep IUD in situ B. Await nucleic acid amplification testing to determine need for IUD removal C. Begin empiric antibiotics and keep the IUD in situ D. Order pelvic ultrasound and remove the IUD E. Remove the IUD and perform dilation and curettage

Answer C Educational objective: Pelvic inflammatory disease is an acute upper genital tract infection that is treated with empiric antibiotics. Intrauterine devices can remain in situ as there is no difference in treatment outcomes with or without removal.

30. A 49-year-old woman comes to the physician with persistent swelling and heaviness in her right lower extremity. The symptoms are worse at the end of the day. She was diagnosed with right femoral vein thrombosis 4 months ago and has been on therapeutic anticoagulation with warfarin since then. The right leg is edematous below the knee, with scaly erythematous patches on the inner surface. A small clean ulcer is noted just behind the medial malleolus. Her INR is 2.4. What is the most appropriate next step in management of this patient? A. Ankle-brachial index B.Cilostazol C.Leg elevation and compression stockings D.Subcutaneous enoxaparin E. Transthoracic echocardiogram

Answer C Educational objective: Post-thrombotic syndrome is an important complication of deep venous thrombosis (DVT) of the lower extremity. Patients with a history of DVT may develop pain, edema, skin hyperpigmentation, and venous dilation and/or ulcers. Management includes compression therapy and leg elevation.

13. A 37-year-old woman comes to the physician because of persistent nasal congestion and runny nose. She states that she developed nasal congestion, a productive cough, and myalgias 4 weeks ago. She has been self-medicating with over-the counter nasal decongestant spray and antitussive liquid for the last 2 weeks, with gradual resolution of her cough and myalgias. However, she has developed recurrent nasal congestion and runny nose that is unresponsive to treatment. Her nasal discharge is clear. She has no associated sneezing, facial pain, or ocular itching. Her overall health is good and she denies similar symptoms in the past. On physical examination, her nasal mucosa appears edematous and red. The remainder of her physical examination is within normal limits. Which of the following is the best next step in management? A. Intranasal fluticasone B.Ipratropium nasal spray C. Nasal endoscopy D. Regular nasal saline lavage E. X-ray of the sinuses

Answer C Educational objective: Rhinitis medicamentosa usually occurs in patients with chronic use of vasoconstrictor nasal sprays. Symptoms are due to a cycle of rebound exacerbation with worsening nasal congestion and discharge. Treatment involves stopping the decongestant spray and initiating a trial of intranasal steroids (e.g., fluticasone).

4. A 39-year-old woman comes to the emergency department due to mild shortness of breath and right-sided pleuritic chest pain that began this morning. The patient underwent a mastectomy for left-sided breast cancer 2 weeks ago. She has a history of hives after eating seafood.Blood pressure is 148/96 mm Hg, pulse is 114/min, and respirations are 21/min. Oxygen saturation is 91% on room air.Chest x-ray is unremarkable and urine pregnancy test is negative. Creatinine is 0.8 mg/dL. Which of the following is the most appropriate diagnostic approach? A. CT pulmonary angiography after pretreatment with glucocorticoids and antihistamines B. CT pulmonary angiography using a high-osmolar radiocontrast material C. CT pulmonary angiography without specific precaution D. Lower extremity compression ultrasonography E.Ventilation-perfusion scan

Answer C Educational objective: Seafood allergy does not confer any increased risk of anaphylaxis from iodinated contrast media and does not necessitate pretreatment with glucocorticoids and antihistamines.

35. A 24-year-old woman comes to the clinic for a routine health maintenance examination. The patient has no concerns. She has no chronic medical conditions and takes no medications other than a daily multivitamin. The patient has been sexually active with a female partner for the past 8 months. She has had only female partners and has had 10 lifetime partners. HIV screening was negative at her annual examination last year. She does not use tobacco, alcohol, or illicit drugs. Vital signs are normal. Physical examination is unremarkable. This patient requires screening for which of the following sexually transmitted infections (STIs)? A. Hepatitis B B. Herpes simplex virus C. Neisseria gonorrhoeae D. No STI screening indicated E. Treponema pallidum

Answer C Educational objective: Sexual minority women (eg, lesbian, gay, bisexual, transgender) may be at increased risk for sexually transmitted infection. All sexually active women age <25 require Chlamydia trachomatis and Neisseria gonorrhoeae screening annually, as do women with multiple or new sex partners, regardless of sexual orientation.

6. A 3-year-old girl is brought to the emergency department after ingesting an unknown tablet. She was playing on the kitchen floor at her grandfather's house when her mother saw her put a tablet in her mouth. Her parents cannot remember what medications the grandfather takes, but he has a history of hypertension, hyperlipidemia, and type II diabetes mellitus. The child's temperature is 36 C (96.8 F), blood pressure is 88/46 mm Hg, pulse is 120/min, and respirations are 28/min. She is pale, lethargic, and difficult to arouse. Serum glucose level is 25mg/dL. Ingestion of which of the following is the most likely etiology of this patient's symptoms? A. Atenolol B. Clonidine C. Glipizide D. Metformin E. Nifedipine

Answer C Educational objective: Sulfonylureas are oral hypoglycemic agents that can be lethal to children in very small doses. Symptoms (eg, irritability, lethargy, diaphoresis) are primarily due to hypoglycemia. Treatment consists of activated charcoal within 2 hours, intravenous dextrose, and octreotide for persistent hypoglycemia.

38. A 65-year-old woman comes to the office for a preoperative evaluation. She is scheduled to undergo a total hip arthroplasty for osteoarthritis in 6 weeks. The patient walks slowly and uses a cane due to hip pain. She has no chest pain or shortness of breath. Her other medical problems include atypical ductal hyperplasia of the breast, gastroesophageal reflux disease, hypertension, and hyperlipidemia. Her mother was diagnosed with breast cancer at age 51, and her father had a myocardial infarction at age 62. She is a lifetime nonsmoker. Medications include amlodipine, naproxen, esomeprazole, rosuvastatin, and tamoxifen. Vital signs are within normal limits. Chest and abdominal examination is unremarkable. The patient asks whether any medication changes are necessary prior to the scheduled surgery. In addition to holding nonsteroidal anti-inflammatory medications 3 days prior to surgery, which of the following is the most appropriate course of action? A. Hold esomeprazole 3 days prior to surgery B. Hold rosuvastatin 3 days prior to surgery C. Hold tamoxifen 2-4 weeks prior to surgery D. Initiate low-dose metoprolol E. No additional intervention

Answer C Educational objective: Tamoxifen is associated with an elevated risk of venous thromboembolism (VTE). Patients who take tamoxifen for primary prevention of breast cancer should discontinue the medication prior to any surgery associated with a moderate or high risk of VTE (eg, hip arthroplasty).

6. An 81-year-old man with multiple comorbidities comes to the emergency department with lower abdominal pain. Initial evaluation reveals a distended bladder due to urinary retention. Which of the following home medications is most likely contributing to this patient's condition? A. Celecoxib B.Digoxin C. Diphenhydramine D. Warfarin E. Zolpidem

Answer C Educational objective: The Beers criteria are a comprehensive list of medications that should be used with caution or avoided in the elderly to help prevent adverse drug effects. First-generation antihistamines such as diphenhydramine have strong anticholinergic activity and can result in memory impairment, delirium, blurred vision, orthostatic hypotension, constipation, and urinary retention.

18. A fibrin degradation product (FDP) test using a venous blood sample is evaluated in patients with suspected pulmonary embolism. The diagnosis is confirmed by CT angiography (gold standard). The study reported that the test had an overall sensitivity of 95% and specificity of 70%. The prevalence of pulmonary embolism in the studied population was 15% (low risk patients). If the study is repeated in high-risk patients using the same gold standard (and assuming the same performance of the gold standard), which of the following characteristics for the FDP test will be most different from the current results? A. Interobserver variation B. Negative likelihood ratio C. Negative predictive value D. Sensitivity E. True negative rate

Answer C Educational objective: The negative and positive predictive values of a diagnostic test depend on disease prevalence.

22. A case-control study analyzes the association between multiple sclerosis and previous infection with Epstein-Barr virus (EBV). Patients with multiple sclerosis and matched controls are assessed for prior EBV infection using serologic markers. Which of the following is the most appropriate measure of association that should be reported for this study? A. Correlation coefficient B. Incidence C. Odds ratio D. Prevalence E. Relative risk

Answer C Educational objective: The odds ratio, a measure of association commonly used in case-control studies, compares the odds of exposure among individuals with the disease (cases) to those among individuals without the disease (controls).

14. In a group of 5000 patients with peripheral artery disease, 10% are taking cilostazol and 90% are not. Among the 10% of patients on cilostazol, 50 experienced a certain adverse event. Among the 90% of patients not on cilostazol, that same adverse event was reported in 250 patients. What is the relative risk of the adverse event in patients taking cilostazol compared to those not taking cilostazol? A. 0.10 B. 0.56 C. 1.80 D. 1.89 E. 5.00

Answer C Educational objective: The relative risk is the risk in the exposed group divided by the risk in the unexposed group.

38. A 62-year-old man with no significant past medical history is brought to the emergency department due to shortness of breath that has been progressive over the last 2 weeks.The patient's temperature is 37.2 C (99 F), blood pressure is 122/72 mm Hg, pulse is 92/min, and respirations are 22/min. His pulse oximetry is 89% on room air. Jugular venous pressure is estimated at 14 cm H2 O. Lung examination reveals bibasilar crackles in the lower one-third of the chest. Cardiac examination shows an audible S3 over the cardiac apex.Initial laboratory results are as follows: Serum chemistry, Sodium 132 mEq/L,Potassium 3.6 mEq/L,Chloride 102 mEq/L,Bicarbonate 20 mEq/l,Blood urea nitrogen 26 mg/dL,Creatinine 1.2 mg/dL,Calcium 9.2 mg/dL,Glucose 162 mg/dL. Electrocardiogram (ECG) shows normal sinus rhythm at a rate of 94/min and left bundle branch block. Which of the following is the best initial therapy for this patient? A. Carvedilol B. Dobutamine C. Furosemide D. Lisinopril E. Nitroglycerin

Answer C Educational objective: Treatment of acute decompensated heart failure (ADHF) includes supplemental oxygen, intravenous diuretics, and sometimes vasodilators. Beta blockers, ACE inhibitors, and aldosterone antagonists are useful in chronic heart failure due to systolic dysfunction but may worsen heart failure symptoms or cause hemodynamic deterioration in ADHF.

25. A 45-year-old woman comes to the physician for follow-up of hypertension. When seen in the physician's office 2 weeks ago, her blood pressure was 165/95 mm Hg initially and 160/92 mm Hg after re-check. The patient had her blood pressure taken 3 times at a nearby pharmacy, which reported readings averaging 120/80 mm Hg. She takes no medications, has no complaints, and feels "pretty healthy." Her blood pressure during this current visit is 169/94 mm Hg initially and 158/94 mm Hg after 20 minutes. Her BMI is 28 kg/m 2 . The ocular fundi appear normal. Physical examination is unremarkable. Electrocardiogram (ECG) is normal. Which of the following is the best next step in management of this patient? A. Measure 24-hour urine metanephrines B. Obtain echocardiogram to evaluate for left ventricular hypertrophy C. Order 24-hour ambulatory blood pressure monitoring D. Schedule repeat office blood pressure checks E. Start low-dose thiazide diuretic

Answer C Educational objective: Twenty-four hour ambulatory blood pressure monitoring can evaluate for suspected "white-coat" hypertension in patients with significant discrepancies between in-office and out-of-office blood pressure readings.

28. A 22-year-old woman comes to the emergency department after a sexual assault a few hours ago. The patient is not on contraception. Menses occur monthly and her last menstrual period was 3 weeks ago. Physical examination shows bruising and genital trauma. The cervix is edematous and friable. Sexually transmitted infection testing and a forensic evaluation are completed. Urine pregnancy test is negative. The patient asks for emergency contraception. Which of the following is the most appropriate management option for this patient? A. Combination oral contraceptives B. Copper-containing intrauterine device C. Levonorgestrel pills D. Methotrexate injection E. Misoprostol pills

Answer C Educational objective: Women who are sexual assault victims are offered postexposure prophylaxis for sexually transmitted infection and emergency contraception. First-line emergency contraception is with levonorgestrel pills (ie, plan B) due to high efficacy rates and oral administration. Intrauterine device placement is contraindicated in patients who have acute cervicitis.

39. A 42-year-old nonsmoker comes to clinic due to daily heartburn for the past 3 months. He also has dry cough and wheezing. The patient has no swallowing difficulty, nausea, vomiting, or weight loss. He has tried over-the-counter calcium carbonate with no significant relief. His father suffered from colon cancer at age 68, and his mother died of breast cancer at age 72.BMI is 30 kg/m 2 , and physical examination is unremarkable. Complete blood count and renal function are within normal limits.Which of the following is the most appropriate next step in management of this patient? A. Barium swallow B. Metoclopramide C. Proton pump inhibitor D. Pulmonary function testing E. Upper gastrointestinal endoscopy

Answer C Educational objective: A trial of proton pump inhibitors is recommended in most patients with gastroesophageal reflux disease. Endoscopy should be considered in patients with alarm features (eg, age >50, dysphagia, weight loss, anemia, hematemesis/melena) or in those who fail empiric treatment.

25. A 46-year-old woman comes to the office with 2 months of swelling on the side of her face. She has a history of drug abuse and hepatitis C infection but recently tested negative for HIV. Physical examination shows painless, firm swelling of the right parotid gland. There is also mild right-sided facial paresis. Which of the following is the most likely diagnosis? A. Benign tumor B. Chronic sialadenitis C. Malignant tumor D. Sialolithiasis E. Sjögren's syndrome

Answer C Educational objective: Benign parotid tumors typically present as slow-growing, localized, and painless masses or swelling without other focal findings. The presence of facial nerve palsies or cervical lymphadenopathy is concerning for malignancy.

1. A 4-year-old boy is brought to the emergency department after ingesting several unknown tablets while playing without supervision in his grandmother's room. When his father checked on him, he saw partially dissolved tablets in the boy's mouth. The boy's grandmother has type 2 diabetes mellitus, hyperlipidemia, hypertension, and depression. His temperature is 36.7 C (98 F), blood pressure is 76/38 mm Hg, pulse is 65/min, and respirations are 16/min. Pulse oximetry shows 93% on room air. The boy is sleepy but arousable. Diffuse expiratory wheezes are heard on auscultation. Which of the following is this child at greatest risk for developing? A. Hyperkalemia B. Hypermagnesemia C. Hypoglycemia D. Metabolic acidosis E. Respiratory alkalosis

Answer C Educational objective: Beta blocker ingestions in children classically present with bradycardia, hypotension, and hypoglycemia. Other manifestations include bronchospasm, altered mental status, and seizures in severe cases.

44. A 24-year-old woman with no past medical history complains of recurrent and painful mouth ulcers over the last 6 months. She also reports frequent abdominal pain, loose stools, and unintentional weight loss. The patient is afebrile with a blood pressure of 118/69 mm Hg and pulse of 71/min. Physical examination shows mild abdominal tenderness, primarily in the lower abdomen, without guarding or rebound. There are several shallow ulcers on the buccal mucosa. A biopsy of one of these ulcers shows granulomatous inflammation. The patient's hematocrit is 42%. Which of the following is the most likely cause of this patient's complaints? A. Anorexia nervosa B. Celiac disease C. Crohn's disease D. Folic acid deficiency E. Hyperthyroidism

Answer C Educational objective: Crohn's disease (CD) can involve any component of the gastrointestinal tract from the mouth to the anus; non-contiguous areas of involvement are characteristic. Although nonspecific, aphthous ulcers can be seen in CD. Granulomas are identified pathologically in up to 30% of patients with CD.

18. A 43-year-old woman is evaluated for nausea, vomiting, and intermittent epigastric pain for the last 6 months. Her symptoms have gradually worsened, and she cannot finish a regular meal without feeling full quickly. Consequently, the patient has lost 6.8 kg (15 lb). She has a history of type 1 diabetes mellitus with diabetic nephropathy and neuropathy. Examination shows a distended and nontender abdomen with an obvious succussion splash. Rectal examination shows brown stool negative for occult blood. Serum creatinine is 1.4 mg/dL and hemoglobin A1c is 7.9%.Which of the following is the most appropriate next step in management? A. Intensify insulin regimen and re-evaluate in 4-6 weeks B. Order abdominal ultrasound C. Perform upper gastrointestinal endoscopy D. Schedule scintigraphic gastric emptying study E. Start metoclopramide therapy

Answer C Educational objective: Diabetic gastroparesis usually presents with early satiety, postprandial reflux, nausea and vomiting, and possible succussion splash on examination. All patients should first be evaluated for mechanical obstruction with an upper gastrointestinal endoscopy or barium test.

13. A 12-day-old girl is brought to the office for a well check by her parents. She was jaundiced in the nursery and this condition has persisted. Her parents report that she is breastfeeding well and produces a normal number of wet and dirty diapers. Her mother had appropriate prenatal care and the pregnancy and delivery were unremarkable. The patient's total bilirubin was 9 mg/dL at nursery discharge and 11.2 mg/dL at age 5 days; the newborn screen was normal. Her birth weight was 3.85 kg (8.48 lb) and she now weighs 4 kg (8.8 lb). Her vital signs are normal. She is diffusely jaundiced and has scleral icterus on examination. The liver edge is palpable at the right costal margin. The remainder of the examination is unremarkable. Liver function studies are as follows:Total Bilirubin 25.3 mg/dL,Direct Bilirubin 15.2 mg/dL,Alkaline phosphatase 520 U/L,Aspartate aminotransferase (SGOT) 47 U/L,Alanine aminotransferase (SGPT) 44 U/L,The most appropriate next step in the management of this infant is? A. Initiation of hydrolyzed formula B. Inpatient phototherapy C. Hepatobiliary ultrasound D. HIDA scan E. Home phototherapy

Answer C Educational objective: Direct hyperbilirubinemia (>2 mg/dL direct bilirubin or >20% of total bilirubin) is always pathologic. Causes include cholestasis or obstructive lesions. Initial evaluation should include liver function tests, newborn screening for metabolic diseases, and hepatobiliary ultrasound.

50. A 26-year-old man comes to the office for evaluation of difficulty swallowing. The patient reports an intermittent sensation of food getting stuck in his chest over the past several months. During dinner yesterday, a piece of steak got stuck in the lower retrosternal area but dislodged after he drank a carbonated beverage. The patient has also had occasional heartburn that does not improve with antacids or omeprazole. He has had no vomiting, hematemesis, melena, or weight loss. The patient has a history of asthma and uses an albuterol inhaler as needed. He takes no other medications and does not use tobacco, alcohol, or illicit drugs.Vital signs are normal. There are no oropharyngeal lesions or cervical lymphadenopathy. Lung auscultation reveals scattered wheezes. The abdomen is soft and nontender with no organomegaly. Stool is negative for occult blood.Complete blood count and comprehensive metabolic panel are within normal limits. Barium esophagram reveals no abnormalities.Which of the following is the most likely diagnosis in this patient? A. Achalasia B. Candida esophagitis C. Eosinophilic esophagitis D. Plummer-Vinson syndrome E. Reflux esophagitis

Answer C Educational objective: Eosinophilic esophagitis is characterized by heartburn and dysphagia that does not respond to antisecretory therapy. It is strongly associated with atopic disorders and food allergies. Endoscopy may show strictures, rings, longitudinal furrows, and white papules. Management includes dietary modification and topical/swallowed corticosteroids.

35. A 4-day-old girl is brought to the physician by her parents for a routine follow-up visit. The pregnancy and delivery were uncomplicated, and the girl and her mother were discharged from the hospital 2 days ago. The patient is breastfeeding exclusively, and her mother feels that her milk supply is increasing. The infant is latching well and has had no vomiting or spitting up. The infant's weight today is 3600 g (7 lb 8 oz), down from 4000 g (8 lb 13 oz) at delivery. Physical examination shows mild scleral icterus and jaundice to the chest but is otherwise normal.Which of the following is the best next step in management of this patient? A. Continue breastfeeding and begin formula supplementation, follow-up in 1-2 days B. Continue breastfeeding, follow-up at age 14 days C. Continue breastfeeding, follow-up in 1-2 days D. Discontinue breastfeeding, begin formula feeding, follow-up at age 14 days E. Intravenous fluids and hospitalization

Answer C Educational objective: Infants can lose up to 10%-12% of their birth weight in the first week of life. They should be evaluated for signs of dehydration or difficulty feeding. Well-appearing infants who have lost >7% of their birth weight should be followed closely. In general, infants should regain their birth weight by age 10-14 days.

32. A previously healthy 19-month-old boy is brought to the emergency department for abdominal pain. He started vomiting 12 hours earlier and has had 10 episodes of emesis. The emesis initially looked like food but has now become a greenish color. Before each episode, the boy would curl into the fetal position and start crying. Although he initially seemed fine after each episode, over the past 3-4 hours he has become increasingly lethargic. A small amount of blood and mucus was seen on his stool, but he has no fever. The boy takes no medications and has no known allergies. His temperature is 37.2 C (99 F), pulse is 132/min, respirations are 26/min, and blood pressure is 96/54 mm Hg. The child appears lethargic but is responsive to physical examination. The abdomen is soft, nontender, and nondistended; no masses are noted. A plain radiograph of the abdomen is shown below.Which of the following is the most likely diagnosis? A. Appendicitis B. Infectious colitis C. Intussusception D. Malrotation with volvulus E. Meckel's diverticulum

Answer C Educational objective: Intussusception classically presents with episodic, crampy abdominal pain and "currant jelly" stools. A "sausage-shaped" mass may be palpated in the right side of the abdomen. Diagnosis is supported by a "target sign" on abdominal ultrasound. Enema is the treatment of choice.

61. A 67-year-old man comes to the physician due to fatigue and low energy for the last 6 months. He has not had any nausea, vomiting, abdominal pain, bleeding, diarrhea, or black stools. He takes senna for occasional constipation. Physical examination is unremarkable.Laboratory results are as follows: Hemoglobin 9.4 g/dL Mean corpuscular volume 72 µm 3 Creatinine 0.8 mg/dLLiver function tests are normal. Serum ferritin level is low, and office-based fecal occult blood test is negative.Which of the following is the best next step in management of this patient? A. Abdominal ultrasound B. Anti-tissue transglutaminase antibodies C. Colonoscopy D. Serum protein electrophoresis E. Vitamin B12 and folate levels

Answer C Educational objective: Iron deficiency anemia in older patients is commonly due to gastrointestinal blood loss. Colonoscopy with subsequent endoscopy (if colonoscopy is negative) is preferred for excluding gastrointestinal sources of blood loss. Further tests are recommended for patients with negative endoscopy to exclude other causes of unexplained iron deficiency anemia.

34. A 48-year-old man comes to the office to discuss weight loss. For the past 6 months, he and his wife have been following a low-fat diet and exercising regularly. However, the patient has not achieved satisfactory weight loss. Medical history is unremarkable, but family history is notable for cardiovascular disease in both parents. The patient quit smoking 13 years ago and has a 10-pack-year history; he drinks alcohol only on social occasions.Blood pressure is 133/86 mm Hg and pulse is 78/min. BMI is 29 kg/m 2 . The patient appears moderately overweight but comfortable. Physical examination is otherwise normal.Laboratory results show fasting blood glucose 105 mg/dL, total cholesterol 210 mg/dL, HDL cholesterol 40 mg/dL, and triglycerides 185 mg/dL.The patient indicates that he and his wife intend to start a very low-carbohydrate diet. Compared to his current program, which of the following is this patient most likely to experience after changing his diet? A. Greater long-term weight loss B. Higher cardiovascular mortality C. Higher incidence of gastrointestinal symptoms D. Increased fasting glucose E. Less reduction in blood pressure

Answer C Educational objective: Low-carbohydrate diets can produce rapid initial weight loss. Compared with low-fat diets, lowcarbohydrate diets produce similar long-term weight loss and improvements in blood pressure and fasting blood glucose. Low-carbohydrate diets are associated with increased incidence of unpleasant gastrointestinal effects.

3. A 76-year-old man is evaluated for fatigue and found to have a monoclonal protein on serum electrophoresis (2.9 mg/dL) identified as IgG. Hemoglobin level is 14.2 g/dL, serum creatinine is 1 mg/dL, and serum calcium is 9.6 mg/dL. Urine protein electrophoresis shows no abnormalities. Which of the following is the most appropriate next step in evaluation of this patient? A. Assay for serum antinuclear antibodies B. Biopsy of abdominal fat pad C. Imaging for lytic bone lesions D. Serologic studies for Treponema pallidum E. Ultrasonography for liver abnormalities

Answer C Educational objective: Monoclonal gammopathy of undetermined significance (MGUS) is a premalignant plasma cell disorder characterized by serum monoclonal protein <3 g/dL, <10% clonal plasma cells on bone marrow biopsy, and no evidence of end-organ damage (eg, anemia, lytic bone lesions, hypercalcemia, renal insufficiency). Patients with suspected MGUS require cross-sectional imaging (eg, CT scan) to evaluate for lytic lesions, which would indicate multiple myeloma. Most also require bone marrow biopsy

31. A 55-year-old man comes to the physician with a 3-month history of gnawing upper abdominal pain. His appetite is decreased and he has had a 10 kg (22 lb) weight loss. He takes ibuprofen and methotrexate for rheumatoid arthritis and lisinopril for hypertension. He consumes 5-6 alcoholic beverages daily.Which of the following is the most likely cause of this patient's symptoms? A. Alcohol abuse B. Chronic infection C. Malignancy D. Medication side effect E. Vasculitis

Answer C Educational objective: Pancreatic cancer is often diagnosed at an advanced stage and has high mortality. Patients with suspected pancreatic cancer should have imaging with ultrasound or CT. Patients with characteristic findings on CT may proceed with surgical resection without confirmatory biopsy.

67. A 65-year-old man comes to the clinic for a screening colonoscopy. He has had no abdominal pain, blood in the stool, weight loss, or changes in bowel pattern. The patient underwent screening colonoscopy at age 60; at that time, 3 small adenomatous polyps were identified and removed. Medical history is notable for hypertension, gout, and osteoarthritis. The patient has no family history of colon cancer or advanced adenomas and does not use tobacco, alcohol, or illicit drugs.Colonoscopy is performed and reveals no abnormal lesions, but visualization of the colonic mucosa is incomplete due to poor bowel preparation.Which of the following is the most appropriate recommendation regarding colon cancer screening in this patient? A. Colonoscopy in 7 years B. Colonoscopy in 10 years C. Colonoscopy within a year D. CT colonography now E. Fecal immunochemical test now

Answer C Educational objective: Poor-quality bowel preparation may result in missing small polyps on surveillance colonoscopy; therefore, patients should be offered repeat colonoscopy within a year.

1.A study examines a new stress testing modality based on magnetic resonance perfusion imaging in diagnosing coronary artery disease (CAD). Coronary angiography is used as the gold standard. The following table is reported: Obstructive CAD on coronary angiography Results of magnetic Present. Absent resonance perfusion imaging Positive. 90. 10 Negative. 30. 70 Based on the study results, what is the sensitivity of the new test for detecting obstructive CAD? A. 60% B. 70% C. 75% D. 88% E. 90%

Answer C Educational objective: Sensitivity refers to the probability that a patient with the disease will have a positive test result. Sensitivity = True positives / (True positives + False negatives).

20. A 72-year-old man with persistent back pain is diagnosed with multiple myeloma. His monoclonal protein is identified as IgG. His hemoglobin level is 11.8 g/dL, serum creatinine is 0.9 mg/dL, and serum calcium level is 10.4 mg/dL. Which of the following is the strongest prognostic indicator for this patient? A. Erythrocyte sedimentation rate B. Haptoglobin level C. Serum albumin level D. Serum aspartate aminotransferase level E. Transferrin saturation

Answer C Educational objective: Staging and prognosis in multiple myeloma are based on serum b 2 microglobulin and albumin levels. Higher albumin levels generally correlate with a more favorable prognosis; elevated b 2 microglobulin levels are associated with greater tumor burden and renal failure.

A 78-year-old man is brought to the emergency department with abdominal pain. According to his wife, hehas had epigastric discomfort for the past 2 days; this morning he refused to get out of bed and appearedconfused. Past medical history is significant for diet-controlled diabetes mellitus, coronary artery diseasewith stable angina, hypertension, and gastroesophageal reflux disease.On examination, the patient is lethargic. His temperature is 38 C (100.4 F), blood pressure is 144/90 mmHg, pulse is 112/min and irregular, and respirations are 20/min. Chest is clear on auscultation. There areno heart murmurs. The abdomen is tender to palpation in the epigastric and right upper quadrant areas.His skin is warm and there are no rashes.Laboratory results are as follows: Hemoglobin 13.5 g/dLLeukocytes 11,000/mmNeutrophils 54%Bands 12%Creatinine 1.0 mg/dLTotal bilirubin 1.2 mg/dLAlkaline phosphatase 44 U/LAspartateaminotransferase(SGOT)22 U/LAlanineaminotransferase(SGPT)19 U/L ECG reveals atrial fibrillation with no significant ST segment or T wave changes. Abdominal ultrasoundshows several small gallstones without wall thickening or edema around the gallbladder. Common bileduct size is normal.Wh

Answer C Educational objective: The diagnosis of acute mesenteric ischemia depends on a high index of clinical suspicion in patients with known risk factors for thromboembolic disease (eg, atrial fibrillation, atherosclerosis). CT or MR angiography of the abdomen is typically used for diagnosis in stable patients.

26. A 39-year-old woman comes to the emergency department due to acute abdominal pain. She has severe but vague periumbilical pain that began spontaneously 6 hours earlier. The patient has no associated nausea, diarrhea, or dysuria. Her temperature is 37.8 C (100 F), blood pressure is 105/60 mm Hg, and pulse is 92/min. Abdominal examination shows marked diffuse tenderness but normal bowel sounds and no peritoneal signs. Leukocyte count is 12,500/mm 3 . The remainder of her laboratory studies, including serum chemistries, blood counts, β-hCG level, and urinalysis, are unremarkable.Which of the following is the most appropriate next step in evaluation of this patient's abdominal pain? A. Abdominal ultrasound B. Cholescintigraphy C. CT of the abdomen and pelvis D. MRI of the abdomen and pelvis E. Transvaginal ultrasound

Answer C Educational objective: The initial imaging test in undiagnosed abdominal pain depends on the location and most likely etiologies. Ultrasound is recommended for right upper quadrant pain. CT is preferred for lower abdominal, diffuse, or poorly localized pain.

25. A 13-year-old girl is brought to the physician for a routine examination. She has no complaints. She is performing well at school and is the captain of the soccer team. Her immunizations are up-to-date, and height and weight are appropriate for age. Family history is significant for osteoporosis in both grandmothers. At the patient's last office visit 3 months ago, increased daily dietary calcium intake was recommended. Current dietary intake includes one 8-oz glass of milk every morning and no fish or other dairy products. She occasionally eats dark-green leafy vegetables.What is the recommended total daily dietary intake of calcium for this patient? A. 700 mg B. 1000 mg C. 1300 mg D. 1800 mg E. 2000 mg

Answer C Educational objective: The recommended daily dietary allowance for calcium is highest during adolescence as it is the primary period of bone mineralization. Children age 9-18 should aim to consume 1300 mg of calcium daily.

64. A 64-year-old man is evaluated for melena over the last 2 days after taking naproxen for knee pain. He had coronary artery bypass grafting 5 years ago but has not had chest pain since then. His heart rate is 98/min with a blood pressure of 106/64 mm Hg supine and 100/72 mm Hg after 3 minutes of standing. The patient appears pale but is otherwise not in distress. Abdominal examination is significant for hyperactive bowel sounds and mild left upper quadrant tenderness. The stool appears melenic and fecal occult blood testing is positive. The patient's hemoglobin is 7.5 mg/dL, platelet count is 90,000/mm 3 , and creatinine is 1 mg/dL.In addition to establishing adequate venous access and starting intravenous crystalloids and pantoprazole, which of the following is the most appropriate management? A. Albumin infusion B. Intravenous octreotide C. Monitoring of hemoglobin only D. Packed red blood cell transfusion E. Platelet transfusion

Answer C Educational objective: The threshold for blood transfusion in most stable patients with upper gastrointestinal bleeding is hemoglobin <7 g/dL as this is associated with fewer complications and reduced mortality. Patients at risk for morbidity in the setting of severe anemia (eg, unstable coronary artery disease) may benefit from maintaining the hemoglobin level at >9 g/dL.

7. A 66-year-old man comes to the office due to 3 months of scrotal discomfort. He reports heaviness and a dull ache in the left scrotal area that is worse toward the end of the day. He has no fever, dysuria, hematuria, or penile discharge. The patient has a history of hypertension and takes amlodipine. He had a vasectomy 20 years ago. He lives with his wife and has 2 grown children. On examination, the abdomen is soft and nontender without organomegaly. Examination in the standing position shows a palpable, soft, irregular, nontender mass above the left testis that disappears in the supine position. There is no skin redness, groin bulge, or transillumination. Both testes are normal in size and without tenderness. Which of the following is the most appropriate next step in management of this patient? A. Annual scrotal ultrasound B. CT scan of abdomen and pelvis C. Scrotal support and analgesics D. Serial semen analysis E. Surgical referral

Answer C Educational objective: Varicoceles are due to tortuous dilation of the pampiniform plexus surrounding the spermatic cord. Asymptomatic patients do not require treatment; those with scrotal discomfort usually improve with analgesics (eg, nonsteroidal anti-inflammatory drugs) and/or scrotal support.

12. A research group is interested in studying a possible association between a commonly used medication X and the development of a rare condition. Patients newly diagnosed with the condition are enlisted and then matched with controls. The researchers administer a simple standardized questionnaire to both groups and ask questions related to medication X use over the past year. Which of the following is a limitation of this study design? A. External validity B. Lead-time bias C. Recall bias D. Reproducibility

Answer C Educational objective: Recall bias results from the inaccurate recall of past exposure by participants in a study. It applies mostly to case-control designs and leads to misclassification of exposure.

8. A 31-year-old farmer develops a severe reaction to a bee (Hymenoptera) sting with hypotension and respiratory distress. He is treated with epinephrine and corticosteroids and recovers rapidly. The patient has no prior similar incidents and no allergies. He is given an epinephrine autoinjector and instructed on its use. Which of the following interventions is also indicated in this patient? A. Antihistamine therapy B. Prolonged course of corticosteroids C. Referral for immunotherapy D. Serum total immunoglobulin E levels E. Serum tryptase level

Answer C Educational objective: Venom immunotherapy reduces the risk of recurrent anaphylaxis from 60% to 5%. It is indicated for patients with a history of systemic reaction to an insect sting and venom-specific immunoglobulin E.

3. In a study comparing treatment to placebo, the relative risk is given as 2.28 in favor of the treatment group with a 95% confidence interval of 1.32-3.96. Which of the following represents the best interpretation of the 95% confidence interval? A. About 95% of patients in the treatment group had a risk between 1.32 and 3.96 B. If the study were repeated 100 times, the confidence interval would be 1.32-3.96 in 95% of cases C. If the study were repeated 100 times, the relative risk would be 2.28 in 95% of cases D. If the study were repeated 100 times, the relative risk would be between 1.32 and 3.96 in 95% of cases E. The results are not statistically significant because the confidence interval does not cross 1.0

Answer D Educational objective: A confidence interval means that if a study were repeated 100 times, the results would fall within that range in 95% of cases

33. A 3-day-old girl is in the newborn nursery with blood in her diaper, which was noticed by the nurse. She was born at 37 weeks gestation by cesarean section due to preeclampsia. Apgar scores were 8 and 9 at 1 and 5 minutes, respectively. She has been breastfeeding well, passed meconium during the first 2 days of life, and has started having yellow-seedy stool today. The parents have refused intramuscular vitamin K and erythromycin ointment. Vital signs are normal. Examination shows an alert infant with normal external female genitalia. Scant blood is seen in the vagina. No other abnormalities are seen. Which of the following is the most likely etiology of this infant's symptoms? A. Ingested blood from delivery B. Irritant diaper dermatitis C. Uric acid crystals in the urine D. Uterine bleeding E. Vitamin K deficiency

Answer D Educational objective: A small amount of uterine bleeding in a newborn girl is normal and common. Parents should be reassured that the bleeding is secondary to withdrawal from maternal hormones.

17. A 19-year-old woman comes to the office for contraception counseling. She has been using condoms with her boyfriend but would like a more reliable contraception method. The patient has Wilson disease and a seizure disorder that is well controlled with carbamazepine. Which of the following is the best contraception method for this patient? A. Combination oral contraceptives B. Condoms plus spermicide C. Copper-containing intrauterine device D. Depot medroxyprogesterone acetate E. Diaphragm plus spermicide

Answer D Educational objective: All combined estrogen/progestin contraceptives are less effective in patients taking hepatic enzymeinducing antiepileptic drugs. These patients should be offered intrauterine devices or depot medroxyprogesterone acetate injection. The copper-containing intrauterine device is contraindicated in patients with Wilson disease.

2. A 39-year-old woman, gravida 4 para 4, comes to the office due to leakage of urine when she sneezes or laughs. The patient also has increasing vaginal pressure and the sensation of a vaginal bulge. She has had 4 vaginal deliveries and has completed childbearing. Menses occur every 30 days and her last menstrual period was 2 weeks ago. Which of the following is the best next step in management of this patient? A. Intermittent self-catheterization B. Oxybutynin C. Pelvic floor surgery D. Pessary placement E. Vaginal estrogen

Answer D Educational objective: Anterior vaginal wall prolapse (cystocele) may present with a sensation of a bulge or pressure in the vagina and stress urinary incontinence. Treatment includes lifestyle modifications, pelvic floor strengthening, and pessary placement for women with persistent symptoms affecting quality of life. Surgery is reserved for patients who have failed conservative treatment or decline the use of a pessary.

5. A 64-year-old man undergoes elective coronary artery bypass surgery for extensive coronary artery disease. His past medical history is significant for hypertension and hyperlipidemia. The surgery is uncomplicated, and he is successfully extubated the same day. His preoperative echocardiogram shows normal left and right ventricular function without any valvular disease. On the second postoperative day, he experiences fluttering in the chest. He has no chest pain, shortness of breath, or lightheadedness. His blood pressure is 132/88 mm Hg and pulse is 125/min. Pulse oximetry shows 98% on room air. ECG shows atrial fibrillation with rapid ventricular response. Which of the following is this patient most likely to experience within the next 24 hours? A. Acute cerebrovascular event B. Acute coronary graft closure C. Pericardial tamponade D. Spontaneous conversion to sinus rhythm E. Ventricular fibrillation

Answer D Educational objective: Atrial fibrillation (AF) is frequently seen after cardiac surgery. Most episodes of postoperative AF are selflimiting, with spontaneous conversion to sinus rhythm occurring in up to 80% of patients within the first 24 hours.

29. A 76-year-old man is seen in the emergency department with cough and worsening dyspnea over the last 2 days. He has known coronary artery disease with a prior myocardial infarction. Blood pressure is 166/100 mm Hg and pulse is 95/min. On examination, the patient is morbidly obese and appears uncomfortable. Lungs show bibasilar crackles and scattered bilateral wheezes. There is 1+ pitting ankle edema seen bilaterally. Serum chemistries are normal except for sodium of 125 mEq/L, creatinine of 1.4 mg/dL, and blood urea nitrogen of 30 mg/dL. Chest x-ray reveals prominent vascular markings and blunting of costophrenic angles. Echocardiogram shows a diffusely hypokinetic left ventricle with ejection fraction of 35%. Brain natriuretic peptide level is 96 pg/mL (normal <100 pg/mL). Which of the following explains the normal brain natriuretic peptide value in this patient? A. Decreased renal function B. Hyponatremia C. Noncardiac cause of dyspnea D. Obesity E. Older age

Answer D Educational objective: Brain natriuretic peptide is released from the ventricles in response to increased filling pressures and can help diagnose heart failure. However, higher levels are found in older patients and those with chronic kidney disease; lower levels are seen in obese patients and should be interpreted with caution.

28. A 47-year-old woman recovering from an upper respiratory infection comes to the office with a 3-day history of ear discomfort. The patient feels that her right ear is "plugged" and hears popping sounds in the ear when chewing or yawning. She also reports some pressure and slight pain in the ear. Her temperature is 37.2 C (99 F), blood pressure is 134/80 mm Hg, pulse is 84/min, and respirations are 14/min. Neck examination is normal and there is no lymphadenopathy. The left tympanic membrane is normal, but the right one appears mildly retracted. Romberg test and heel-to-toe gait are normal. A tuning fork placed in the middle of the head lateralizes to the right. The remainder of the physical examination is unremarkable. What is the best next step in management of this patient? A. Audiometric evaluation B. Head CT scan C. Nasal endoscopy D. Oral decongestants E. Oral glucocorticoids

Answer D Educational objective: Ear fullness, popping sounds, and decreased hearing sometimes can be due to obstruction of the eustachian tube during or following an upper respiratory infection. A short-term trial of decongestants is often effective.

22. A 35-year-old woman comes to the emergency department with persistent fever, severe sore throat, and progressive difficulty swallowing. She was seen in the office for fever and sore throat 3 days ago. At that time, point-of-care testing for group A Streptococcus was negative and symptomatic therapy was advised. Her temperature now is 38.9 C (102 F), blood pressure is 118/78 mm Hg, pulse is 90/min, and respirations are 16/min. She appears ill. Oropharyngeal examination is normal. Point-of-care testing for the EpsteinBarr virus is negative. Which of the following is the most appropriate next step in management of this patient? A. Epstein-Barr virus serology B. Group A Streptococcus throat culture C. HIV enzyme-linked immunosorbent assay testing D. Lateral neck x-ray E. Oral amoxicillin with outpatient follow-up

Answer D Educational objective: Epiglottitis should be suspected in patients whose sore throat severity is out of proportion to the findings on oropharyngeal examination. Drooling and stridor are seen in <40% of adult patients. Odynophagia and dysphagia are much more common (>90%).

17. A 4-year-old boy is brought to the emergency department due to a sore throat. The patient has had cough and fever for the last 2 days. Since this morning, he has not wanted to talk due to throat pain. He has received no childhood vaccinations. Temperature is 38 C (100.4 F), blood pressure is 108/70 mm Hg, pulse is 140/min, and respirations are 32/min. Pulse oximetry shows 86% on room air. The patient sits on the examination table with his hands braced on his knees. He is drooling and leaning forward with his neck extended. He has inspiratory stridor, and his responses to questions are difficult to understand due to a muffled voice. Which of the following is the best next step in management of this patient? A. Administer intravenous corticosteroids B. Obtain blood cultures C. Perform lateral neck x-ray D. Prepare for endotracheal intubation E. Provide nebulized albuterol

Answer D Educational objective: Epiglottitis, most commonly caused by Haemophilus influenzae type b, presents with acute onset of fever, sore throat, muffled voice, drooling, and stridor due to airway obstruction. Due to risk of rapid respiratory deterioration (particularly in children), airway management is the primary goal, including endotracheal intubation.

9. A parent contacts the on-call physician regarding a 3-year-old boy who has had a nosebleed for the past 20 minutes. This is the patient's third episode of nose bleeding; the prior episodes stopped after a few minutes when the mother inserted tissues in his nostrils. He is otherwise healthy with no medical problems. There is no family history of easy bleeding or bruisability. The mother is concerned about the recurrent nosebleeds and would like to know what she should do. Which of the following is the most appropriate management of this patient's epistaxis? A. Apply a topical vasoconstrictor to both nostrils B. Refer for urgent silver nitrate cauterization and laboratory studies C. Tilt the head back and apply direct pressure over the nasal bridge for ≥5 minutes D. Tilt the head forward and compress the nasal alae together for ≥5 minutes E. Tilt the head forward and insert tissues in both nostrils for ≥5 minutes

Answer D Educational objective: Epistaxis is usually due to bleeding from the Kiesselbach plexus. Initial management consists of continuous compression of the nasal alae for >5 minutes with the head tilted forward. Epistaxis that does not respond to compression can be treated with a topical vasoconstrictor.

49. A 32-year-old woman comes to the office to discuss sexual symptoms. She married 3 years ago and had regular sexual intercourse with her husband during the first year. However, for the last 2 years, sexual activity has declined significantly. The patient has no interest in coitus with her husband despite his frequent attempts. During these 2 years, the few times she has engaged in sexual activity were not pleasurable for her, although she does not have dyspareunia. Medical history is unremarkable; her only medication is an oral contraceptive, which she has been taking for the last 10 years. The patient does not use tobacco or abuse alcohol. Pelvic examination is normal. Which of the following is the most likely diagnosis in this patient? A. Depression B. Medication-induced sexual dysfunction C. Orgasmic disorder D. Sexual interest/arousal disorder E. Vulvovaginal atrophy

Answer D Educational objective: Female sexual interest/arousal disorder is characterized by reduced sexual interest and reduced responsiveness to sexual stimuli. Management is initially with couple's sex therapy. Bupropion and flibanserin can be considered.

5. A 48-year-old man is hospitalized with nausea, abdominal distension, and decreased appetite for the last 2 weeks. He also reports passing dark, tarry stool the day before admission. The patient has a history of intravenous drug abuse. On the second day of hospitalization, he appears confused and inattentive and cannot count backwards from 10. The patient's blood pressure is 110/60 mm Hg, pulse is 80/min, and respirations are 16/min. Cranial nerves appear intact. Abdominal distension with shifting dullness is present. When the patient holds his arms in front of him with wrist extended, his hands repeatedly jerk up and down. Laboratory results are as follows:Sodium 130 mEq/L,Potassium 3.1 mEq/l,Bicarbonate 20 mEq/L,Creatinine 0.9 mg/dL,Blood urea nitrogen 32 mg/dL. Which of the following is the most appropriate treatment for this patient's altered mental status? A. Chlordiazepoxide B. Furosemide C. Intravenous thiamine D. Lactulose E. Naloxone

Answer D Educational objective: Hepatic encephalopathy is an alteration in central nervous system function due to poor hepatic clearance of toxins. It is often precipitated by sedating medications, infection, or gastrointestinal bleeding, and symptoms may range from mild confusion to coma. Treatment includes lactulose, rifaximin, and laxatives.

28. An 82-year-old woman has intermittent dizziness and has fallen twice but was not injured. Her medical problems include hypertension, type 2 diabetes mellitus, severe osteoporosis, coronary artery disease, and chronic renal disease. Her medications include hydrochlorothiazide, metformin, vitamin D, metoprolol, and alendronate for the past 4 years. Blood pressure is 112/65 mm Hg, and pulse is 86/min and regular. Physical examination is unremarkable. Laboratory results are as follows: Serum chemistry, Sodium 132 mEq/L ,Potassium 3.6 mEq/L ,Chloride 100 mEq/L ,Bicarbonate 20 mEq/L ,Blood urea nitrogen 19 mg/dL ,Creatinine 1.3 mg/dL ,Calcium 10 mg/dL ,HbA 1c 7.5%, Complete blood count is within normal limits. Which of the following is the most appropriate next step in management of this patient? A. Add amlodipine B.Add glyburide C.Stop alendronate D. Stop hydrochlorothiazide E. Stop metformin, start glyburide

Answer D Educational objective: Hypertension and diabetes should be managed less aggressively in patients age >80, especially those who have adverse effects from therapy. Recommended targets for elderly patients include: Blood pressure 140-150/80 mm Hg HbA 1c <8%

41. A 58-year-old man comes to the emergency department with chest pain that started 30 minutes ago when he was lifting heavy boxes. He has squeezing pain behind the sternum that waxes and wanes in intensity and currently has a severity of 2 on a scale of 1-10. Medical history includes hypertension and type 2 diabetes mellitus. Blood pressure is 154/90 mm Hg and pulse is 95/min. Oxygen saturation is 98% on room air. Examination shows mild diaphoresis. There are no heart murmurs, and the chest is clear on auscultation. ECG shows normal sinus rhythm without any significant abnormalities. Which of the following is the best next step in management of this patient? A.CT pulmonary angiography B. Oral ibuprofen C. Reassurance and oral alprazolam D. Repeat ECG in 10 minutes E. Treadmill stress testing

Answer D Educational objective: Initial resting ECG and cardiac biomarkers are often normal in patients with acute coronary syndrome (ACS). Patients presenting to the emergency department with chest pain suggesting ACS but with negative results on initial ECG should be held for serial testing.

36. A 23-year-old woman, gravida 1 para 0, at 33 weeks gestation comes to the office for a routine prenatal visit and to discuss postpartum contraception options. She has tried oral contraceptive pills in the past, but discontinued use due to moodiness and breast tenderness. The patient is interested in spacing her pregnancies using a natural method. Which of the following is the best postpartum contraception choice for this patient? A. Basal body temperature charting B. Cervical mucus monitoring C. Coitus interruptus D. Lactational amenorrhea method E. Rhythm method

Answer D Educational objective: Lactational amenorrhea is the most effective method of natural pregnancy prevention for postpartum women who are exclusively breastfeeding, amenorrheic, and within 6 months of delivery.

20. A 77-year-old man with a long-standing history of hypertension comes to the physician due to exertional shortness of breath and fatigue for the last 3 months. His medications include furosemide 40 mg, amlodipine 10 mg, and low-dose aspirin. His blood pressure is 138/76 mm Hg and pulse is 65/min. Jugular venous pressure is estimated at 8 cm H2 O. Cardiac examination is notable for an S4 gallop and nondisplaced apical impulse. Lung auscultation shows bibasilar crackles. There is bilateral pitting ankle edema. ECG shows normal sinus rhythm and voltage criteria for left ventricular hypertrophy. Echocardiography shows left atrial enlargement, left ventricular hypertrophy, and left ventricular ejection fraction of 65%. Results of a basic metabolic panel are unremarkable. Which of the following is the best treatment option for this patient? A. Angiotensin II receptor blocker B. Beta blocker C. Digoxin D. Increased dose of loop diuretic E. Metolazone

Answer D Educational objective: Left ventricular diastolic dysfunction is a common cause of acute decompensated heart failure. The primary goals in management of patients with heart failure with preserved ejection fraction are to maintain euvolemia with diuretics, control systolic and diastolic blood pressure, control ventricular rate in those with atrial fibrillation, and optimize coronary revascularization in patients with documented ischemia.

3. A 66-year-old woman comes to the emergency department due to watery diarrhea, nausea, and vomiting for the past 2 days. Ten days ago, she was discharged from the hospital after being treated for communityacquired pneumonia.Temperature is 39.3 C (102.7 F), blood pressure is 90/60 mm Hg, and pulse is 120/min. Physical examination shows abdominal distension and diffuse tenderness but no rebound or rigidity. CT scan of the abdomen reveals dilation of the colon, primarily in the ascending and transverse colon (8 cm), with mucosal thickening. Laboratory results are as follows:Leukocytes 23,600/mm,Creatinine 2.6 mg/dL (baseline: 1.5), Plasma lactic acid 4.1 mEq/LIn addition to intravenous metronidazole and oral vancomycin, which of the following is the most important next step in management of this patient? A. Colonoscopic decompression B. Intravenous corticosteroids C. Rectal vancomycin D. Surgical evaluation

Answer D Educational objective: Management of patients with fulminant Clostridioides difficile infection (ie, toxic megacolon, ileus, hypotension) includes high-dose oral vancomycin, intravenous metronidazole, and immediate surgical evaluation. Patients may benefit from colonic resection or diverting loop ileostomy with colonic lavage.

15. A 28-year-old woman comes to the office due to bilateral breast discomfort that occasionally interferes with physical activity. The pain begins the week prior to menses and subsides on the first day of her menstrual cycle, which occurs every 30 days; her last menstrual period was 3 weeks ago. Two years ago, she was diagnosed with leg vein thrombosis after an ankle fracture and was treated with a direct oral anticoagulant for 3 months. Family history is significant for osteoporosis in her grandmother and aunt. Vital signs are normal. BMI is 26 kg/m 2 . Physical examination reveals bilateral, mildly tender nodularity of the upper lateral breast quadrants. There are no skin changes, palpable lymph node enlargement, or nipple discharge. Which of the following is the best next step in management of this patient? A. Breast ultrasound B. Progestin therapy C. Serum estradiol, FSH, and prolactin levels D. Supportive bra E. Tamoxifen therapy

Answer D Educational objective: Mastalgia is a benign breast condition characterized by bilateral breast pain and tenderness that occurs up to 2 weeks prior to menses and wanes with the onset of menses. Symptomatic relief can be achieved by wearing a properly fitted bra that provides adequate breast support.

27. A 27-year-old woman comes to the emergency department due to acute onset right-sided pelvic pain. Blood pressure is 140/80 mm Hg and pulse is 114/min. The abdomen is tender to palpation and rebound and guarding are present. On bimanual examination, a tender mass is palpated in the right adnexa. Urine pregnancy test is negative. The specimen is surgically removed and gross pathology reveals the following: Which of the following is the most likely diagnosis in this patient? A. Endometrioma B. Functional cyst C. Hemorrhagic cyst D. Mature cystic teratoma E. Polycystic ovary syndrome

Answer D Educational objective: Mature cystic teratomas (dermoids) are benign tumors composed of mature tissue of endodermal (eg, thyroid), mesodermal (eg, muscle, bone), and ectodermal (eg, hair, adipose, sebum) origin. Because teratomas increase the risk for ovarian torsion and necrosis, management is with ovarian cystectomy.

8. A 52-year-old woman comes to the office for a health maintenance examination. The patient's last menstrual period was 8 months ago, and she has had no hot flashes or night sweats. She has hypertension, hyperlipidemia, and a 30-pack-year history of cigarette use. Her mother recently died from metastatic colon cancer and her father has Alzheimer disease. Blood pressure is 130/94 mm Hg. BMI is 22 kg/m 2 . In this patient, menopause hormone therapy is indicated to prevent which of the following conditions? A. Colorectal cancer B. Coronary heart disease C. Dementia D. Hormone therapy not indicated E. Osteoporotic fracture

Answer D Educational objective: Menopausal hormone therapy is indicated to treat patients with vasomotor (ie, hot flashes) and genitourinary (eg, vaginal atrophy) symptoms of menopause. It is not indicated for the prevention of chronic conditions (eg, coronary heart disease, dementia, colorectal cancer).

22. A 48-year-old woman comes to the office due to irregular menstrual cycles. Over the last year, the patient's menstrual cycles have increased in length. She currently has an average cycle length of 40 days with 5 days of heavy bleeding. She previously had 28-day cycles with 4 days of bleeding. The patient feels more irritable at work and also has night sweats and difficulty sleeping 2 or 3 nights a week. Physical examination is normal. A pregnancy test is negative. Which of the following is the best next step in management of this patient? A. Coagulation studies B. FSH level C. Pelvic ultrasound D. Reassurance only E. TSH level

Answer D Educational objective: Menopausal transition is characterized by menstrual irregularity and hypoestrogenic symptoms (eg, vasomotor symptoms, sleep disturbances). Women age >45 with these symptoms require no additional testing and should be provided reassurance.

23. A 15-year-old girl is brought to the office for evaluation of primary amenorrhea. The patient began to have breast development 2 years ago but has not reached menarche. She has no chronic medical conditions and has had no surgery. Vital signs are normal. Height and weight are at the 40th percentile. Physical examination shows Tanner stage 4 breasts and pubic hair. The external genitalia appear normal, but speculum examination reveals a blind vaginal pouch. FSH, TSH, and prolactin levels are normal. A pelvic ultrasound reveals an absent uterus. Karyotype is 46, XX. Which of the following is the best next step in management of this patient? A. Bone mineral density B. Brain MRI C. Echocardiogram D. Renal ultrasound E. Thyroid ultrasound

Answer D Educational objective: Müllerian agenesis causes primary amenorrhea due to failed uterine development. Patients with uterine anomalies often have associated renal anomalies; therefore, these patients require evaluation with a renal ultrasound.

19. A 62-year-old man comes to the office for follow-up of diabetes mellitus type 2. He admits missing his appointments and medications and not following a strict diet. The patient does not smoke but has chewed tobacco for many years. He drinks 6-10 beers each weekend. His body mass index is 27.5 kg/m 2 . On oral examination, a white patch is seen on the buccal mucosa. The lesion appears to have a granular texture, is not indurated, and is not removed by scraping with a tongue depressor. There is no regional lymphadenopathy. His hemoglobin A 1c is 8.5%. Which of the following is the most likely diagnosis? A. Aphthous stomatitis B. Candidiasis C. Herpes simplex virus infection D. Leukoplakia E. Squamous cell carcinoma

Answer D Educational objective: Oral leukoplakia is characterized by white patches or plaques over the oral mucosa that usually cannot be scraped off. Risk factors for development of leukoplakia are similar to those for squamous cell carcinoma (tobacco and alcohol use). Development of areas with induration and/or ulceration should prompt biopsy to rule out malignant transformation of the lesion.

3. A 4-year-old girl is brought to the emergency department with ear pain. For the past 4 days, she has been complaining of worsening right ear pain, itchiness, and difficulty hearing. She has no ear discharge or fever and has never had similar symptoms. Her parents clean her ears daily with cotton swabs. The child is otherwise healthy and has no chronic medical conditions. She has been enjoying swimming lessons at the neighborhood pool. She takes no medications and has no known allergies. Examination shows an afebrile girl in no acute distress. She winces with right tragal pressure. Her right ear canal is erythematous and partially occluded by edema. Whitish cerumen and pus are removed with a wire loop. Both tympanic membranes appear normal. Which of the following is the most appropriate treatment for this child's condition? A. Oral antibiotics B. Oral and topical antibiotics C. Topical acidifying solution D. Topical antibiotics and corticosteroids E. Topical antibiotics and wick placement

Answer D Educational objective: Otitis externa is characterized by inflammation and infection of the external ear canal. Patients typically present with ear pain, discharge, and decreased hearing along with tenderness to palpation of the tragus. Treatment for moderate-to-severe infection consists of aural toilet, topical antibiotics, and topical steroids.

30. A 40-year-old nulligravid woman comes to the office due to heavy menstrual bleeding and pelvic pain. The patient has had 8-10 days of heavy bleeding with the passage of large clots every month. She has constant dull pelvic pressure unrelieved by ibuprofen. The patient had an appendectomy 4 years ago with no complications. She stopped using oral contraceptives 3 years ago and is not sexually active. BMI is 23 kg/m 2 . Physical examination reveals an enlarged, irregularly shaped uterus and no other abnormalities. Which of the following is the most likely diagnosis in this patient? A. Adenomyosis B. Endometrial hyperplasia C. Endometriosis D. Fibroid uterus E. von Willebrand disease

Answer D Educational objective: Patients with a fibroid uterus can have heavy, prolonged menses; bulk symptoms (eg, pelvic pressure/pain); and an irregularly shaped, enlarged uterus. Treatment depends on symptom severity and desire for future fertility.

47. A 36-year-old woman, gravida 3 para 3, comes to the office due to amenorrhea. Her last menstrual period was prior to her most recent pregnancy, which ended in vaginal delivery 11 months ago. The patient underwent a bilateral tubal ligation after the delivery. Her postpartum course was uncomplicated and she stopped breastfeeding 2 months ago. Since then, the patient has had difficulty sleeping due to night sweats. BMI is 20 kg/m 2 . Physical examination is normal. Laboratory results are as follows: β-hCG <5 mlU/mL FSH 62 mIU/mL Prolactin 13 ng/mL TSH 3 μU/mL Which of the following is the most likely cause of this patient's amenorrhea? A. Adhesions from the tubal ligation B. Functional hypothalamic amenorrhea C. Lactational amenorrhea D. Primary ovarian insufficiency E. Sheehan syndrome

Answer D Educational objective: Patients with primary ovarian insufficiency have symptoms of menopause (eg, irregular menses or amenorrhea, hot flashes, night sweats) at age <40 and an elevated FSH (postmenopausal) level.

37. A 4-day-old girl is brought to the office for evaluation of jaundice. She was born at term to a 26-year-old woman by uncomplicated vaginal delivery and discharged home on the second day of life. The pregnancy was uncomplicated and the maternal blood type was B positive. The girl has been breastfeeding exclusively. She is voiding and stooling 5-6 times a day. Birth weight was 3.4 kg (7 lb 8 oz) and current weight is 3.2 kg (7 lb 1 oz). Examination shows jaundice of the face and neck. The rest of the examination is normal. Laboratory studies are as follows: 24 hours of life. 96 hours of life Total bilirubin 6 mg/dL. 15 mg/dL Direct bilirubin. 0.3mg/dL. 0.3 mg/dL Which of the following is the most likely cause of this newborn's jaundice? a. ABO incompatibility b. Breastfeeding failure jaundice c. Breast milk jaundice d. Physiologic jaundice e. Rh incompatibility

Answer D Educational objective: Physiologic jaundice of the newborn is common and usually self-limiting, although some infants may require phototherapy. The indirect hyperbilirubinemia is due to increased bilirubin production, decreased bilirubin clearance, and increased enterohepatic recycling.

1. A 63-year-old man reports nocturia once or twice a night, and he has noticed a decrease in the size and force of his urinary stream over the past 6 months. On examination, he appears healthy. General physical examination is within normal limits. On digital rectal examination, his prostate is mildly enlarged with a 1.5cm hard nodule in the right lateral lobe of the prostate. His prostate-specific antigen level is 1.5 ng/mL (normal <4 ng/mL). Which of the following is the best next step in this patient's management? A. Order MRI B. Reassessment of symptoms in 6 months C. Repeat prostate-specific antigen level in 6 months D. Schedule transrectal ultrasound-guided prostate biopsy E. Start tamsulosin and dutasteride

Answer D Educational objective: Prostatic nodules found on digital rectal examination should be evaluated promptly by transrectal ultrasound-guided prostatic biopsy, regardless of the prostate-specific antigen level.

10. Two randomized controlled trials compared the impact of a new antidiabetic agent on HbA1c . One trial included 4,000 participants and the other included 2,000 participants. The larger trial will have higher: A. Confidence interval limits B. External validity C. Outcome misclassification rate D. Precision E. Risk of type II error

Answer D Educational objective: Sample size is related to power, or the ability to detect a difference between groups when such a difference exists. A study with a larger sample size will have more precision to detect that difference.

4. A 35-year-old man comes to the office due to bloating, excessive flatus, and loose stools since an episode of food poisoning several months ago. The patient had been on a cruise ship when he developed nausea, vomiting, and diarrhea. Several other patrons developed similar symptoms that resolved with supportive care only. The patient's current symptoms began 1-2 weeks after returning home and usually occur in the morning shortly after breakfast. His usual breakfast consists of coffee with whole milk and cereal or oatmeal. Soon after eating, he develops bloating followed by crampy abdominal pain and loose stools. The symptoms occasionally recur after dinner. Medical history is insignificant. The patient drinks 1 or 2 beers on weekends and does not use tobacco or illicit drugs. He reports no other recent travel outside the country or unusual food exposures. Vital signs are normal. The abdomen is soft and nontender. Bowel sounds are present with loud borborygmi. There is no hepatomegaly or splenomegaly. Stool occult blood testing is negative.Complete blood count, serum chemistries, TSH, erythrocyte sedimentation rate, and HIV serology are within normal limits.Which of the following is the

Answer D Educational objective: Secondary lactose intolerance can develop after acute infection or inflammation and is characterized by crampy abdominal pain, bloating, increased flatulence, and diarrhea. Management involves a lactoserestricted diet, enzyme replacement, and vitamin D and calcium supplementation. Once the inciting insult resolves, recovery of lactose tolerance can occur over a period of months.

40. A 59-year-old woman with hypertension comes to the office for follow-up of an episode of transient, rightsided weakness that lasted less than 10 minutes and resolved spontaneously. Medications include lisinopril, amlodipine, and aspirin. Vital signs are normal. Cardiovascular examination shows a normal S1 and S2 with a grade 2/6 ejection-type murmur at the right upper sternal border. Peripheral pulses are 2+ and no edema is present. Neurologic examination is normal.An MRI of the brain reveals no acute abnormalities. MR angiography of the neck and head demonstrates 60% stenosis of the left internal carotid artery and 50% stenosis of the right internal carotid artery. ECG shows normal sinus rhythm, and 48-hour Holter monitor is unrevealing. Echocardiogram reveals mild left ventricular hypertrophy and normal left ventricular systolic function.Which of the following is the most appropriate intervention to prevent future neurovascular events in this patient? A. Carotid artery stenting B.Carotid endarterectomy C. Rivaroxaban D. Rosuvastatin E. Warfarin

Answer D Educational objective: Secondary prevention in patients with TIA or stroke due to carotid atherosclerosis should include antiplatelet agents (ie, aspirin, clopidogrel, or aspirin-dipyridamole), careful control of blood pressure, and lipid-lowering therapy with a statin. In addition, carotid endarterectomy is recommended for patients with TIA or stroke associated with high-grade (70%-99%) carotid stenosis.

8. An 84-year-old woman comes to the office after a recent fall. She was walking in a hallway the previous evening when she tripped and fell onto her left hip. She had no symptoms preceding the fall and no loss of consciousness. The patient, who lives alone, lay on the floor for about an hour before her daughter found her. She sustained no major injuries, only minor bruising on her left side. The patient's blood pressure is 155/70 mm Hg supine and 150/73 mm Hg standing, and pulse is 78/min and regular. Crepitus is heard on flexion of the knees bilaterally. Her hips have full range of motion bilaterally. Cranial nerves are within normal limits. Pinprick and light touch sensations are intact. X-rays of the left hip and pelvis reveal no fractures. Which of the following is the best next step in assessment of this patient? A. Carotid sinus massage B. Geriatric depression scale C. Mini-mental state examination D. Visual acuity testing E. Vitamin B 12 level

Answer D Educational objective: Sensory disturbances (eg, loss of vision, proprioception) are a common cause of falls in the elderly and should be assessed and treated as part of a multifactorial fall prevention plan.

33. A 47-year-old man comes to the clinic for follow-up after laboratory testing 2 months ago found an elevated fasting blood glucose level (150 mg/dL). He has no significant medical history and does not use tobacco, alcohol, or illicit drugs. Temperature is 36.7 C (98 F), blood pressure is 132/80 mm Hg, pulse is 72/min, and respirations are 16/min. BMI is 30 kg/m 2 . Fasting laboratory results are as follows: Glucose 165 mg/dL ,Total cholesterol 210 mg/dL ,LDL 140 mg/dL ,Triglycerides 170 mg/dL ,HDL 35 mg/dL ,Hemoglobin A1c 7.2% ,Serum creatinine 1.1 mg/dL The patient is counseled on intensive lifestyle modification, including dietary changes, exercise, and weight loss. Which of the following additional interventions is indicated for this patient? A. Fish oil supplementation B.Initiation of glimepiride C.Initiation of niacin D. Initiation of rosuvastatin E.No additional interventions

Answer D Educational objective: Statin therapy is indicated for all patients age ≥40 with diabetes. The dose depends on the overall risk of cardiovascular disease: patients with a calculated 10-year risk ≥20% should be given high-intensity statin therapy, whereas those with a 10-year risk <20% may be considered for moderate-intensity therapy.

37. A 34-year-old non-obese man with a strong family history of cardiovascular disease (first-degree relatives with heart attack and stroke) asks about the best diet to follow. Which of the following diets has been shown in a randomized clinical trial to reduce the risk of cardiovascular events? A. High-protein diet B.Low-fat diet C. Low-sodium diet D. Mediterranean diet E. Very low-calorie diet

Answer D Educational objective: The Mediterranean diet has been shown to reduce the risk of cardiovascular events. Characteristics of the Mediterranean diet include high intake of fruits, vegetables, whole grains, nuts, beans, and seeds; olive oil as the main source of unsaturated fat; moderate wine intake; and low red meat consumption.

16. A new test for streptococcal pharyngitis is based on Streptococcus pyogenes antigen detection. A study is conducted in high-risk subjects in an outpatient clinic network with a 60% prevalence of streptococcal pharyngitis (based on throat culture, the gold standard). The new test is reported to have a sensitivity of 75% and a specificity of 90%. Which of the following is the positive likelihood ratio for the new test? A. 0.28 (11%) B. 1 (7%) C. 2.5 (38%) D. 7.5 (33%) E. 9 (9%)

Answer D Educational objective: The likelihood ratio (LR) reflects how well a test rules in or rules out a disease. Positive LR = Sensitivity/(1 - Specificity) Negative LR = (1 - Sensitivity)/Specificity LRs (like sensitivity and specificity) are independent of disease prevalence.

23. A term boy is born via emergency cesarean section following complete placental abruption. He is pale, apneic, and floppy with no respiratory effort and no detectable heart rate. Positive pressure ventilation is provided at 40-60 breaths/min. The patient undergoes endotracheal intubation, and the ventilator pressure is increased. One minute later, the heart rate is 80/min, but there is still no respiratory effort. What is the most appropriate next step in management of this infant? A. Administer epinephrine via endotracheal tube B. Administer epinephrine via umbilical catheter C. Begin chest compressions D. Continue positive pressure ventilation E. Increase ventilation rate to 90 breaths/min

Answer D Educational objective: The most important step in newborn resuscitation is establishing effective positive pressure ventilation. This may require corrective ventilatory steps (eg, repositioning the mask, suctioning the airway, increasing pressure) or ultimately intubating the infant. If the infant's heart rate remains <60 after these steps, then chest compressions and epinephrine are indicated.

2. A 52-year-old man comes to the office for an annual preventive visit. He feels well, has an unremarkable medical history, and takes no medications. Colonoscopy 2 years ago was normal. He drinks 1 or 2 beers a night and does not use tobacco or illicit drugs. The patient lives alone, and his diet consists mainly of fast food and prepackaged frozen meals. Blood pressure is 128/79 mm Hg and pulse is 76/min. BMI is 29 kg/m 2 . Physical examination is unremarkable. Fasting laboratory studies reveal total cholesterol is 205 mg/dL, HDL cholesterol is 39 mg/dL, LDL cholesterol (calculated) is 124 mg/dL, triglycerides are 210 mg/dL, and glucose is 105 g/dL.Which of the following is the most appropriate dietary advice for this patient? A. A low-fat diet is preferred to a low-carbohydrate diet to prevent diabetes in this patient B. Adding a fiber supplement can reduce this patient's risk of colon cancer C. Reducing this patient's caloric intake by 500 kcal/day will lead to long-term weight loss of 0.45 kg (1 lb) per week D. This patient should reduce intake of trans fatty acids to lower the risk of cardiovascular mortality E. This patient should restrict sodium intake to <1,500 mg/day to p

Answer D Educational objective: Trans fatty acids are found in deep-fried, commercially baked, and heavily processed foods. Diets high in trans fatty acids are associated with increased LDL and triglycerides, reduced HDL, and elevated rates of cardiovascular mortality. Other fats (eg, saturated, monounsaturated, polyunsaturated) have less adverse effects.

19. A 61-year-old man comes to the emergency department with a 2-day history of lower abdominal pain. The patient has no nausea, vomiting, back pain, or urinary symptoms. His medical conditions include hypertension for which he takes lisinopril. Temperature is 37.8 C (100 F), blood pressure is 122/70 mm Hg, and pulse is 92/min. Mucous membranes are moist. There is moderate tenderness in the left lower quadrant without rebound tenderness. Laboratory results are as follows. Leukocytes 13,000/mm,Creatinine 0.7 mg/dL CT scan of the abdomen shows localized wall thickening in the sigmoid colon with pericolic fat stranding. Which of the following is the best treatment plan for management of this patient? A. Hospitalization, intravenous (IV) antibiotics, and serial abdominal examinations B. Hospitalization, IV antibiotics, and repeat CT scan in 2 days C. Hospitalization, IV antibiotics, and sigmoidoscopy D. Oral antibiotics with outpatient follow-up instructions E. Peripherally inserted central line placement and outpatient IV antibiotics

Answer D Educational objective: Uncomplicated diverticulitis can be managed with oral antibiotics and close outpatient follow-up. Hospitalization is advised for patients with structural complications (eg, abscess, fistula formation, colonic obstruction), high fever, significant leukocytosis, immunosuppression, impaired oral intake, significant comorbidities, or advanced age.

10. A 32-year-old woman comes to the emergency department with pelvic pain and increasing vaginal discharge. She first noticed intermittent pain 3 days ago, but the pain is now constant. The patient has type 2 diabetes mellitus controlled with a single oral medication. She has a penicillin allergy. Temperature is 38 C (100.4 F); all other vital signs are normal. The lower abdomen is mildly tender. Pelvic examination reveals uterine and cervical motion tenderness and purulent endocervical discharge. Pregnancy test is positive. Ultrasound shows a 6-week intrauterine pregnancy with cardiac activity and no pelvic abnormalities. Which of the following is an indication for hospitalization and inpatient treatment? A. Current age B. History of penicillin allergy C. History of type 2 diabetes mellitus D. Pregnancy status E. Uterine tenderness on examination

Answer D Educational objective: Uncomplicated pelvic inflammatory disease can be treated in the outpatient setting with oral antibiotics. However, certain conditions, including pregnancy, require hospitalization for parenteral antibiotics due to the increased risk of maternal morbidity.

31. A 22-year-old woman comes to the physician due to episodes of recurrent syncope over the last 3 years. She has occasional fainting episodes that happen in crowded places and after prolonged standing, and are preceded by nausea, sweating, a sensation of warmth, and lightheadedness. Physical examination and electrocardiogram (ECG) are unremarkable. Which of the following is the best initial management of this patient? A. Carotid sinus massage education B. Cognitive behavioral therapy C.Low-dose beta blocker therapy D.Physical counterpressure maneuver education E. Serotonin reuptake inhibitor therapy

Answer D Educational objective: Vasovagal syncope is often preceded by nausea, diaphoresis, bradycardia, and/or pallor. Management includes reassurance, avoiding triggers, and assuming the supine position at the onset of symptoms. Physical counterpressure maneuvers can abort or delay an episode of syncope.

22. A 9-hour-old girl in the newborn nursery has jaundice. Her transcutaneous bilirubin level at 8 hours of life is 8.4 mg/dL. The patient was born vaginally to a 31-year-old primigravida woman after an uncomplicated pregnancy. Her mother is O+ and had normal prenatal laboratory studies and ultrasound. The infant's vital signs are normal. Physical examination shows jaundice from the head to the abdomen and scleral icterus. The rest of the examination is unremarkable. Laboratory results at 9 hours of life are as follows: Complete blood count Hemoglobin 11.2 g/dL Platelets 260,000/mm Leukocytes 11,000/mm Reticulocyte count 5.6% Liver function studies Total bilirubin 10 mg/dL Direct bilirubin 0.7 mg/dL Immunology Coombs test Positive Blood type B+ Which of the following is the best next step in the management of this infant? A. Double-volume exchange transfusion B. Intravenous immunoglobulin C. Partial exchange transfusion D. Phototherapy E. Repeat bilirubin levels in 6 hours

Answer D Educational objective: ABO hemolytic disease is characterized by jaundice in the first 24 hours of life, anemia, elevated reticulocyte count, and a positive Coombs test. Newborns at highest risk are those with A or B blood type born to a mother with O blood type.

1. A 67-year-old woman comes to the emergency department with abdominal pain that started 2 days ago in the left lower quadrant but today involves the entire abdomen. The pain worsens with coughing. The patient has a history of diverticulitis that required hospitalization a year ago. Her temperature is 37.8 C (100 F), blood pressure is 90/60 mm Hg, and pulse is 104/min. Physical examination shows a diffusely tender abdomen with increased muscular tone. X-ray shows gas under the diaphragm. Which of the following is the most appropriate next step in management of this patient? A. Colonoscopy B. CT scan of the chest without contrast C. Nothing by mouth, intravenous antibiotics, and observation D. Surgical consult E. Upper endoscopy

Answer D Educational objective: Bowel perforation is an important complication of acute diverticulitis. Radiography may demonstrate free air under the diaphragm; management requires urgent surgical evaluation.

5. A 67-year-old diabetic woman complains of right arm swelling and pain. She was diagnosed with right foot osteomyelitis and started on intravenous ertapenem a week ago. A peripherally inserted central catheter in the right arm is used to administer the antibiotic on an outpatient basis. Her vital signs are normal. The right arm and hand demonstrate mild pitting edema. The catheter insertion site appears clear and has no discharge. The line flushes and draws blood easily. Ultrasound examination shows right subclavian vein thrombosis. Which of the following is the best next step in management of this patient? A. Administer tissue plasminogen activator B. Reassure the patient and repeat the ultrasound in a week C. Remove the catheter and observe D. Start anticoagulation E. Start vancomycin

Answer D Educational objective: Central catheters are the most common cause of upper-extremity venous thrombosis. Catheters should not be removed as long as they are intact and still needed. Anticoagulation should be started immediately and continued for 3 months following removal.

36. A 4-month-old full-term girl is brought to the emergency department by her mother due to concerns about poor weight gain. The infant is fed 3-oz bottles every 2-3 hours, after which she occasionally has small spit-ups. At age 2 months, her weight was at the 10th percentile for age and sex. At age 3 months, weight was at the 5th percentile, so her primary care physician increased the caloric content of her formula. Today, the infant's weight is at the 3rd percentile for age and sex. Vital signs are normal, and the remainder of her physical examination is normal.Which of the following is the most appropriate next step in management of this infant? A. Admit the infant to the hospital for further workup and observation B. Consult the child protective services team C. Obtain a complete blood count, metabolic panel, and urinalysis D. Obtain additional history from the mother E. Prescribe antireflux medication and discharge with close follow-up

Answer D Educational objective: Failure to thrive (FTT) is a term used to describe infants or children who are not gaining weight or whose weight is less than expected. Workup of an infant with FTT consists of a thorough psychosocial and medical history and physical examination. Some patients may require hospitalization for observation and further workup, which may include screening laboratory studies and diagnostic imaging.

52. A healthy 9-month-old girl is brought to the physician for a well-child visit. She is currently eating pureed oatmeal, fruits, vegetables, and meat. In addition, she breastfeeds 5 times daily and drinks 4-6 oz of water per day. The girl has not yet had eggs, nuts, dairy, or fish introduced into her diet. She has no allergies and has enjoyed all new foods. Her grandparents want the girl to have a more varied diet that includes cow's milk.Which of the following is the most appropriate advice for the parents? A. Avoid eggs until age 2 B. Avoid peanuts, tree nuts, shellfish, and fish until age 3 C. Avoid yogurt and cheese until age 1 D. Introduce cow's milk after age 1 E. Introduce cow's milk any time as a substitute for breast milk

Answer D Educational objective: Introduction of cow's milk should be delayed until after age 1 due to an increased risk of iron deficiency anemia and increased renal solute load. Other dairy products can be safely consumed in small amounts after age 6 months. Highly allergenic foods can also be introduced.

4. A 32-year-old man treated with combination chemotherapy for sarcoma comes to the emergency department with malaise and chills for the last 24 hours. He has had vague right-sided abdominal pain and diarrhea. The patient has no chest pain, shortness of breath, cough, or skin rash. His temperature is 38.9 C (102 F), blood pressure is 118/70 mm Hg, pulse is 102/min, and respirations are 18/min. Examination shows a pale man in no acute discomfort. He has mild gingival erythema but no evidence of bleeding. The chest is clear to auscultation. There are no heart murmurs. The abdomen is soft and nontender on palpation. The perianal area has no lesions. Laboratory results are as follows: Leukocytes 690/µL with 20% neutrophils Hemoglobin 8.6 g/dL Hematocrit 25% Platelets 74,000/µL Creatinine 0.9 mg/dL Chest x-ray shows no infiltrates. Urinalysis is normal. Which of the following is the best next step in management of this patient? A. Give acetaminophen and discharge home with close follow-up B. Give filgrastim and observe in the hospital for 24 hours C. Obtain blood cultures and observe closely in the hospital D. Obtain blood cultures and start intravenous piperacillin-tazobactam E.

Answer D Educational objective: Patients with febrile neutropenia should be started on empiric broad-spectrum antibiotics as soon as possible after blood cultures are obtained. Empiric monotherapy with broad-spectrum coverage (eg, cefepime, meropenem, piperacillin-tazobactam) is recommended for initial management.

29. A 56-year-old woman comes to the hospital with shortness of breath and is diagnosed with submassive pulmonary embolism and right femoral vein thrombosis. She has a history of locally advanced breast cancer treated with surgery, radiation, and hormonal therapy. The patient's BMI is 45 kg/m 2 . Serum creatinine level is 0.8 mg/dL. She is started on subcutaneous enoxaparin therapy. Which of the following is an indication for anti-Xa level monitoring in this patient? A. History of malignancy B. History of radiation therapy C. Hormonal therapy D. Morbid obesity E. Submassive pulmonary embolism

Answer D Educational objective: Patients with morbid obesity (BMI >40 kg/m 2 ) and/or renal failure receiving low molecular-weight heparin should have anti-factor Xa activity measured for appropriate dose adjustments.

35. A 24-year-old woman comes to the physician for evaluation of a low platelet count (45,000/µL) that had been detected during a screening for life insurance. She has no symptoms and her past medical history is unremarkable. She does not take any medications and does not smoke or drink alcohol. Physical examination is normal. Repeat platelet count is 39,000/µL. Peripheral blood smear shows a paucity of platelets but otherwise has no abnormalities. Pregnancy test is negative. Which of the following is the best next step in management of this patient? A. Antinuclear antibodies B. Bone marrow biopsy C. Flow cytometry of the peripheral blood D. HIV testing E. Serum protein electrophoresis

Answer D Educational objective: Patients with suspected immune thrombocytopenia should be screened for HIV and hepatitis C, as these are common secondary causes of thrombocytopenia.

43. A 44-year-old woman with no significant past medical history comes to the physician due to fatigue over the last 6 months. Examination shows multiple skin xanthelasmata, few hyperpigmented spots, and tenderness in the right upper quadrant of the abdomen. The liver is palpable 3 cm below the costal margin and mild splenomegaly is present. Laboratory studies show an alkaline phosphatase of 450 U/L, normal aspartate aminotransferase and alanine aminotransferase, and a positive antinuclear antibody. Which of the following is the most likely diagnosis in this patient? A. Familial hypercholesterolemia B. Hepatocellular carcinoma C. Nonalcoholic fatty liver disease D. Primary biliary cholangitis E. Systemic lupus erythematosus

Answer D Educational objective: Primary biliary cholangitis can present in middle-aged women with fatigue, pruritus, skin hyperpigmentation, and an elevated alkaline phosphatase. A positive antimitochondrial antibody supports the diagnosis. Ursodeoxycholic acid is the preferred initial treatment.

17. A 43-year-old woman with no significant medical history comes to the physician with episodic rectal pain for the past 8 months. About once a month she has severe rectal pain that lasts for minutes and resolves without intervention. She has no fever, diarrhea, anal trauma, rectal bleeding, or weight loss. She has 1 formed bowel movement every 3 days without any straining and does not associate the pain with bowel movements. She takes no medications.Her blood pressure is 110/60 mm Hg, pulse is 76/min, and respirations are 14/min. Physical examination reveals a soft, nontender abdomen with normal bowel sounds. There is no hepatomegaly or splenomegaly. Rectal examination shows normal tone, no external hemorrhoids, and guaiac-negative brown stool. Pelvic examination is also unremarkable.Laboratory results are as follows:Hematocrit 38% Mean corpuscular volume 84 fL Platelets 280,000/µL Leukocytes 7,500/µ.Colonoscopy is notable for small internal hemorrhoids and mild diverticulosis. Which of the following is the most likely cause of this patient's symptoms? A. Endometriosis B. Fibromyalgia C. Irritable bowel syndrome D. Proctalgia fugax E. Somatization disorder

Answer D Educational objective: Proctalgia fugax usually presents with severe and episodic rectal pain lasting seconds to minutes that is unrelated to bowel movements. Patients are usually asymptomatic between episodes. Diagnosis is made after excluding other gastrointestinal causes of anal and rectal pain.

24. A 67-year-old man with diabetes has had right shoulder pain for the last 4-5 weeks. He describes it as nagging and poorly responsive to over-the-counter ibuprofen. The patient's diabetes is not well controlled due to lack of dietary compliance. On physical examination, his right shoulder has full range of motion and no local tenderness. There is decreased vibration sense at both ankles and first metatarsal heads. Digital rectal examination shows an enlarged nodular prostate. Laboratory results are as follows: Hemoglobin 14.2 g/dL Creatinine 1.2 mg/dL Hemoglobin A1c 11.5% Prostate-specific antigen 22 ng/mL (normal <4 ng/mL) Plain radiographs of the right shoulder show no abnormalities. Results of a transrectal prostate biopsy are pending. Which of the following is the best next step in management of this patient's shoulder pain? A. CT scan of the right shoulder B. Dual-energy x-ray absorptiometry C. Physical therapy referral D. Radioisotope bone scan E. Trial of gabapentin

Answer D Educational objective: Radioisotope bone scans detect areas of increased bone turnover and are very sensitive for identifying osteoblastic bone metastases of prostate and other cancers.

3. A 34-year-old man with no significant past medical history is brought to the emergency department with skin flushing, dizziness, chest tightness, and abdominal cramps. The symptoms started after he finished eating at a local seafood restaurant. His girlfriend was dining with him and has similar but milder symptoms. His blood pressure is 100/70 mm Hg and pulse is 105/min. He is afebrile. Physical examination shows flushed skin and scattered urticaria. Which of the following is the most likely diagnosis? A. Acute allergic reaction B. Botulism C. Pufferfish poisoning D. Scombroid poisoning E. Staphylococcal food poisoning

Answer D Educational objective: Scombroid poisoning is due to consumption of improperly stored fish containing elevated levels of histamine and other toxins. Patients develop acute flushing and urticaria in the face and neck, gastrointestinal symptoms (nausea, vomiting, diarrhea), and headache.

26. A 34-year-old woman comes to the emergency department due to right-sided weakness and difficulty speaking since awakening this morning. She has had no fever, headaches, loss of consciousness, or vision problems. Her past medical history is significant for 2 miscarriages in the second trimester. Which of the following additional findings would be most consistent with antiphospholipid syndrome? A. Blast cells B. Erythrocytosis C. Lymphocytopenia D. Prolonged activated partial thromboplastin time E. Vitamin K deficiency

Answer D Educational objective: The lupus anticoagulant, an anti-phospholipid antibody, is a pro-thrombotic immunoglobulin that causes a spuriously prolonged partial thromboplastin time (PTT) in vitro. Younger patients who present with stroke (or venous thromboembolism), thrombocytopenia, and prolonged PTT should be tested for antiphospholipid antibody syndrome

14. A 76-year-old woman comes to the office for a follow-up 3 months after hospital discharge. She underwent right total knee replacement surgery and afterwards developed deep venous thrombosis of the right popliteal vein. The patient has never had blood clots before and has no cardiac history. Currently she has no symptoms. She takes warfarin daily. Physical examination shows a well-healed wound and no lower extremity swelling. Which of the following is the best advice for anticoagulation in this patient? A. Continue warfarin for 3 more months B. Continue warfarin for 9 more months C.Continue warfarin indefinitely D. Stop anticoagulation now E. Stop warfarin and start clopidogrel

Answer D Educational objective: Warfarin anticoagulation should be continued for a minimum of 3 months in patients who develop deep venous thrombosis as a result of reversible or time-limited risk factors (eg, surgery, pregnancy, oral contraceptive use, or trauma). Therapy can be discontinued 3 months after the provocative factor resolves.

13. A study on a rare neurodegenerative disease evaluated the sensitivity and specificity of diagnostic tests using different types of specimens (compared to muscle biopsy as the gold standard). The following results were reported: Specimen sensitivity% specificity% CSF. 76. 99 Nasal swab. 90. 97 Skin. 92. 96 Urine. 98. 95 Which of the following specimen types could potentially provide the best screening strategy for the disease? A. Cerebrospinal fluid B. Nasal swab C. Skin D. Urine

Answer D Educational objective: Highly sensitive tests are useful for screening; highly specific tests are useful for confirmation. However, other characteristics such as disease prevalence and overall testing and treatment options should be taken into account when evaluating the usefulness of tests

12. A 67-year-old woman with hypertension and type 2 diabetes mellitus comes to the emergency department with palpitations and is diagnosed with atrial fibrillation. The arrhythmia resolves spontaneously in the emergency department without intervention. Her renal function is normal and she has no history of bleeding. Which of the following is the best treatment option for this patient? A. Aspirin B. Aspirin and clopidogrel C. Dipyridamole D. Rivaroxaban E. Ticagrelor

Answer D Educational objective: Anticoagulation with warfarin or target-specific oral anticoagulants (eg, dabigatran, rivaroxaban) is recommended to prevent thromboembolic events in patients with nonvalvular atrial fibrillation. Warfarin is preferred for patients with mitral stenosis, prosthetic heart valves, end-stage renal disease, and severe decompensated valvular disease likely to require valve replacement.

23. Investigators are conducting a study regarding the impact of a behavioral intervention on blood pressure control among patients in a network of primary care clinics. Patients with uncontrolled hypertension on 2 successive visits are approached and those who are interested in participating are enrolled after informed consent. Over a 6-week period, these patients receive a telephone call to verify their information and confirm their availability. At the next follow-up visit, patients who have agreed to continue with the study are randomized to a control group or an intensive counseling group. The study design best illustrates which of the following A. Allocation concealment B. Berkson bias C. Lead-time bias D.Neyman bias E. Run-in period

Answer E Educational objective: A run-in period is a short period in a trial before the intervention is applied. Run-in periods can be appropriate as a washout but if they are used to assess adherence, they can introduce bias and reduce the external validity of a study.

26. A 32-year-old woman comes to the physician after 2 weeks of worsening malaise, fever, and nasal congestion. The patient also has right-sided facial pain. She initially had "a cold" that resolved but shortly thereafter developed her current symptoms. The patient's past medical history is insignificant. Her temperature is 39.1 C (102.4 F), blood pressure is 110/70 mm Hg, pulse is 84/min, and respirations are 12/min. Physical examination shows tenderness to palpation over the right maxillary sinus, purulent nasal discharge, and right anterior cervical adenopathy. The remainder of the examination shows no abnormalities. Which of the following is the most appropriate next step in management of this patient? A. Bacterial culture of nasal discharge B. CT scan of the sinuses C. Plain x-rays of the sinuses D. Subspecialty referral for sinus endoscopy E. Treatment with antibiotics

Answer E Educational objective: Acute rhinosinusitis lasting <10 days is usually viral and typically resolves spontaneously; it does not require treatment with antibiotics. Severe symptoms and symptoms lasting >10 days suggest bacterial superinfection, and antibiotic therapy is recommended without imaging or additional testing in most cases.

9. A 19-year-old woman, gravida 1 para 0 aborta 1, comes to the student health center due to amenorrhea. The patient's last menstrual period was 6 months ago. Previously, her menses occurred every 28-30 days and would last 4 days. The patient had an elective termination of pregnancy at age 16. She plays lacrosse and just completed the season. Blood pressure is 100/60 mm Hg and pulse is 56/min. BMI is 18 kg/m 2 . Laboratory results are as follows: β-hCG <5 mIU/L FSH 3.8 mIU/mL (4-30 mIU/mL) Prolactin 11 ng/mL (<20 ng/mL) TSH 2.8 μU/mL (0.5-5 μU/mL) Which of the following is the best next step in management of this patient? A. Perform hysteroscopic lysis of intrauterine adhesions B. Prescribe a bisphosphonate C. Prescribe a combined hormonal contraceptive D. Prescribe olanzapine E. Recommend increasing caloric intake

Answer E Educational objective: Adolescent girls can present with the female athlete triad of inadequate nutritional intake, menstrual abnormalities (eg, amenorrhea, oligomenorrhea), and decreased bone mineral density. Treatment consists of increasing caloric intake and/or decreasing exercise.

21. A 72-year-old man with ischemic cardiomyopathy comes to the office following a hospitalization for decompensated heart failure. He says that he tires easily and becomes short of breath after walking a few blocks on a level surface. Medications include aspirin, carvedilol, sacubitril-valsartan, atorvastatin, and furosemide. Blood pressure is 128/76 mm Hg, pulse is 76/min, and respirations are 16/min. Lung examination shows clear lung fields. Cardiac examination shows a third heart sound. There is no peripheral edema. A recent echocardiogram showed left ventricular ejection fraction of 30%. Laboratory results show serum potassium of 3.9 mEq/L and creatinine of 1.0 mg/dL. Which of the following additional treatments would most benefit this patient? A. Amlodipine B. Digoxin C. Hydralazine D. Metolazone E. Spironolactone

Answer E Educational objective: Aldosterone antagonists (spironolactone, eplerenone) are recommended for patients with heart failure with reduced ejection fraction who continue to have symptoms despite appropriate therapy with a diuretic, beta blocker, and angiotensin system blocker.

12. A 24-month-old boy is brought to the office for a routine well-child visit. The child is growing and developing normally and has no medical conditions. He weighs 12 kg (26.4 lb), which is at the 50th percentile for age. The family has been using public transportation but will soon be purchasing a car. The parents request advice on car seat safety and placement. Which of the following is the most appropriate advice for the parents? A. Children ≥10 kg (22 lb) should be placed in front-facing car safety seats in a back passenger row B. Children age ≥24 months can transition from car safety seats to belt-positioning booster seats in the front seat C. Children age ≥24 months should be placed in front-facing car safety seats in a back passenger row D. Children can be placed in rear-facing car safety seats in the front seat if the airbag is active E. Children should be placed in a rear-facing car safety seat in a back passenger row until they outgrow it

Answer E Educational objective: Children can transition from rear-facing car safety seats to forward-facing car seats once they surpass the weight or height limits of the car seat (typically age 2-4). All children age <13 should remain in a back passenger row away from active airbags.

16. A 54-year-old man has experienced fatigue for the last 6 months. He has no significant past medical history. Examination shows mild splenomegaly. Laboratory results are as follows: Hemoglobin 9.4 g/dL Mean corpuscular volume 84 fL Platelets 110,000/mm Leukocytes 52,000/mm Segmented 55% neutrophils Bands 8% Metamyelocytes 6% Blasts 3% Basophils 8% Lymphocytes 11% Monocytes 9% Cytogenetic testing shows a bcr/abl translocation. Which of the following is the best initial treatment option for this patient? A. Alkylating agent B. Bone marrow transplantation C. Leukapheresis D.Monoclonal anti-CD20 antibody E. Tyrosine kinase inhibitor

Answer E Educational objective: Chronic myeloid leukemia (CML) is caused by a translocation of chromosomes 9 and 22, which produces the Philadelphia chromosome. The presence of the bcr/abl fusion protein is diagnostic of CML and results in unregulated tyrosine kinase activity. Tyrosine kinase inhibitors such as imatinib are the initial treatment of choice in almost all patients.

13. A 45-year-old woman, gravida 2 para 2, comes to the office for contraception counseling. For the past year, the patient has had irregular bleeding, which sometimes soaks through her clothing, and she cannot differentiate it from her menses. The patient has obesity, insulin-dependent type 2 diabetes mellitus, and hypertension. In her twenties, she had pelvic inflammatory disease that was treated with intravenous antibiotics. Sexually transmitted infection screening has since been negative. The patient smokes half a pack of cigarettes daily. Which of the following is a contraindication for placement of a progestin-releasing intrauterine device in this patient? A. Cigarette use B. Insulin-dependent diabetes mellitus C. Obesity D. Prior pelvic inflammatory disease E. Unexplained vaginal bleeding

Answer E Educational objective: Contraindications to intrauterine device placement include unexplained vaginal bleeding, pregnancy, acute pelvic infection, endometrial or cervical cancer, and uterine cavity distortion.

5. A 3-year-old boy is brought to the physician with 2 days of watery diarrhea, abdominal pain, and a fever of 38 C (100.4 F). He has not eaten much in the past day but is drinking small amounts of fluids without vomiting. He has no underlying health problems and has no other complaints. He takes no medications and has no allergies. During the examination, the parents ask if the child is dehydrated. A change in which of the following is the most accurate method of assessing the degree of dehydration? A. Capillary refill B. Pulse C. Respiratory rate D. Skin elasticity E. Weight

Answer E Educational objective: Decreased oral intake, diarrhea, and fever are risk factors for dehydration. Percent weight loss is the gold standard in determining degree of dehydration. Familiarity with other clinical signs of dehydration is essential to guide appropriate rehydration therapy and prevent severe dehydration.

10. An 80-year-old man comes to the office due to generalized weakness and intermittently feeling "dizzy" and unsteady over the past year. He fell 9 months ago when he tripped over a step. The patient has had no syncope or chest pain and has not attempted to use any mobility aids. History is notable for hypertension, type 2 diabetes mellitus, hypercholesterolemia, osteoarthritis, coronary artery disease, and gastroesophageal reflux disease. Current medications include metformin, glipizide, hydrochlorothiazide, aspirin, atenolol, simvastatin, and omeprazole. The patient does not use tobacco or alcohol. Blood pressure is 150/90 mm Hg and pulse is 68/min. There are no orthostatic changes. BMI is 30 kg/m 2 . Examination shows normal jugular venous pressure, clear lung fields, and normal S1 and S2. The abdomen is soft and nontender. There is no tenderness to palpation of the lower back. There are symmetric joint deformities of both knees and generalized arthritic changes in both hands. Cranial nerve examination is normal. Muscle power is 4/5 in both upper and lower extremities. Pinprick and light touch sensations are mildly decreased in both feet. Achilles reflexes are absent bilaterally. C

Answer E Educational objective: Disequilibrium is a nonspecific sense of weakness or imbalance with ambulation. It is common in elderly patients with multiple comorbid conditions. Contributing factors may include cerebellar dysfunction, cervical spondylosis, Parkinsonism, peripheral neuropathy, medication side effects, arthritis, muscle weakness, and visual impairment. Management usually requires a multidisciplinary program including regular exercise, physical therapy, and occupational therapy

9. A 74-year-old man recovering from recent ischemic stroke develops right ankle and calf swelling. He has mild, persistent right-sided hemiplegia and has been receiving daily physical therapy for the last week. Compression ultrasonography confirms right-sided popliteal vein thrombosis. The patient is started on a therapeutic dose of enoxaparin twice daily. When is it appropriate for this patient to resume mobilization and physical therapy? A. 3-5 days after starting anticoagulation B. After inferior vena cava filter placement C. After transition to oral anticoagulant D. After ultrasound confirms resolution of the thrombus E. Within 24 hours of starting enoxaparin

Answer E Educational objective: Early ambulation following deep venous thrombosis is associated with fewer complications compared to prolonged bed rest. It is reasonable to start mobilization once therapeutic levels of anticoagulation have been reached.

7. A 13-month-old girl is brought to the physician for a routine well-child visit. She recently learned how to walk and has sucked her thumb since age 2 months. She eats a variety of soft finger foods and is nursed to sleep every night. Her parents brush her teeth twice a day with fluoride toothpaste. The girl has no underlying health problems and her immunizations are up-to-date. Her growth is tracking at the 50th percentile for height, weight, and head circumference. Examination shows 4 mandibular and 4 maxillary incisors, all with no decay. Which of the following is the most appropriate recommendation for this patient? A. Brush teeth with fluoride-free toothpaste B. Discontinue breastfeeding C.Discourage thumb-sucking D. Initiate fluoride supplementation E. Referral to a dentist

Answer E Educational objective: Education on oral hygiene and healthy dietary behaviors should be provided at every well-child visit during infancy. Children with significant risk factors (frequent ingestion of sugary snacks, juice in bottle) should be referred to a dentist promptly. The dental home should be established by age 1 for all children.

2. A 9-month-old boy is brought to his physician for routine examination. He has no acute illness and no significant medical history. The boy's growth parameters and physical examination are normal. His parents are concerned that he is not yet pulling up to the standing position and are considering buying an infant walker to help his motor development. Which of the following is the most appropriate recommendation for this family? A. An infant walker can improve the ability to stand B. An infant walker encourages early ambulation C. An infant walker is safe on the ground floor of a home D. An infant walker is safe when the parents are in the same room E. An infant walker is unsafe and should not be purchased

Answer E Educational objective: Falls (particularly down stairs), burns, immersion injuries and death have been associated with mobile infant walkers. There are no circumstances in which mobile infant walkers are considered safe, nor are there any demonstrable benefits to infant development. As a result, mobile infant walkers are not recommended.

31. An 18-year-old woman comes to the office due to amenorrhea. Her menses were previously normal, with 4-5 days of moderate bleeding each month. However, over the past year, her periods have become increasingly irregular. She now has not had a period in 5 months. The patient has no chronic medical conditions. She is not sexually active and does not use contraception. She started a new job 6 months ago. Blood pressure is 100/70 mm Hg and pulse is 56/min. BMI is 18 kg/m 2 . Physical examination is normal. Laboratory results are as follows: Follicle-stimulating hormone 3 mIU/mL Prolactin, serum 5 ng/mL Thyroid-stimulating hormone 2.7 µU/mL Urine pregnancy test negative This patient is at greatest risk for which of the following long-term complications? A. Aortic root dilation B. Bitemporal hemianopsia C. Endometrial adenocarcinoma D. Epithelial ovarian cancer E. Osteoporotic fracture

Answer E Educational objective: Functional hypothalamic amenorrhea is due to suppression of the hypothalamic-pituitary-ovarian axis by stress, excessive exercise, or relative caloric deficiency. Long-term complications include infertility and osteoporosis due to estrogen deficiency.

3. A 16-year-old previously healthy girl comes to the office for evaluation of a rash. She ate a crab cake for lunch and developed diffuse hives with a rash on her arms, legs, and trunk that resolved later that day. The patient also had an episode of scratchy throat and nausea after eating fish a few months ago. At that time, her lips became swollen without associated dyspnea or wheezing. She has no known allergies and never had any symptoms after eating seafood in the past. Her examination is within normal limits. Which of the following is the best next step in management of this patient? A. Advise lifelong avoidance of seafood and prescribe injectable epinephrine B. Obtain food-specific IgG4 levels C. Obtain serum C1 inhibitor functional level D. Perform oral food challenge E. Refer to allergy specialist for skin prick test

Answer E Educational objective: IgE-mediated food allergies typically begin within minutes to hours after ingestion and can present with acute urticaria, angioedema, respiratory or gastrointestinal symptoms, or anaphylaxis. Suspected IgEmediated food allergies should prompt allergy testing (eg, skin prick testing or in vitro tests) for confirmation.

16. According to the United States Preventive Services Task Force (USPSTF), in which of the following scenarios would intimate partner violence screening be most appropriate? A. Annual examination for a 40-year-old woman who has been married for 15 years and has 2 children B. Contraceptive counseling visit for 17-year-old girl with 2 new male sexual partners over the past 6 months C. In vitro fertilization consultation for a 33-year-old woman in a monogamous same-sex relationship for the past 5 years D. Initial prenatal visit for 28-year-old primigravida at 12 weeks gestation with a boyfriend of 1 year E. Screening is appropriate in all women of childbearing age regardless of relationship status

Answer E Educational objective: Intimate partner violence (IPV), which includes both physical/sexual and psychological abuse, is more common in women. Due to high rates of both IPV and underreporting, all women of childbearing age should undergo routine IPV screening (regardless of risk factors).

1. A 24-year-old woman, gravida 2 para 1, at 39 weeks gestation is admitted in active labor. Her pregnancy has been uncomplicated. The patient received an epidural on admission, and the pain has improved. Vital signs are normal. The cervix is 6 cm dilated and 100% effaced, and the fetal head is at 0 station. Which of the following is the best next step in management of this patient? A. Continue current management B. Decrease the epidural infusion rate C. Perform an emergency cesarean delivery D. Place an intrauterine pressure catheter E. Place the patient in the left lateral position

Answer E Educational objective: Management of a category II fetal heart rate includes intrauterine resuscitation measures (eg, maternal repositioning) and correction of the underlying etiology. If the fetal heart rate tracing persists or worsens, emergency delivery is recommended.

2. A 72-year-old Chinese-speaking woman is brought to the office by her daughter due to a persistent cough, weight loss, and hemoptysis. Chest x-ray suggests a tumor and the physician recommends a diagnostic bronchoscopy. The physician explains the reasons for the bronchoscopy and risks of the procedure to the daughter, who is fluent in both English and Chinese. The daughter translates the information into Chinese and indicates that her mother agrees with the plan. Which of the following is the most appropriate course of action in obtaining informed consent? A. Asking the daughter to witness the informed consent form B. Asking the patient to repeat back what she has understood, using the daughter to translate C. Having the daughter translate any questions the patient may have D. Having the patient read and sign a Chinese language version of the consent form E. Using a trained Chinese language interpreter

Answer E Educational objective: Medically trained interpreters should be used during the informed consent process to ensure that patients have adequate understanding of and full participation in the decision-making process.

7. A 14-year-old girl is brought to the office by her mother for evaluation of heavy menstrual bleeding. The girl has monthly menses with 4-5 days of bleeding and passage of large clots. The patient is wheel-chair bound due to a spinal cord injury from a motor vehicle collision several years ago. Hemoglobin is 9.8 g/dL; all other laboratory results are normal. Which of the following is the most appropriate management option for this patient? A. Depot medroxyprogesterone acetate B. Dilation and curettage C. Endometrial ablation D. Gonadotropin-releasing hormone agonist E. Progestin-containing intrauterine device

Answer E Educational objective: Menstrual management with hormonal contraception (eg, continuous oral contraceptives, progestincontaining intrauterine device) can be used to manage pubertal complications (eg, heavy menstrual bleeding, anemia) in adolescents with disabilities.

14. A 53-year-old business executive comes to the office with right ear fullness and decreased hearing after returning from a trip to China 3 days ago. The patient developed ear discomfort while the plane was landing, and it has persisted. He has a history of hypertension and diet-controlled diabetes mellitus. Otoscopic examination reveals an intact, noninflamed tympanic membrane with an air-fluid level. Which of the following is the best management for this patient's condition? A. Azithromycin B. Diphenhydramine C. Loratadine D. Prednisone E. Reassurance and follow-up only

Answer E Educational objective: Middle ear barotrauma can occur due to air travel, underwater diving, skydiving, or blast injuries. Patients can develop hearing loss, tinnitus, middle ear effusion, and tympanic membrane bruising with possible rupture. Treatment is supportive.

21. A 24-year-old woman comes to the office due to a painful lesion in her mouth. She has had similar lesions once or twice a year over the last several years that caused significant discomfort with chewing and lasted 6-7 days. She has no associated rashes, joint pains or swelling, or genital lesions. She does not have any significant past medical history and does not use tobacco or alcohol. The patient is HIV-negative. Her mother has rheumatoid arthritis. Physical examination findings are shown in the image below. Complete blood count and comprehensive metabolic panel are within normal limits. Erythrocyte sedimentation rate is 12 mm/hr. Which of the following is the best next step in management of this patient? A. Anti-cyclic citrullinated peptide antibodies B. Antinuclear antibodies C. Colonoscopy D. Thyroid function tests E. Topical treatment

Answer E Educational objective: Minor variant recurrent aphthous stomatitis presents with a few ulcers that are small, occur on the nonkeratinized portions of the tongue, cheeks, and lips, and heal in 7-10 days without scarring. They typically recur every 1-4 months. Topical glucocorticoids are the treatment of choice.

2. A new antiplatelet agent reduces the risk of major bleeding over a year from 3% to 2% compared to a comparator drug. What is the number needed to treat to avoid 1 instance of major bleeding? A. 1 B. 10 C. 33 D. 50 E. 100

Answer E Educational objective: NNT = 1 / ARR, where NNT is number needed to treat and ARR is absolute risk reduction.

15. When adopting a screening test for an indolent cancer, there is a risk of overestimating survival among screen-detected cases due to the detection of very indolent tumors that do not manifest before the patient dies of other causes. This type of bias is called: A. Allocation bias B. Compliance bias C. Contamination bias D. Lead-time bias E. Overdiagnosis bias

Answer E Educational objective: Overdiagnosis bias increases the risk of overestimating survival among screen-detected cases due to the detection of very indolent tumors that do not manifest before the patient dies of other causes.

50. A 56-year-old woman comes to the office due to breast pruritus. The patient reports increasing pruritus around the right areola over the last 3 months that has not resolved with topical emollients. The pruritus is continuous and she is constantly scratching the breast. The patient has now noticed a lesion around the nipple and serosanguinous discharge on her clothing. She has had no change in skin products or detergents. The patient has type 2 diabetes mellitus controlled with metformin. She has had no previous surgeries and has no known allergies. Her last mammogram, 2 years ago, was normal. The patient does not use tobacco, alcohol, or illicit drugs. Vital signs are normal. The right breast has an ulcerative lesion over the right nipple and areola that does not extend to the surrounding tissue; there are no palpable masses or axillary lymph nodes. Serous fluid is expressed from the nipple. Examination of the left breast is unremarkable. Which of the following is the most likely diagnosis in this patient? A. Candida mastitis B. Contact dermatitis C. Ductal ectasia D. Intraductal papilloma E. Paget disease of the breast

Answer E Educational objective: Paget disease of the breast is characterized by a unilateral, erythematous, intensely pruritic, ulcerative lesion of the nipple-areolar complex. Diagnosis is by tissue biopsy. Patients also require diagnostic bilateral mammography, as many have underlying breast malignancy.

32. A 31-year-old woman with known moderate rheumatic mitral stenosis comes to the emergency department with palpitations and is diagnosed with atrial fibrillation. The patient's arrhythmia resolves spontaneously. She has normal renal function and has no history of bleeding. Which of the following is the best treatment option for this patient? A. Aspirin B.Aspirin and clopidogrel C.Rivaroxaban D.Ticagrelor E. Warfarin

Answer E Educational objective: Patients with rheumatic mitral stenosis (MS) and atrial fibrillation (AF) have a very high risk of systemic thromboembolism. Oral anticoagulation with warfarin to a goal International Normalized Ratio of 2.5 (range 2.0-3.0) is recommended for patients with MS who develop AF or have a prior embolic event or left atrial thrombus

46. A 64-year-old man comes to the emergency department due to a right forearm laceration after tripping over a rake in his garden. He has some bleeding from the wound but no other symptoms. The patient was diagnosed with hypertension several years ago but has not followed up and takes no medications. Blood pressure is 210/118 mm Hg, and pulse is 78/min and regular. Cardiopulmonary and funduscopic examinations are unremarkable. ECG shows normal sinus rhythm with voltage criteria for left ventricular hypertrophy. Serum potassium is 3.9 mEq/L and creatinine is 0.8 mg/dL. The laceration is sutured and a tetanus vaccine given. Repeat blood pressure is 200/116 mm Hg.Which of the following is the best next step in management of this patient? A. Admit to hospital for intravenous antihypertensive therapy B.Advise lifestyle modifications and home blood pressure monitoring C.Obtain serial cardiac biomarkers and echocardiography D. Order diagnostic tests for secondary causes of hypertension E. Prescribe oral antihypertensive medication and arrange outpatient follow-up

Answer E Educational objective: Patients with severe asymptomatic hypertension (hypertensive urgency) warrant initiation of oral medication to lower blood pressure over hours to days but usually do not require inpatient care. Shortacting oral agents can be given initially and the blood pressure rechecked. The patient can then be prescribed long-acting agents to continue at home. Close outpatient follow-up is recommended.

15. A 63-year-old woman with known systemic sclerosis complains of abdominal distension, pain, and bloating accompanied by foul-smelling diarrhea for the past 6 months. She has lost 9 kg (20 lb) during this period. Her abdomen is distended and mildly tender diffusely. The liver and spleen cannot be felt. No synovitis or effusion is noted at any joints. Laboratory studies show macrocytic anemia. Barium enema shows multiple wide-mouthed diverticula in the colon. Small-bowel studies are pending. This patient's abdominal symptoms will most likely be relieved by which of the following medications? A. Extended-release nifedipine B. Glucocorticoids and cyclophosphamide C. Metoclopramide and erythromycin D. Octreotide E. Rifaximin

Answer E Educational objective: Patients with systemic sclerosis may develop small intestinal bacterial overgrowth due to reduced peristalsis, stasis, and intestinal dilation. Initial symptoms are abdominal bloating and pain; later symptoms include malabsorption with diarrhea, steatorrhea, and weight loss. Antibiotic therapy will usually alleviate the symptoms, but recurrent courses may be required.

21. A 35-year-old woman, gravida 1 para 1, who is 8 weeks postpartum comes to the office with multiple painful lumps in her breasts for the past week. She is breastfeeding and is otherwise healthy. Her only medication is a prenatal vitamin. The patient's mother had postmenopausal breast cancer. Vital signs are normal. Physical examination reveals multiple small and tender masses in bilateral breasts; there is no surrounding erythema. Milky discharge is expressed bilaterally. Which of the following is the most likely cause of this patient's symptoms? A. Galactocele B. Inflammatory breast cancer C. Mastitis D. Normal breast tissue E. Plugged ducts

Answer E Educational objective: Plugged milk ducts are localized areas of milk stasis that cause distension of breast tissue, resulting in pain, tender and palpable masses, and no signs of infection in breastfeeding women. Treatment is supportive and includes optimizing breastfeeding.

32. A 39-year-old woman comes to the office due to irritability, anxiety, and social withdrawal a week prior to menses. The patient has associated fatigue and abdominal bloating that often causes her to miss work and to cancel social engagements. The symptoms resolve on the third day of her menstrual period but have recurred with each cycle for the last year. She has a bilateral tubal ligation. Vital signs and physical examination are normal. Which of the following is the most appropriate treatment for this patient? A. Benzodiazepine B. Combination oral contraceptive C. GnRH agonist D.Nonsteroidal anti-inflammatory drug E. Selective serotonin reuptake inhibitor

Answer E Educational objective: Premenstrual dysphoric disorder (PMDD) causes significant affective symptoms during the luteal phase of the menstrual cycle, resulting in social impairment. Cyclic changes in progesterone and estrogen alter serotonin levels in patients with PMDD. Therefore, first-line treatment is with selective serotonin reuptake inhibitors.

15. A 70-year-old man comes to the office complaining of difficulty hearing. The hearing problem occurs during crowded family gatherings but not when he talks to family members at home. In addition, he has recently had to raise the television volume to a higher level than normal. He has no ear pain or discharge, headache, or dizziness. The patient has no history of significant noise exposure. On examination, he has decreased hearing bilaterally. What is the most likely diagnosis in this patient? A. Acoustic neuroma B. Ménière's disease C. Middle-ear effusion D. Otosclerosis E. Presbycusis

Answer E Educational objective: Presbycusis is characterized by sensorineural hearing loss that occurs with aging. It is usually first noticed in the sixth decade of life, and causes high-frequency bilateral hearing loss. Patients with presbycusis often have difficulty hearing in noisy and crowded environments.

2. A 15-year-old girl comes to the physician for evaluation of her third episode of "fever and runny nose" over the past few months. Two months ago, she was treated with 10 days of amoxicillin with improvement of symptoms, but the symptoms returned approximately 1 week after she completed treatment. The patient subsequently was treated with 2 weeks of cephalexin with temporary improvement of symptoms. She currently complains of fever, headache, and nasal congestion. She has no other medical problems and no known allergies. Her family history is unremarkable, but both parents smoke cigarettes. On examination, the girl has purulent nasal discharge and maxillary sinus tenderness. Nasal turbinates are swollen and erythematous. Respiratory, cardiac, and abdominal examinations are normal. Computed tomography scan demonstrates mucosal thickening, opacification, and air-fluid levels in the maxillary sinuses. What is the most likely cause of the recurrent infection in this patient? A. Cocaine abuse B. Cystic fibrosis C. Granulomatosis with polyangiitis D. Immunoglobulin G subclass deficiency E. Inadequate antibiotic treatment

Answer E Educational objective: Recurrent bacterial rhinosinusitis commonly results from inadequate antibiotic treatment due to increasing antibiotic resistance. Other predisposing factors include structural abnormalities, exposure to cigarette smoke, immunodeficiency, and inherited disorders affecting mucociliary clearance.

8. A 17-year-old boy is brought to the physician for evaluation of facial pain and swelling. The pain started on the right side of his jaw about 1 week ago. The pain is 7/10 in severity and worse with eating. The patient takes diphenhydramine a few nights a week as a sleep aid but no other medications. His temperature is 36.7 C (98 F). Examination shows swelling of the region anterior to the right ear and superior to the angle of the mandible. A small, hard mass is palpated near the opening of Stensen's duct in the right buccal mucosa. Which of the following is the most likely cause of this patient's condition? A. Acute viral infection of the parotid gland B. Autoimmune destruction of the salivary glands C. Malignant infiltration of the oral cavity D.Periodontal abscess secondary to poor oral hygiene E. Salivary duct obstruction

Answer E Educational objective: Sialolithiasis typically presents with a small, hard calcification at the salivary duct with surrounding pain and swelling. First-line treatment consists of hydration and sialagogues.

3. A 2-month-old full-term girl is brought to the physician for a routine well-child visit. She has been growing and developing normally. She breastfeeds for 30 minutes every 2-3 hours and wakes appropriately for feedings at night. The girl is put to sleep in the supine position in a bassinet next to her mother's bed and falls asleep with a pacifier. She lives in an apartment with her 23-year-old mother and maternal grandparents. There are no smokers in the home. The girl's mother previously smoked cigarettes but quit during the second trimester when she discovered she was pregnant. Physical examination is normal. Rotavirus, diphtheria-tetanus-acellular pertussis, haemophilus, pneumococcus, and inactivated polio vaccines are administered. Which of the following factors increases this infant's risk of sudden infant death syndrome? A. Infant's immunizations B. Infant's pacifier use C. Infant's sleep environment D. Maternal age E. Maternal history of smoking

Answer E Educational objective: Sudden infant death syndrome is the leading cause of mortality in infants. The most significant modifiable risk factors are prone sleeping position and prenatal and postnatal smoking exposure. Parents should be advised to place the infant supine on a firm crib or bassinet mattress in the same room as the parents. Breastfeeding and immunizations should also be encouraged as a preventive factor and for overall good health.

11. A 55-year-old postmenopausal woman comes to the office due to increasing pelvic pressure over the last 6 months. The patient has also had 2 months of occasional vaginal spotting. In addition, she has constipation unrelieved by stool softeners. Five years ago, the patient underwent a mastectomy, radiation therapy, and tamoxifen adjuvant therapy for early-stage estrogen receptor-positive breast cancer. Vital signs are normal. Pelvic examination reveals an enlarged, irregularly shaped uterus. Speculum examination reveals a small cervix with no active bleeding or lesions. A fecal occult blood test is negative. Which of the following is the most likely diagnosis in this patient? A. Adenomyosis B. Cervical cancer C. Endometriosis D. Metastatic breast cancer E. Uterine sarcoma

Answer E Educational objective: Tamoxifen, a selective estrogen receptor modulator, has estrogen agonist effects on the uterus that increase the risk of uterine sarcoma and endometrial hyperplasia/cancer. Uterine sarcoma can present in postmenopausal patients with an enlarged, irregularly shaped uterus that causes bulk symptoms (eg, pelvic pressure, constipation, urinary frequency) and postmenopausal bleeding.

19. A 75-year-old man comes to the physician for a sore on his ear that he says "won't heal." He is a retired lineman for an electrical company in a rural area. He first noticed a small lump on the top of his ear 1 year ago. It has grown and opened up into a painful sore that has not responded to a topical antibiotic cream. He has a history of hypertension, chronic gouty arthritis, hyperlipidemia, and chronic kidney disease with a stable serum creatinine of 2.0 mg/dL. What is the most likely diagnosis for this patient's nonhealing ulcer? A. Actinic keratosis B. Calciphylaxis C. Gouty tophus D. Pyogenic granuloma E. Squamous cell carcinoma

Answer E Educational objective: Tender, bleeding, or ulcerated papules, plaques, or nodules in sun-exposed areas or areas of chronic inflammation or scarring require prompt evaluation for cutaneous squamous cell carcinoma.

6. A 27-year-old man comes to the office due to a scrotal lump that his girlfriend noticed prior to sexual intercourse. The patient has no testicular pain or associated symptoms. Medical history is unremarkable, and the patient takes no medications. He does not smoke and drinks 2 or 3 alcoholic beverages a day. Vital signs are normal. Examination shows a 2-cm, nontender mass in the left testicle. Transillumination reveals no appreciable transmission of light. Which of the following is the most appropriate next step in management of this patient's condition? A. Antibiotic therapy B. Beta-hCG and alpha fetoprotein assay C. CT scan of the abdomen and pelvis D. MRI of the pelvis and scrotum E. Scrotal ultrasound

Answer E Educational objective: Testicular cancer is the most common solid tumor in young men, and typically presents with a painless testicular mass. Scrotal ultrasound is the initial diagnostic test of choice as it can quickly differentiate intrinsic from extrinsic lesions and determine if testicular cancer is likely.

41. An 18-year-old woman comes to the office for evaluation of a 3-day history of vaginal discharge. The patient has 2 sexual partners and uses condoms inconsistently. Examination shows a frothy, malodorous vaginal discharge and erythema of the vulva and vaginal mucosa. Which of the following is the most appropriate treatment recommendation for this patient? A. Ceftriaxone and doxycycline for the patient and partners B. Clindamycin for the patient only C. Doxycycline for the patient and partners D. Fluconazole for the patient only E. Metronidazole for the patient and partners

Answer E Educational objective: Trichomonas vaginalis infection can be asymptomatic or cause yellow-green, frothy vaginal discharge. Patients can have associated vulvovaginal pruritus, erythema, dysuria, and dyspareunia. Diagnosis is with wet mount microscopy (ie, visible, flagellated, motile protozoan) and treatment is with oral metronidazole for the patient and sexual partners to prevent recurrence.

20. A 20-year-old woman comes to the physician with pain, itching, and malodorous discharge from her left ear. She has been swimming frequently and first noticed the itching a few days ago. The patient has no other symptoms. She does not wear earrings and has no known skin disorders. Vital signs are within normal limits. There is no warmth or redness around the ear and no pain to percussion of the mastoid area, although she has pain on movement of the tragus. There is a moderate amount of debris blocking the auditory canal. Removal of the debris shows an erythematous and edematous external canal. The tympanic membrane appears intact and mobile with a pneumatic otoscope. Which of the following is the most appropriate management for this patient? A. Cultures of the debris B. Oral ciprofloxacin C.Oral linezolid D. Referral to an otolaryngologist E. Topical ciprofloxacin and hydrocortisone

Answer E Educational objective: Uncomplicated otitis externa is treated by cleaning the ear canal and applying a topical glucocorticoid solution. Moderate to severe cases should also be treated with a topical antibiotic. Patients who are immunocompromised or have involvement of the deep tissues will require referral for more aggressive therapy.

11. A 2-year-old boy comes to the emergency department with nasal discharge. The discharge was clear 2 days ago but is now purulent and foul smelling. He has nasal pain but no fever or cough. The patient's other medical problems include eczema and reactive airway disease. Examination shows a well-appearing child with foul-smelling, purulent discharge from the right nostril and no facial tenderness to palpation. Which of the following is the most appropriate next step in management of this patient? A. Intranasal fluticasone B. Oral amoxicillin-clavulanate C. Oral pseudoephedrine D. Sinus CT scan E. Visualization of posterior nasal cavity

Answer E Educational objective: Unilateral purulent, foul-smelling discharge in a young child should raise suspicion for a nasal foreign body. Visualization of the posterior nasal cavity is necessary to make the diagnosis. Positive pressure and mechanical extraction techniques can be used to remove nasal foreign bodies.

2. A 16-year-old boy is brought to the office for evaluation of "heaviness" in his left hemiscrotum. Initial examination in the standing position shows normal external male genitalia. When the patient performs a Valsalva maneuver, a soft, irregular, nontender mass is palpated in the left hemiscrotum. The left hemiscrotum does not transilluminate. These findings disappear when the patient is supine. Which of the following is the most likely diagnosis in this patient? A. Hydrocele B. Inguinal hernia C. Spermatocele D. Testicular cancer E. Varicocele

Answer E Educational objective: Varicoceles are caused by dilation of the veins of the pampiniform plexus and may present with scrotal heaviness. The classic finding is a palpable, soft, irregular mass on scrotal examination that worsens with the Valsalva maneuver.

9. A large, randomized, placebo controlled study evaluated the efficacy of a statin in preventing myocardial infarction in patients with moderate hyperlipidemia and no history of coronary artery disease. Patients who took the statin had a relative risk of 0.67 of having a myocardial infarction. The absolute risk was 7.5% in the placebo group and 5.0% in the treatment group. What was the relative risk reduction? A. 2.5% B. 33% C. 40% D. 50% E. 67%

Answer B Educational objective: Relative risk reduction (RRR) can be calculated in 2 equivalent ways: 1. RRR = (risk in unexposed - risk in exposed) / (risk in unexposed) 2. RRR = 1 - relative risk

10. A previously healthy 1-year-old girl is brought to the emergency department for evaluation of unresponsiveness after crying. Her parents say that the girl had a tantrum and was crying incessantly when they put her in her crib at bedtime. The girl emitted a long, drawn out scream after which she turned "blue" and went limp and unresponsive for "a few seconds." Subsequently, she was alert and acted like her usual self. Vital signs and physical examination are unremarkable. A recent complete blood count was normal. Which of the following is the best next step in management of this patient? A. Brain MRI B. Echocardiogram C. Electrocardiogram D. Electroencephalogram E. Reassurance

Answer E Educational objective: Breath-holding spells are generally benign and characterized by a brief period of apnea and skin color change associated with an emotional trigger. Patients should be evaluated for iron-deficiency anemia, but additional workup is usually unnecessary.

1. A 26-year-old man comes to the outpatient clinic with a 1-week history of right-ear pain. The pain is "nagging" and "boring" and increases in severity when he chews food. He has intermittent headaches but no ear discharge, sinus tenderness, or skin rash. He has a history of otitis media as a child. His past medical history is otherwise insignificant. His girlfriend says that he grinds his teeth in his sleep. He is an avid swimmer. On examination, his ears are normal with only a mild amount of wax. Pulling on the pinna does not elicit pain. There are no hearing deficits appreciated. Mobility of the tympanic membrane is normal and the Weber and Rinne test results are within normal limits. What is the most likely diagnosis in this patient? A. Glossopharyngeal neuralgia B. Otitis externa C. Otitis media D. Ramsay Hunt syndrome E. Temporomandibular joint dysfunction

Answer E Educational objective: Temporomandibular joint dysfunction can result in referred pain to the ear that worsens with chewing. Most patients report a history of nocturnal teeth grinding.

44. A 67-year-old man comes to the physician to discuss syncope. He felt lightheaded and passed out briefly while jogging in the park the previous day. He says that he is physically active and likes outdoor activities but over the last year has been somewhat limited by exertional shortness of breath, fatigue, and lightheadedness. His past medical history is significant for diet-controlled diabetes mellitus. Which of the following physical examination findings is most likely in this patient? A. Capillary pulsations B.Late diastolic murmur C.Pleural friction rub D. Pulsus paradoxus E. Systolic ejection murmur

Answer E Educational objective: The typical symptoms of aortic stenosis are exertional dyspnea, syncope, and angina. Physical examination will show a systolic ejection murmur radiating to the apex and carotid arteries.


Related study sets

EXAM 1: History of Life, Evolution & Speciation

View Set

Baroreceptor and Chemoreceptor Reflexes

View Set

Chapter 18 Nursing Management of the Newborn

View Set

gov ap exam quizlet (-chapters 6, 7, 13, 14, 15, & 16)

View Set

Criminological Theory: What Explains Crime?

View Set

NCLEX Practice Questions FUNDS Exam 1

View Set